You are on page 1of 154

Generalitat de Catalunya

Departament d’Educació
Institut Gelida Departament de Matemàtiques

RECULL DE PROBLEMES DE SELECTIVITAT SOBRE GEOMETRIA AFÍ I MÈTRICA


(AMB SOLUCIÓ)

1) PAU LOGSE 1999 Sèrie 1 Problema 1:


Donat el tetràedre de vèrtexs A = (0, 0, 0), B = (1, 1, 1), C = (3, 0, 0) i D = (0, 3, 0)
a) Calculeu l'equació del pla que conté la cara BCD i la del pla que conté la cara
ACD.
b) Calculeu les equacions de dues de les altures del tetràedre, la que passa pel vèrtex
A i la que passa pel vèrtex B, respectivament. (Nota: altura d'un tetràedre és la recta
que passa per un vèrtex i és perpendicular al pla que determina la cara oposada.)
c) Comproveu que les dues altures anteriors es tallen en un punt P.
d) Comproveu si la recta que uneix qualsevol vèrtex del tetràedre amb P és
perpendicular a la cara oposada (i és, per tant, una altura del tetràedre).
Solució

2) PAU LOGSE 1999 Sèrie 2 Qüestió 2:


x−3 z +1
Considereu la recta r de l'espai d'equacions =y=
2 2
Trobeu l'equació cartesiana del pla que conté r i que passa pel punt P = (1, 1, 1)
(equació cartesiana vol dir la de la forma ax + by + cz = d).
Solució

3) PAU LOGSE 1999 Sèrie 2 Problema 2:


Donats els punts de l'espai A = (2, 1, 0), B = (0, 2, 0), C = (–3, 0, 0) i D = (0, –1, 0)
a) Són coplanaris? Formen un paral·lelogram?
b) Calculeu l'àrea del polígon ABCD.
c) Calculeu el punt simètric del punt E = (1, 1, 2) respecte del pla que determinen A, B i
C.
d) Calculeu la distància entre la recta que passa per E i A i la recta que passa per B i C.
Solució

4) PAU LOGSE 1999 Sèrie 5 Qüestió 2:


4 x − y − z = 0 y
Donades les rectes r1 :  i r2 : x = = z
2 x + y − 2 z − 1 = 0 3
Calculeu l'equació del pla paral·lel a les dues rectes que passa per l'origen.
Solució

5) PAU LOGSE 1999 Sèrie 6 Qüestió 4:


x −1 x − 2z = 5
Considereu les rectes r : = y = z −2 i s: :
2  x − 2 y = 11
Comproveu que aquestes dues rectes són paral·leles i calculeu l'equació del pla que
les conté.
Solució

C/ Pasqual i Batlle, 1-15 08790 Gelida Telèfon: 93 779 04 50 Pàg. 1


e-mail: a8035246@xtec.cat https://agora.xtec.cat/iesgelida
Generalitat de Catalunya
Departament d’Educació
Institut Gelida Departament de Matemàtiques

6) PAU LOGSE 2000 Sèrie 1 Problema 2:


x = 2z − 3
Donats el pla  d'equació x + 2y + 3z – 1 = 0, la recta r d'equacions  i el
y = z + 4
punt P = (2, 1, 1), calculeu:
a) Unes equacions de la recta que passa per P i és perpendicular a  .
b) L'equació del pla que passa per P i és perpendicular a la recta r.
c) Unes equacions de la recta que passa per P i talla perpendicularment r.
d) Unes equacions de la recta que passa per P, és paral·lela al pla  i tal que el seu
vector director és perpendicular al de r.
Solució

7) PAU LOGSE 2000 Sèrie 2 Problema 2:


2 x − 5 y − z − 3 = 0
Considereu la recta  i el pla 2x – y + az + 2 = 0, on a és un
x − 3y − z − 2 = 0
paràmetre.
a) Per a quin valor de a la recta i el pla són paral·lels? Quina serà llavors la distància
entre el punt P = (1, 0, –1) de la recta i el pla?
b) Existeix algun valor de a per al qual la recta i el pla siguin perpendiculars?
c) Determineu el valor de a perquè la recta i el pla formin un angle de 30º.
Solució

8) PAU LOGSE 2000 Sèrie 3 Problema 2:


Un quadrat de l'espai té tres dels seus vèrtexs consecutius situats en els punts de
coordenades enteres P = (3, –2, 4), Q = (a, –1, a + 1) i R = (2, –3, 0).
a) Tenint en compte que els vectors QP i QR han de ser perpendiculars, calculeu el
valor del nombre enter a.
b) Calculeu l'equació del pla que conté aquest quadrat.
c) Calculeu el quart vèrtex d'aquest quadrat.
d) Calculeu l'àrea d'aquest quadrat.
Solució

9) PAU LOGSE 2000 Sèrie 6 Qüestió 3:


Calculeu el peu de la recta perpendicular a la recta ( x, y, z ) = (1, −1,1) +  ( 0,1,1)
traçada des del punt (1, 0, –1).
Solució

10) PAU LOGSE 2000 Sèrie 6 Problema 1:


Considereu la recta r de l'espai que passa pel punt P = (1, 1, 3) i té per vector director
v = (1 − a, a, 1) . Sigui  el pla que té per equació 2x + y – z = 1.
a) Determineu per a cada valor del paràmetre a la posició relativa de la recta r
respecte al pla  (paral·lela, continguda o amb un punt d'intersecció).
b) Hi ha alguna de les rectes r que sigui perpendicular al pla  ?
c) Calculeu la distància que hi ha entre el punt P i el pla  .
Solució

C/ Pasqual i Batlle, 1-15 08790 Gelida Telèfon: 93 779 04 50 Pàg. 2


e-mail: a8035246@xtec.cat https://agora.xtec.cat/iesgelida
Generalitat de Catalunya
Departament d’Educació
Institut Gelida Departament de Matemàtiques

11) PAU LOGSE 2001 Sèrie 2 Qüestió 3:


Donats els punts de l'espai A = (2, 0, 0), B = (0, 1, 0) i C = (0, 0, 3).
a) Determineu l'equació del pla  que els conté.
b) Calculeu l'equació de la recta r perpendicular al pla  i que passa per
l'origen.
Solució

12) PAU LOGSE 2001 Sèrie 4 Qüestió 1:


Determineu per a quins valors del paràmetre a el pla  : ax + 2y + z = a és paral·lel a la
 x − ay + z = 1
recta r : 
ax + z = a + 1
Solució

13) PAU LOGSE 2001 Sèrie 4 Problema 2:


Sigui  el pla d'equació x – y + 2z = 3 i P el punt (1, 1, 0).
a) Calculeu la distància d de P a  .
b) Determineu l'equació de l'altre pla  ' paral·lel a  que també dista d del punt P.
c) Determineu l'equació de la recta r perpendicular a  que passa per P.
d) Calculeu la intersecció de la recta r amb el pla  .
Solució

14) PAU LOGSE 2001 Sèrie 5 Problema 1:


x − 2 y + 3 z +1
Considereu a l'espai la recta r d'equacions = = i la recta s
2 −3 −1
x + 4 y −1 z + 4
d'equacions = =
−2 3 1
a) Determineu el punt de tall de la recta r amb el pla z = 0.
b) Comproveu que les rectes r i s són paral·leles i calculeu la distància entre elles.
c) Quina és l'equació del pla que conté les dues rectes?
d) Calculeu la distància del pla anterior a l'origen de coordenades.
Solució

15) PAU LOGSE 2002 Sèrie 1 Qüestió 3:


Comproveu que la recta que passa pels punts A = (4, 0, 0) i B = (0, 2, 2) és paral·lela al
pla d'equació x – 3y + 5z = 2, i calculeu la distància entre la recta i el pla.
Solució

C/ Pasqual i Batlle, 1-15 08790 Gelida Telèfon: 93 779 04 50 Pàg. 3


e-mail: a8035246@xtec.cat https://agora.xtec.cat/iesgelida
Generalitat de Catalunya
Departament d’Educació
Institut Gelida Departament de Matemàtiques

16) PAU LOGSE 2002 Sèrie 1 Problema 2:


Considerem el cub de vèrtexs A, B, C, D, E, F, G, H que té l'aresta de longitud 4 dm.

a) Determineu l'equació del pla inclinat EHBC si prenem


com a origen de coordenades el vèrtex D i
com a eixos de coordenades DA, DC i DH en aquest ordre, tenint en compte que el
sentit positiu de cada un d'ells és el que sortint de l'origen D va cap a A, C i H,
respectivament.
b) Calculeu les equacions de les diagonals CE i AG i utilitzeu-les per calcular les
coordenades del seu punt d'intersecció.
Solució

17) PAU LOGSE 2002 Sèrie 2 Qüestió 4:

Calculeu l'angle que forma el pla x – 2y + z = 1 amb la recta determinada per les
x = t

equacions  y = 1 + t
z = 2

Solució

18) PAU LOGSE 2002 Sèrie 2 Problema 1:


Considereu les rectes r i s amb les equacions següents:
x − y + 3 = 0  y + 13 = 0
r:  s: 
2 x − z + 2 = 0  x − 2 z − 3 = 0
a) Calculeu, de cada una de les rectes, un punt i un vector director.
b) Determineu si existeix cada un dels objectes següents i en cas afirmatiu calculeu la
seva equació:
i) El pla paral·lel a la recta s que conté la recta r.
ii) El pla perpendicular a la recta s que conté la recta r.
iii) La recta perpendicular a les rectes r i s que passa per (0, 0, 0).
Solució

19) PAU LOGSE 2002 Sèrie 3 Qüestió 3:


Considereu els plans d'equacions: 1 : x + 2 y − z = 3 i  2 : ax + ( a − 2) y + 2 z = 4 .
a) Hi ha algun valor del paràmetre a per al qual la intersecció dels plans 1 i  2 no és
una recta?
b) Calculeu un vector director de la recta que s'obté quan es fa la intersecció de 1 i
2 per al valor del paràmetre a = 0.
Solució

C/ Pasqual i Batlle, 1-15 08790 Gelida Telèfon: 93 779 04 50 Pàg. 4


e-mail: a8035246@xtec.cat https://agora.xtec.cat/iesgelida
Generalitat de Catalunya
Departament d’Educació
Institut Gelida Departament de Matemàtiques

20) PAU LOGSE 2002 Sèrie 3 Qüestió 4:


y −5 z −7
Considereu la recta r d'equacions: x − 1 = = . Calculeu els punts d'aquesta
−3 −4
recta situats a una distància 3 del punt A = (1, 0, 1).
Solució

21) PAU LOGSE 2003 Sèrie 2 Qüestió 3:


x −1 y +1 z
Considereu el punt P = (5, –2, 9) i la recta r : = = .
−2 −3 6
a) Calculeu l’equació de la recta s que talla perpendicularment r i que passa per P.
b) Calculeu el punt de tall Q entre les rectes r i s.
Solució

22) PAU LOGSE 2003 Sèrie 3 Qüestió 4:


Considereu els punts de l’espai A = (0, –2a – 1, 4a – 2), B = (1, –3, 4), C = (3, –5, 3).
a) Comproveu que el triangle de vèrtexs A, B i C és rectangle en B per a qualsevol
valor de a.
b) Calculeu els valors de a que fan que aquest triangle sigui isòsceles.
Solució

23) PAU LOGSE 2003 Sèrie 3 Problema 2:


Un segment d’extrems A = (5, 3, 1) i B = (4, 2, –1) es divideix en tres parts iguals
mitjançant dos plans perpendiculars a aquest segment. Calculeu les equacions dels
dos plans i la distància entre ells.
Solució

24) PAU LOGSE 2003 Sèrie 5 Qüestió 2:


y
Determineu l'equació del pla que conté a la recta x − 1 = = z + 1 i passa per l'origen
2
de coordenades.
Solució

25) PAU LOGSE 2004 Sèrie 1 Qüestió 2:


Considereu els punts de l’espai A(0, 0, 1), B(1, 1, 2) i C(0, –1, –1).
a) Trobeu l’equació del pla ABC.
b) Si D és el punt de coordenades (k, 0, 0), quant ha de valer k per tal que els quatre
punts A, B, C i D siguin coplanaris?
Solució

26) PAU LOGSE 2004 Sèrie 3 Problema 2:


Tenim quatre punts a l’espai: A(0, 0, 0); B(0, 0, 2); C(0, 2, 0) i D(2, 0, 0). Es demana:
a) representeu gràficament els quatre punts;
b) calculeu el volum del tetràedre (piràmide de base triangular) ABCD;
c) trobeu l’equació del pla que passa per B, C i D;
d) calculeu la distància de l’origen al pla de l’apartat anterior.
Solució

C/ Pasqual i Batlle, 1-15 08790 Gelida Telèfon: 93 779 04 50 Pàg. 5


e-mail: a8035246@xtec.cat https://agora.xtec.cat/iesgelida
Generalitat de Catalunya
Departament d’Educació
Institut Gelida Departament de Matemàtiques

27) PAU LOGSE 2004 Sèrie 4 Qüestió 1:


Considereu els punts de l’espai A(1, 1, 2), B(0, 1, 1) i C(k, 1, 5).
a) Trobeu l’equació de la recta que passa per A i B.
b) Per a quins valors de k els punts A, B i C formen un triangle?
Solució

28) PAU LOGSE 2004 Sèrie 4 Problema 2:


 x = −3 + 2t

Considereu la recta r d’equació  y = 5 − 2t i el punt M (2, 3, 7).
z = 3 + t

a) Trobeu, en funció de t, la distància de M a un punt qualsevol de la recta r.
b) Trobeu les coordenades dels punts A i B de r situats a distància 3 2 del punt M.
c) El triangle AMB, és rectangle en M?
d) Els punts A i B formen part d’un paral·lelogram de vèrtexs ABCD que té el centre de
simetria en el punt M. Calculeu les coordenades de C i D.
Solució

29) PAU LOGSE 2004 Sèrie 5 Problema 2:


 x = 1 + 3t
x − 2 y +1 z 
Considereu les rectes r : = = i s :  y = −1 − 4t
−2 1 −2 z = 5 + t

a) Estudieu la seva posició relativa.
b) Trobeu l’equació del pla que conté s i és paral·lel a r.
c) Calculeu la distància entre r i s.
Solució

30) PAU LOGSE 2005 Sèrie 1 Qüestió 3:


x − 3 y −1 z + 2
Trobeu la distància entre la recta r : = = i el pla  : 3x + 4y + 7 = 0.
4 −3 3
Solució

31) PAU LOGSE 2005 Sèrie 3 Qüestió 3:


x − 3 y −1 z + 2
Trobeu la distància entre la recta r : = = i el pla  : 2x – 3y + 3z + 5 = 0.
2 −3 3
Solució

C/ Pasqual i Batlle, 1-15 08790 Gelida Telèfon: 93 779 04 50 Pàg. 6


e-mail: a8035246@xtec.cat https://agora.xtec.cat/iesgelida
Generalitat de Catalunya
Departament d’Educació
Institut Gelida Departament de Matemàtiques

32) PAU LOGSE 2005 Sèrie 3 Qüestió 4:


Donats els punts A = (1, 0, 0) i B (0, 0, 1):
x = 1

a) Trobeu un punt C sobre la recta d’equació paramètrica  y = 1 +  que faci que el
z = 1+ 

triangle ABC sigui rectangle en C.
b) Trobeu l’àrea del triangle ABC.
Solució

33) PAU LOGSE 2005 Sèrie 4 Problema 2:


Una piràmide de base quadrada té el vèrtex en el pla d’equació z = 3. Tres dels vèrtexs
de la base són els punts del pla OXY: A = (1, 0, 0), B = (1, 1, 0) i C = (0, 1, 0).
a) Feu un gràfic dels elements del problema. Quines són les coordenades del quart
vèrtex de la base, D?
 àrea base  altura 
b) Quin és el volum de la piràmide? Volum = 
 3
c) Si el vèrtex de la piràmide és el punt V = (a, b, 3), quina és l’equació de la recta que
conté l’altura sobre la base?
Solució

34) PAU LOGSE 2006 Sèrie 1 Qüestió 1:


Trobeu les coordenades dels punts situats sobre la recta d’equació
(x,y,z) = (–1,1,1) + t · (1,2,1) que estan a distància 1 del pla 2x + 2y + z = 5.
Solució

35) PAU LOGSE 2006 Sèrie 1 Problema 1:


Una recta r passa pel punt A = (3,0,2) i té la direcció del vector (–1,1,4).
a) Trobeu quin angle forma r amb el pla horitzontal.
b) Comproveu que no passa pel punt B = (1,3,10).
c) Trobeu l’equació de la recta que passa per A i B.
Solució

36) PAU LOGSE 2006 Sèrie 3 Qüestió 3:


x − y −1 = 0
Determineu l’equació del pla perpendicular a la recta r: que passa pel
x + z + 2 = 0
punt (1,1,2). Quina distància hi ha d’aquest pla a l’origen de coordenades?
Solució

37) PAU LOGSE 2006 Sèrie 3 Problema 2:


2 x − 5 y − z − 3 = 0
Considereu la recta r: i el pla  : 2x – y + az + 2 = 0 on a és un
x − 3y − z − 2 = 0
paràmetre.
a) Trobeu un vector director de la recta i un vector perpendicular al pla.
b) Quin ha de ser el valor de a per tal que la recta i el pla siguin paral·lels?
c) Esbrineu si existeixen valors de a per als quals la recta i el pla siguin perpendiculars.
En cas afirmatiu, calculeu-los.
d) Esbrineu si existeixen valors de a per als quals la recta i el pla formin un angle de 30º.
En cas afirmatiu, calculeu-los.
Solució

C/ Pasqual i Batlle, 1-15 08790 Gelida Telèfon: 93 779 04 50 Pàg. 7


e-mail: a8035246@xtec.cat https://agora.xtec.cat/iesgelida
Generalitat de Catalunya
Departament d’Educació
Institut Gelida Departament de Matemàtiques

38) PAU LOGSE 2006 Sèrie 4 Qüestió 3:


x + y − z = 0
Calculeu l’equació de la recta paral·lela a la recta r: i que passa pel
2 x − y + z = 1
punt (0,1,0).
Solució

39) PAU LOGSE 2006 Sèrie 4 Qüestió 4:


Determineu els extrems d’un segment AB sabent que el punt A pertany al pla
x −1 y − 2 z
2 x + y + z = 0 , el punt B pertany a la recta = = i el punt mitjà del segment
2 −1 3
és ( 0,0,0)
Solució

40) PAU LOGSE 2007 Sèrie 1 Qüestió 2:


Considereu els punts de l’espai P = (–1, a – 1, 3), Q = (0, a – 2, 1 – a) i R = (2, –1, 6 – 6a).
a) Trobeu el valor de a per al qual els tres punts estan alineats.
b) Quan els tres punts estan alineats, quina és l’equació de la recta que els conté?
Solució

41) PAU LOGSE 2007 Sèrie 1 Qüestió 4:


Trobeu l’equació de la recta continguda en el pla  : x + 2 y + 6 z − 2 = 0 , que talla els
eixos OY i OZ.
Solució

42) PAU LOGSE 2007 Sèrie 1 Problema 1:


y − 2 z −1
Considereu la recta d’equació r : x = = .
2 2
a) Expresseu el quadrat de la distància d’un punt qualsevol (x, y, z) de la recta al punt
P = (1, 2, 5) com una funció de la coordenada x.
b) Trobeu quin valor de x fa mínima aquesta funció, deduïu quin punt Q de la recta és
el més proper a P i calculeu la distància del punt a la recta.
c) Escriviu l’equació de la recta que passa per P i Q i comproveu que és perpendicular
a r.
Solució

43) PAU LOGSE 2007 Sèrie 2 Qüestió 1:


x + y + z = 1
Trobeu l’equació del pla perpendicular a la recta  que passa per l’origen
2 x + y = 3
de coordenades.
Solució

C/ Pasqual i Batlle, 1-15 08790 Gelida Telèfon: 93 779 04 50 Pàg. 8


e-mail: a8035246@xtec.cat https://agora.xtec.cat/iesgelida
Generalitat de Catalunya
Departament d’Educació
Institut Gelida Departament de Matemàtiques

44) PAU LOGSE 2007 Sèrie 2 Qüestió 4:


Trobeu els punts de la recta r : x − 1 = y + 2 = z que equidisten dels plans
 1 : 4 x − 3z − 1 = 0 i  2 : 3x + 4 y − 1 = 0 .
Solució

45) PAU LOGSE 2007 Sèrie 2 Problema 2:


A l’espai es consideren els tres plans d’equacions:
1 : x + 2 y + z = 1  2 : px + y + pz = 1  3 : px + y + 2 z = 1 , on p és un paràmetre real.
a) Esbrineu per a quins valors de p els tres plans es tallen en un únic punt. Trobeu aquest
punt quan p = 1.
b) Hi ha algun valor de p que faci que la intersecció comuna sigui una recta? Si és així,
escriviu l’equació vectorial d’aquesta recta.
c) Trobeu quina és la posició relativa dels tres plans quan p = 1/2.
Solució

46) PAU LOGSE 2007 Sèrie 3 Qüestió 1:


Trobeu les equacions dels plans paral·lels a  : 2 x − y + 2 z = 3 situats a 6 unitats de
distància d’aquest.
Solució

47) PAU LOGSE 2007 Sèrie 3 Problema 1:


Una recta r és paral·lela a la recta s : x − 1 = y − 1 = z − 1 , talla en un punt A la recta
x −1 y x − 2 y −1 z
t: = = z + 1 , i en un punt B la recta l : = = .
3 2 2 2 3
a) Trobeu l’equació del pla determinat per les rectes r i t.
b) Trobeu el punt B calculant el punt d’intersecció del pla anterior amb la recta l.
c) Trobeu l’equació de la recta r.
d) Trobeu el punt A.
Solució

48) PAU LOGSE 2008 Sèrie 2 Qüestió 4:


Trobeu l’equació de la recta perpendicular al pla  : 2 x − y + z + 3 = 0 , que passa pel
punt ( −1,3, a ) del pla.
Solució

49) PAU LOGSE 2008 Sèrie 2 Problema 2:


x − 2 y +1 z x −1 y + 7 z + 5
Donades les rectes r : = = i s: = = i el punt P = (1,1, −1) ,
1 2 −1 1 2 3
volem trobar l’equació de la recta que passa per P i que talla r i s. Per aconseguir-ho:
a) Trobeu l’equació general o cartesiana (és a dir, l’equació de la forma Ax + By + Cz +
D = 0) del pla  que conté la recta r i el punt P.
b) Trobeu el punt M calculant el punt d’intersecció del pla  amb la recta s.
c) Trobeu l’equació de la recta que passa pels punts P i M.
d) Comproveu que la recta trobada en l’apartat anterior és la que busquem.
Solució

C/ Pasqual i Batlle, 1-15 08790 Gelida Telèfon: 93 779 04 50 Pàg. 9


e-mail: a8035246@xtec.cat https://agora.xtec.cat/iesgelida
Generalitat de Catalunya
Departament d’Educació
Institut Gelida Departament de Matemàtiques

50) PAU LOGSE 2008 Sèrie 4 Qüestió 4:


3x − 2 y + 2 z = 7
Donats el punt P = ( 7,5,1) , el pla  : x − 2 y − 3z = 10 i la recta r: :
x − 6 y − 2z = 5
a) Trobeu la distància del punt P al pla  .
b) Trobeu la distància del punt P a la recta r.
c) Trobeu la distància de la recta r al pla  .
Solució

51) PAU LOGSE 2008 Sèrie 4 Problema 2:


x − a y z +1 x + 2 y −b z −4
Les rectes r1 : = = i r2 : = = són coplanàries (és a dir,
2 1 4 1 2 −1
estan incloses en un mateix pla).
a) Expliqueu, raonadament, quina és la posició relativa d’aquestes rectes.
b) Trobeu la relació que hi ha entre els paràmetres a i b.
c) Trobeu els valors de a i b si el pla que les conté passa pel punt P = (2, 4, 6).
Solució

52) PAU LOGSE 2008 Sèrie 5 Qüestió 4:

x −1 y +1 z + 2
Donats el pla  : 3 x − 2 y + 5 z = 6 i la recta r : = = , busqueu el punt de
2 1 −3
tall, si existeix.
Solució

53) PAU LOGSE 2009 Sèrie 1 Qüestió 2:


3
Considereu en l’espai les rectes r i s, les equacions respectives de les quals són:
 x + 2 y + mz = 0
r : ( x, y, z ) = ( 4,1,0) +  ( m,1,1) i s :  en què m és un paràmetre real.
x + y + z = 1
Estudieu si hi ha cap valor d’aquest paràmetre per al qual les rectes siguin
perpendiculars i es tallin.
Solució

54) PAU LOGSE 2009 Sèrie 3 Qüestió 4:


Donats el pla  : x + 2 y − z = 0 i el punt P = ( 3, 2,1)
a) Calculeu l’equació contínua de la recta r que passa per P i és perpendicular a  .
b) Calculeu el punt simètric del punt P respecte del pla  .
Solució

55) PAU LOGSE 2009 Sèrie 3 Problema 2:


3
Siguin P = (3 – 2a, b, –4), Q = (a – 1, 2 + b, 0) i R = (3, –2, –2) tres punts de l’espai .
a) Calculeu el valor dels paràmetres a i b per als quals aquests tres punts estiguin
alineats.
b) Trobeu l’equació contínua de la recta que els conté quan estan alineats.
c) Quan b = 0, trobeu els valors del paràmetre a perquè la distància entre els punts P i
Q sigui la mateixa que la distància entre els punts P i R.
d) Si b = 0, calculeu el valor del paràmetre a perquè els punts P, Q i R determinin un
triangle equilàter.
Solució

C/ Pasqual i Batlle, 1-15 08790 Gelida Telèfon: 93 779 04 50 Pàg. 10


e-mail: a8035246@xtec.cat https://agora.xtec.cat/iesgelida
Generalitat de Catalunya
Departament d’Educació
Institut Gelida Departament de Matemàtiques

56) PAU LOGSE 2009 Sèrie 4 Qüestió 1:

x −1 y + 2 z − 5
Donats el punt P = (1, 2,3) i la recta r : = =
2 3 −1
a) Trobeu l’equació cartesiana (és a dir, de la forma Ax + By + Cz + D = 0) del pla 
que passa per P i és perpendicular a la recta r.
b) Trobeu el punt de tall entre la recta r i el pla  .
Solució

57) PAU LOGSE 2009 Sèrie 4 Problema 2:


Siguin r i s dues rectes de l’espai les equacions respectives de les quals, que depenen
bx + y + 3z = 1  x y − b z +1
d’un paràmetre real b, són les següents: r: , s: = =
 x + 2 y + 5z = 1 1 b +1 −1
a) Trobeu el punt de tall de la recta r amb el pla d’equació x = 0 i el punt de tall de la
recta s amb aquest mateix pla.
b) Calculeu un vector director per a cada una de les dues rectes.
c) Estudieu la posició relativa de les dues rectes en funció del paràmetre b.
Solució

58) PAU LOGSE 2010 Sèrie 1 Qüestió 1:


Trobeu l’equació general (és a dir, de la forma Ax + By + Cz + D = 0) del pla que conté
x −1 x − y − z = 0
la recta r1 : = y = 2 − z i és paral·lel a la recta r2 :  .
2 x − 2 y + z = 0
Solució

59) PAU LOGSE 2010 Sèrie 1 Qüestió 4:


x + 5 y −1 z − 2 2 x + y + 2 z + 5 = 0
Donades les rectes r1 : = = i r2 :  :
3 2 −4 2 x − y + z + 11 = 0
a) Comproveu que són paral·leles.
b) Trobeu l’equació general (és a dir, de la forma Ax + By + Cz + D = 0) del pla que les
conté.
Solució

60) PAU LOGSE 2010 Sèrie 2 Qüestió 2:


ax − y = 2
Donats el pla  : 5 x + y + 3 z = 4 i la recta r: , estudieu-ne la posició
2 y + z = −3
relativa en funció del paràmetre a.
Solució

61) PAU LOGSE 2010 Sèrie 2 Qüestió 6:


x + 4 y −1
Considereu la recta r : = = z −1 .
−2 −1
a) Trobeu els dos punts, A i B, de la recta r que estan situats a una distància d= 6 del
punt P = (–1, 1, 2).
b) Trobeu l’àrea del triangle de vèrtexs A, B i P.
Solució
C/ Pasqual i Batlle, 1-15 08790 Gelida Telèfon: 93 779 04 50 Pàg. 11
e-mail: a8035246@xtec.cat https://agora.xtec.cat/iesgelida
Generalitat de Catalunya
Departament d’Educació
Institut Gelida Departament de Matemàtiques

62) PAU LOGSE 2010 Sèrie 4 Qüestió 1:


Donats el pla  : x + 2 y + 3 z − 4 = 0 i els punts P = (3, 1, –2) i Q = (0, 1, 2):
a) Calculeu l’equació contínua de la recta perpendicular al pla  que passa pel punt
P.
b) Calculeu l’equació general (és a dir, de la forma Ax + By + Cz + D = 0) del pla
perpendicular a  que passa pels punts P i Q .
Solució

63) PAU LOGSE 2010 Sèrie 5 Qüestió 2:


x−5 y −3 z +3
Donats el punt P = (1, 0, –2) i la recta r : = = :
2 2 −3
a) Trobeu l’equació contínua de la recta que passa pel punt P i talla
perpendicularment la recta r.
b) Calculeu la distància del punt P a la recta r.
Solució

64) PAU LOGSE 2010 Sèrie 5 Qüestió 5:


Siguin r i s dues rectes d’equacions
y−2 z−a
r : ( x, y, z ) = ( −4,3,4) + t ( 2, −1,1) i s : x + 1 = =
−1 3
a) Trobeu el valor del paràmetre a perquè aquestes rectes es tallin.
b) En el cas en què es tallen, trobeu l’equació general (és a dir, de la forma Ax + By +
Cz + D = 0) del pla que les conté.
Solució

65) PAU LOGSE 2011 Sèrie 1 Qüestió 1:


2 x − y + 3z = 2
Donada la recta r: :
x + z + 1 = 0
a) Trobeu-ne un vector director.
b) Calculeu l’equació contínua de la recta paral·lela a r que passa pel punt P=(1, 0,
−1).
Solució

66) PAU LOGSE 2011 Sèrie 1 Qüestió 5:


y − 3 1− z x+3 z +1
Siguin i r1 : x − 2 = = i r2 : = y +1 = .
2 2 2 2
a) Comproveu que r1 i r2 són perpendiculars.
b) Comproveu que es tallen mitjançant la determinació del punt de tall.
Solució

67) PAU LOGSE 2011 Sèrie 2 Qüestió 2:


2 x − y + 3z = 2 
Donada la recta  , calculeu l’equació general (és a dir, de la forma Ax
x + z +1 = 0 
+ By + Cz +D= 0) del pla perpendicular a la recta que passa pel punt P = (1, 0, –1).
Solució
C/ Pasqual i Batlle, 1-15 08790 Gelida Telèfon: 93 779 04 50 Pàg. 12
e-mail: a8035246@xtec.cat https://agora.xtec.cat/iesgelida
Generalitat de Catalunya
Departament d’Educació
Institut Gelida Departament de Matemàtiques

68) PAU LOGSE 2011 Sèrie 2 Qüestió 5:


x −1 y + 2
Considereu la recta r : = = z − a i el pla  : 2 x + y − 5 z = 5 .
3 −1
a) Estudieu la posició relativa de la recta r i el pla  en funció del paràmetre a.
b) Quan a = 3 , calculeu la distància de la recta r al pla  .
Solució

69) PAU LOGSE 2011 Sèrie 4 Qüestió 2:


Donat el pla  : 2 x + y − z = 5 :
a) Calculeu l’equació del pla paral·lel al pla  que passa pel punt P=(1, 0, −1).
b) Determineu també la distància entre el punt P i el pla  .
Solució

70) PAU LOGSE 2011 Sèrie 4 Qüestió 5:


Calculeu l’equació general (és a dir, de la forma Ax+By+Cz+D=0) dels plans que
y = 2
contenen la recta r: i que formen un angle de 45° amb el pla z = 0 .
z = 1
Solució

71) PAU LOGSE 2012 Sèrie 1 Qüestió 1:


Donats els plans 1 : 3x + y − 2 z + 15 = 0 i  2 : x + y + 2 z − 103 = 0 ,
a) Comproveu que són perpendiculars.
b) Calculeu l’equació cartesiana (és a dir, de la forma Ax+By+Cz+D=0) del pla
perpendicular a 1 i  2 , que passa pel punt P=(1, 3, 2).
Solució

72) PAU LOGSE 2012 Sèrie 1 Qüestió 6:

3x + y = 1
Siguin  : x − 3 y + 2 z = 1 i r: . Estudieu-ne la posició relativa segons
2 x − y + mz = 1
el valor del paràmetre m.
Solució

73) PAU LOGSE 2012 Sèrie 3 Qüestió 1:


Digueu per a quin valor del paràmetre m els plans 1 : x − y + mz = 1 ,  2 : x − y + z = m
i  3 : my + 2 z = 3 tenen com a intersecció una recta.
Solució

74) PAU 2012 Sèrie 3 Qüestió 3:


x + y + z = 0
Donats el pla  : x − y + 2 z − 5 = 0 i la recta r:
2 x − y + z = 10
a) Calculeu el punt d’intersecció entre el pla i la recta.
b) Trobeu l’equació contínua de la recta s continguda en el pla  , que és
perpendicular a la recta r i talla la recta r.
Solució

C/ Pasqual i Batlle, 1-15 08790 Gelida Telèfon: 93 779 04 50 Pàg. 13


e-mail: a8035246@xtec.cat https://agora.xtec.cat/iesgelida
Generalitat de Catalunya
Departament d’Educació
Institut Gelida Departament de Matemàtiques

75) PAU 2012 Sèrie 3 Qüestió 6:


Donats els punts P = (1, 0, 0), Q= (0, 2, 0), R = (0, 0, 3) i S = (1, 2, 3),
a) Calculeu l’equació cartesiana (és a dir, de la forma Ax+By+Cz+D=0) del pla que
conté els punts P, Q i R.
b) Comproveu si els quatre punts són coplanaris (és a dir, si els quatre estan continguts
en un mateix pla).
Solució

76) PAU 2012 Sèrie 3 Qüestió 6:


Considereu les rectes de l’espai següents:
x +1 z −1 x − 4 y −1 z − 2
r: = y −1 = , s: = =
2 −1 3 −1 2
a) Comproveu que són secants.
b) Calculeu l’equació contínua de la recta que les talla i que és perpendicular a totes
dues.
Solució

77) PAU 2013 Sèrie 3 Qüestió 1:


Sigui  : 3 x − 2 y + z = 10 .
a) Trobeu l’equació continua de la recta r perpendicular a  que passa pel punt
P = ( −1,3,2) .
b) Trobeu també l’equació cartesiana (és a dir, de la forma Ax + By + Cz + D = 0 del
pla 1 paral·lel a  que passa pel mateix punt P.
Solució

78) PAU 2013 Sèrie 3 Qüestió 4:


Un triangle d’àrea 3/ 2 té dos dels vèrtexs als punts P = ( 0,0,0) i Q = ( 2,0,1) . El tercer
x + y + z = 0
vèrtex, R , és un punt de la recta r :  i té la primera coordenada no
y =1
nul·la. Calculeu les coordenades del vèrtex R .
Solució

79) PAU 2013 Sèrie 4 Qüestió 5:


Donats els punts P = (1,0, −1) i Q = ( −1,2,3) , trobeu un punt R de la recta
x+3 y +4 z −3
r: = = que compleixi que el triangle de vèrtexs P , Q i R és isòsceles,
2 3 −1
en què PR i QR són els costats iguals del triangle.
Solució

80) PAU 2013 Sèrie 5 Qüestió 1:


Siguin 1  2 el pla x − 2 y − 4 z = 10 .
el pla 2 x + 3 y − z = 4 i
a) Comproveu que els plans  1 i  2 són perpendiculars.
b) Trobeu l’equació continua de la recta paral·lela als plans 1 i  2 i que passa

pel que P = ( −1,3,2) .


Solució

C/ Pasqual i Batlle, 1-15 08790 Gelida Telèfon: 93 779 04 50 Pàg. 14


e-mail: a8035246@xtec.cat https://agora.xtec.cat/iesgelida
Generalitat de Catalunya
Departament d’Educació
Institut Gelida Departament de Matemàtiques

81) PAU 2013 Sèrie 5 Qüestió 3:


Donats els punts P = (1, −1,2) , Q = ( 2,0,1) i R = ( 3,2, −1) ,
a) Trobeu l’equació cartesiana (és a dir, de la forma Ax + By + Cz + D = 0 ) del
pla que determinen.
x − 5 y −1 z − 5
b) Trobeu un punt S pertanyent a la recta r : = = , de manera
2 −1 −3
que el tetraedre de vèrtexs P , Q , R i S tinguin un volum igual a 1/ 2 .
Solució

82) PAU 2013 Sèrie 5 Qüestió 5:


Considereu els punts A = ( −1,2,4) i B = ( 3,0, −2 ) .
Trobeu l’equació del pla format per tots els punts que equidisten de A i B.
Donat un punt C = ( x, y, z ) , dividim el segment AC en tres parts iguals i obtenim els
punts A, A1 , B i C . Trobeu el punt C .
Solució

83) PAU 2013 Sèrie 1 Qüestió 3:


x −1 z+m
Donats el pla  : x + 2y − z = 3 i la recta r : =y= ,
2 4
a) Comproveu que el vector característic (o normal) de  i el vector director de r són
perpendiculars.
b) Estudieu la posició relativa de  i r en funció del paràmetre m.
Solució

84) PAU 2013 Sèrie 1 Qüestió 5:


Donats el pla  : 2 x − y + 3 z − 8 = 0 i el punt P = ( 6, −3,7 ) ,
a) Trobeu l’equació contínua de la recta que passa per P i és perpendicular a .
b) Trobeu el punt del pla  que està més proper al punt P .
Solució

85) PAU 2014 Sèrie 3 Qüestió 2:


Considereu el punt A = (1,2,3) .
a) Calculeu el punt simètric del punt A respecte de la recta d’equació
r : ( x, y, z ) = ( 3 + , 1, 3 −  ) .
b) Calculeu el punt simètric del punt A respecte del pla que té per equació
 : x + y + z = 3.
Solució

C/ Pasqual i Batlle, 1-15 08790 Gelida Telèfon: 93 779 04 50 Pàg. 15


e-mail: a8035246@xtec.cat https://agora.xtec.cat/iesgelida
Generalitat de Catalunya
Departament d’Educació
Institut Gelida Departament de Matemàtiques

86) PAU 2014 Sèrie 3 Qüestió 5:


x−2 z +1
Siguin r i s les rectes de
3
d’equacions r: =y= i
3 4
s : ( x, y, z ) = (1 + 2 , 3 −  , 4 + 3 ) , amb   .
a) Comproveu que els punts mitjans dels segments que tenen un extrem situat sobre la
recta r i l’altre extrem situat sobre la recta s formen un pla.
b) Trobeu l’equació general (és a dir, que té la forma Ax + By + Cz = D ) del pla de
l’apartat anterior.
Solució

87) PAU 2014 Sèrie 4 Qüestió 3:


Siguin els punts P = (1,1,0) , Q = (1,0,1) i R = ( 0,1,1) i el pla  :x+ y+z =4.
a) Trobeu l’equació general (es a dir, que té la forma Ax + By + Cz = D ) del pla que
passa pels punts P, Q i R.
b) Si S és un punt de  , comproveu que el volum del tetraedre de vèrtexs P, Q, R i S no
depèn del punt S.
Solució

88) PAU 2014 Sèrie 4 Qüestió 4:


Donats els plans 1 : x − 4 y + z = 2m − 1 i  2 : 2 x − ( 2m + 2) y + 2 z = 3m + 1,
a) Determineu els valors de m perquè els plans 1 i  2 s’intersequin en una recta i
calculeu un vector director de la recta resultant que no depengui de m.
b) Sigui el pla  : 3 x − 2 y + 3 z = 8 . Estudieu la posició relativa del pla  amb la recta r
definida per la intersecció dels plans 1 i  2 quan m = 1.
Solució

89) PAU 2014 Sèrie 5 Qüestió 1:


3
Siguin r i s les rectes de que tenen les equacions següents:
z −3 x − 3 y − 2 z +1
r:x+5= y −5= i s: = =
2 2 3 −1
a) Estudieu el paral·lelisme i la perpendicularitat entre les rectes r i s .
b) Trobeu l’equació general (és a dir, que té la forma Ax + By + Cz + D = 0 ) del pla 
que conté la recta r i és paral·lel a la recta s . Calculeu la distància entre la recta s i
el pla  obtingut.
Solució

90) PAU 2015 Sèrie 2 Qüestió 2:


x −1 y + 3
Sigui r la recta de l’espai que té per equació r : = = z i sigui P el punt de
2 −1
coordenades ( 6,0, −1) .
a) Trobeu l’equació cartesiana (és a dir, que té la forma Ax + By + Cz = D del pla que
passa pel punt P i talla perpendicularment la recta r .
b) Trobeu l’equació paramètrica del pla que passa pel punt P i conté la recta r .
Solució

C/ Pasqual i Batlle, 1-15 08790 Gelida Telèfon: 93 779 04 50 Pàg. 16


e-mail: a8035246@xtec.cat https://agora.xtec.cat/iesgelida
Generalitat de Catalunya
Departament d’Educació
Institut Gelida Departament de Matemàtiques

91) PAU 2015 Sèrie 2 Qüestió 4:


Considereu a
3
la recta que té per equació r : ( x, y, z ) = ( −4 + 2, −2,1 −  ) i els

plans 1 i  2 1 : x + 2 y + 2 z = −1 i  2 : x − 2 y + 2 z = −3 , respectivament.
d’equacions
a) Determineu la posició relativa de  1 i  2 .
b) Comproveu que tots els punts de la recta r estan situats a la mateixa distància dels
plans 1 i  2 .

NOTA: Podeu calcular la distància d’un punt de coordenades ( x0 , y0 , z0 ) al pla


Ax0 + By0 + Cz0 + D
d’equació Ax + By + Cz + D = 0 amb l’expressió
A2 + B 2 + C 2
Solució

92) PAU 2015 Sèrie 4 Qüestió 3:


Siguin el punt P = ( 2,0,2) i el pla  d’equació x − y + z = 1 .
a) Calculeu l’equació paramètrica de la recta que passa pel punt P i és
perpendicular al pla  .
b) Calculeu la distància del punt P al pla  .
NOTA: Podeu calcular la distància d’un punt de coordenades ( x0 , y0 , z0 ) al pla
Ax0 + By0 + Cz0 + D
d’equació Ax + By + Cz + D = 0 amb l’expressió .
A2 + B 2 + C 2
Solució

93) PAU 2015 Sèrie 4 Qüestió 6:


Siguin a
3
P = ( 2,3,3) i la recta r : ( x, y, z ) = (1,2,3) + t (1,1,1) .
el punt
a) Calculeu l’equació paramètrica del pla que passa pel punt P i conté la recta r .
b) Calculeu l’equació cartesiana (és a dir, que té la forma Ax + By + Cz = D del pla
que passa pel punt P i és perpendicular a la recta r .
Solució

94) PAU 2015 Sèrie 5 Qüestió 2:


A l’espai tridimensional considereu la recta r : ( x, y, z ) = ( 3 + 2 , − ,3 −  ) i els plans
1 : x + y + z = −1 i  2 : ( x, y, z ) = ( 2 + ,1 −  + ,  ) .
a) Calculeu l’equació cartesiana (és a dir, que té la forma Ax + By + Cz = D del pla
2.
b) Trobeu els dos punts de la recta r que equidisten dels plans 1 i  2 .
NOTA: Podeu calcular la distància d’un punt de coordenades ( x0 , y0 , z0 ) al pla
Ax0 + By0 + Cz0 + D
d’equació Ax + By + Cz + D = 0 amb l’expressió .
A2 + B 2 + C 2
Solució

C/ Pasqual i Batlle, 1-15 08790 Gelida Telèfon: 93 779 04 50 Pàg. 17


e-mail: a8035246@xtec.cat https://agora.xtec.cat/iesgelida
Generalitat de Catalunya
Departament d’Educació
Institut Gelida Departament de Matemàtiques

95) PAU 2015 Sèrie 5 Qüestió 4:


Siguin els plans de
3
1 : − y + z = 2 ,  2 : −2 x + y + z = 1 i  3 : 2 x − 2 z = −1 .
a) Calculeu la posició relativa dels tres plans.
b) Comproveu que el pla 3 és paral·lel a la recta definida per la intersecció dels
plans 1 i  2 .
Solució

96) PAU 2016 Sèrie 1 Qüestió 1:


Siguin la recta r: (x, y, z) = (5 + k, k, –2 – 2k) i els punts P = (1, 0, –1) i Q = (2, 1, 1).
a) Calculeu l’equació paramètrica de la recta que passa pel punt Q i és perpendicular
al pla determinat per la recta r i el punt P.
b) Calculeu el punt de la recta r que equidista dels punts P i Q.
Solució

97) PAU 2016 Sèrie 1 Qüestió 6:


Responeu a les qüestions següents:
a) Calculeu l’equació cartesiana (és a dir, que té la forma Ax + By + Cz = D) del pla
que passa pel punt de coordenades (0, 0, 1) i és perpendicular als plans 3x + y – z = 1 i x
+ y + 2z = 5.
b) Suposeu que un pla 1 és perpendicular a un segon pla 2 i que el pla 2 és a la
vegada perpendicular a un tercer pla  3 . Expliqueu raonadament si necessàriament
els plans 1 i  3 han de ser perpendiculars entre ells.
Solució

98) PAU 2016 Sèrie 3 Qüestió 2:

A
3
, siguin la recta r que té per equació ( x, y, z ) = (1 + , , 1 −  ) i el pla 
d’equació 2 x − y + z = −2 .
a) Determineu la posició relativa de la recta r i el pla .
b) Calculeu la distància entre la recta r i el pla  .
Nota: Podeu calcular la distància d’un punt de coordenades ( x0 , y0 , z0 ) al pla
Ax0 + By0 + Cz0 + D
d’equació Ax + By + Cz + D = 0 amb l’expressió .
A2 + B 2 + C 2
Solució

99) PAU 2016 Sèrie 5 Qüestió 2:


Siguin a
3
el pla  d’equació x − y + 2 z = 2 i els punts A = ( 3, −1,2 ) i B = (1,1, −2 ) .
a) Comproveu que els punts A i B són simètrics respecte del pla  .
b) Si r és la recta dels punts P que té per equació P = B + v , en què  és un
paràmetre real i v = (1,1,0 ) , verifiqueu que els punts mitjans dels segments AP
pertanyen al pla .
Solució

C/ Pasqual i Batlle, 1-15 08790 Gelida Telèfon: 93 779 04 50 Pàg. 18


e-mail: a8035246@xtec.cat https://agora.xtec.cat/iesgelida
Generalitat de Catalunya
Departament d’Educació
Institut Gelida Departament de Matemàtiques

100) PAU 2016 Sèrie 5 Qüestió 5:


x−3
Siguin les rectes r : ( x, y, z ) = ( 2,3, −3) +  (1, −1,0 ) i s : = y −5= z +2.
2
a) Estudieu si les rectes r i s són paral·leles o perpendiculars.
b) Determineu la posició relativa de les rectes r i s i calculeu l’equació paramètrica
de la recta t que talla perpendicularment la recta r i la recta s .
Solució

101) PAU 2017 Sèrie 1 Qüestió 2:


a) Considereu els plans 1 : 5 x − y − 7 z = 1 i  2 : 2 x + 3 y + z = 5 .
Determineu l’equació general (és a dir, la que té la forma Ax + By + Cz = D ) del pla
que passa per l’origen de coordenades i és perpendicular als plans 1 i  2 .
b) Calculeu l’angle que formen els plans 1 i  2 .
Solució

102) PAU 2017 Sèrie 2 Qüestió 1:


Considereu el pla  : x + y + z =1 i la recta r que passa pels punts P = ( 0,0,6 ) i
Q = (1, 2,3)
a) Estudieu la posició relativa de la recta r i el pla  .
b) Calculeu la distància entre la recta r i el pla  .
NOTA: Podeu calcular la distància d’un punt de coordenades ( x0 , y0 , z0 ) al pla
Ax0 + By0 + Cz0 + D
d’equació Ax + By + Cz + D = 0 amb l’expressió .
A2 + B 2 + C 2
Solució

103) PAU 2017 Sèrie 2 Qüestió 5:


x − z = 2
i el punt P = ( 0,1, −1) .
3
A , siguin la recta r:
2 y + z = 4
a) Calculeu l’equació general (és a dir, la que té la forma Ax + By + Cz = D ) del pla 
perpendicular a la recta r i que passa pel punt P .
b) Calculeu el punt simètric del punt P respecte del pla x + y + z = −3 .
Solució

104) PAU 2018 Sèrie 1 Qüestió 2:


Sigui r la recta que passa pels puntsA = ( 0,1,1) i B = (1,1, −1) .
a) Trobeu l’equació paramètrica de la recta r .
b) Calculeu tots els punts de la recta r que estan a la mateixa distància dels plans
1 : x + y = −2 i  2 : x − z = 1 .
NOTA: Podeu calcular la distància d’un punt de coordenades ( x0 , y0 , z0 ) al pla
Ax0 + By0 + Cz0 + D
d’equació Ax + By + Cz + D = 0 amb l’expressió .
A2 + B 2 + C 2
Solució

C/ Pasqual i Batlle, 1-15 08790 Gelida Telèfon: 93 779 04 50 Pàg. 19


e-mail: a8035246@xtec.cat https://agora.xtec.cat/iesgelida
Generalitat de Catalunya
Departament d’Educació
Institut Gelida Departament de Matemàtiques

105) PAU 2018 Sèrie 1 Qüestió 4:


Considereu els punts P = ( 3, −2,1) , Q = ( 5,0,3) , R = (1,2,3) i la recta

x + y + 1 = 0
r:
2 y + 3 z − 5 = 0
a) Determineu l’equació general (és a dir, que té la forma Ax + By + Cz = D del pla
que passa per P i Q i és paral·lel a la recta r .
b) Donats el pla x + 2 y + mz = 7 i el pla que passa per P , Q i R , trobeu m perquè
siguin paral·lels i no coincidents.
Solució

106) PAU 2018 Sèrie 3 Qüestió 3:


Considereu el pla que té com a vectors directors u = ( −1,3,2 ) i v = ( 2,1,0) i que passa
pel punt A = (1,0,3) .
a) Calculeu l’equació de la recta que és perpendicular al pla i passa pel punt A.
b) Calculeu la distància del punt P = (1,5,0 ) al pla.
NOTA: Podeu calcular la distància d’un punt de coordenades ( x0 , y0 , z0 ) al pla
Ax0 + By0 + Cz0 + D
d’equació Ax + By + Cz + D = 0 amb l’expressió .
A2 + B 2 + C 2
Solució
107) PAU 2018 Sèrie 3 Qüestió 5:
Considereu els punts de l’espai tridimensional A = (1,1,0) , B = ( 3,5,0) i C = (1,0,0) i la
z
recta r : x = y − 1 = .
2
a) Trobeu el punt d’intersecció de la recta r amb el pla que passa pels punts A, B i
C.
b) Trobeu els punts P de la recta r per als quals el tetraedre de vèrtexs P, A, B i C
3
té un volum de 2u .
NOTA: El volum d’un tetraedre de vèrtexs P, Q, R i S es pot calcular amb l’expressió
1
6
(
det PQ, PR, PS . )
Solució

108) PAU 2018 Sèrie 5 Qüestió 2:


Siguin el pla d’equació  : x + y − z = 0 i el punt P = ( 2,3,2) .
a) Calculeu el punt simètric del punt P respecte del pla  .
b) Calculeu l’equació cartesiana (és a dir, que té la forma Ax + By + Cz = D ) dels dos
plans paral·lels a  que estan a una distància 3 del punt P.
NOTA: Podeu calcular la distància d’un punt de coordenades ( x0 , y0 , z0 ) al pla
Ax0 + By0 + Cz0 + D
d’equació Ax + By + Cz + D = 0 amb l’expressió .
A2 + B 2 + C 2
Solució

C/ Pasqual i Batlle, 1-15 08790 Gelida Telèfon: 93 779 04 50 Pàg. 20


e-mail: a8035246@xtec.cat https://agora.xtec.cat/iesgelida
Generalitat de Catalunya
Departament d’Educació
Institut Gelida Departament de Matemàtiques

109) PAU 2018 Sèrie 5 Qüestió 5:


y−2
Siguin les rectes r1 : x − 1 == z − 5 i r2 : ( x, y, z ) = ( 2 − 3, −1 + ,2 ) .
−1
a) Trobeu l’equació cartesiana (és a dir, que té la forma Ax + By + Cz = D ) del pla
que conté la recta r1 i és paral·lel a la recta r2 .
b) Digueu quina condició s’ha de complir perquè existeixi un pla que contingui la recta
r1 i sigui perpendicular a la recta
r2 . Amb les rectes r1 i r2 de l’enunciat, comproveu si
existeix un pla que contingui la recta r1 i sigui perpendicular a la recta r2 .
Solució

110) PAU 2019 Sèrie 1 Qüestió 3:


Un dron es troba en el puntP = ( 2, −3,1) i volem dirigir-lo en línia recta fins al punt més
proper del pla d’equació  : 3x + 4z + 15 = 0 .
a) Calculeu l’equació de la recta, en forma paramètrica, que ha de seguir el dron.
Quina distància ha de recórrer fins a arribar al pla?
b) Trobeu les coordenades del punt del pla on arribarà el dron.
NOTA: Podeu calcular la distància que hi ha d’un punt de coordenades ( x0 , y0 , z0 ) al
Ax0 + By0 + Cz0 + D
pla d’equació Ax + By + Cz + D = 0 amb l’expressió .
A2 + B 2 + C 2
Solució

111) PAU 2019 Sèrie 4 Qüestió 2:


x = 2
Siguin la recta r: i el pla  : x − z = 3 .
y − z =1
a) Calculeu l’equació paramètrica de la recta que és perpendicular al pla  i que el
talla en el mateix punt en què el talla la recta r.
b) Trobeu els punts de r que estan a una distància de8 unitats del pla  .
NOTA: Podeu calcular la distància que hi ha d’un punt de coordenades ( x0 , y0 , z0 ) al
Ax0 + By0 + Cz0 + D
pla d’equació Ax + By + Cz + D = 0 amb l’expressió .
A2 + B 2 + C 2
Solució

112) PAU 2019 Sèrie 4 Qüestió 5:


Considereu els plans 1 : 2 x + ay + z = 5 ,  2 : x + ay + z = 1 i
 3 : 2 x + ( a + 1) y + ( a + 1) z = 0 , en què a és un paràmetre
real.
a) Estudieu per a quins valors del paràmetre a els tres plans
es tallen en un punt.

b) Comproveu que per al cas a = 1 la interpretació


geomètrica del sistema format per les equacions dels tres
plans és la que es mostra en la imatge següent:

C/ Pasqual i Batlle, 1-15 08790 Gelida Telèfon: 93 779 04 50 Pàg. 21


e-mail: a8035246@xtec.cat https://agora.xtec.cat/iesgelida
Generalitat de Catalunya
Departament d’Educació
Institut Gelida Departament de Matemàtiques

Solució
SOLUCIONARI:

1) PAU LOGSE 1999 Sèrie 1 Problema 1:


Donat el tetràedre de vèrtexs A = (0, 0, 0), B = (1, 1, 1), C = (3, 0, 0) i D = (0, 3, 0)

a) Calculeu l'equació del pla que conté la cara BCD i la del pla que conté la cara
ACD.

• Equació del pla que conté la cara BCD:


BC = C − B = ( 3,0,0 ) − (1,1,1) = ( 2, −1, −1)
BD = D − B = ( 0,3,0 ) − (1,1,1) = ( −1, 2, −1)
Equació del pla que passa pel punt B amb vectors directors BC i BD :
x − 1 2 −1
1 : y − 1 −1 2 = 0 → x + y + z − 3 = 0 (Equació del pla que conté la cara
z − 1 −1 −1
BCD)

• Equació del pla que conté la cara ACD:


AC = C − A = ( 3,0,0 ) − ( 0,0,0 ) = ( 3,0,0 )
AD = D − A = ( 0,3,0 ) − ( 0,0,0 ) = ( 0,3,0 )
Equació del pla que passa pel punt A amb vectors directors AC i AD :
x−0 3 0 x 3 0
 2 : y − 0 0 3 = 0 → y 0 3 = 0 → 9z = 0 → z = 0 (Equació del pla que
z −0 0 0 z 0 0
conté la cara ACD)

b) Calculeu les equacions de dues de les altures del tetràedre, la que passa pel vèrtex
A i la que passa pel vèrtex B, respectivament. (Nota: altura d'un tetràedre és la recta
que passa per un vèrtex i és perpendicular al pla que determina la cara oposada.)

• Equació de l’altura que passa per A:


Aquesta altura serà la recta r1 que passa per A i és perpendicular a la cara del
tetraedre que formen els tres vèrtexs que no són A, és a dir, la recta r1 que passa
per A i és perpendicular a la cara BCD. Per tant, la recta r1 que passa per A i és
perpendicular al pla 1 : x + y + z − 3 = 0 .
r1 perpendicular a 1 → vr = n → vr = (1,1,1)
1 1 1

Per tant, r1 passa pel punt A = ( 0,0,0 ) i té com a vector director vr1 = (1,1,1) :
x−0 y−0 z −0 x = y
r1 : = = → r1 : x = y = z → r1 : 
1 1 1 x = z

C/ Pasqual i Batlle, 1-15 08790 Gelida Telèfon: 93 779 04 50 Pàg. 22


e-mail: a8035246@xtec.cat https://agora.xtec.cat/iesgelida
Generalitat de Catalunya
Departament d’Educació
Institut Gelida Departament de Matemàtiques

• Equació de l’altura que passa per B:


Aquesta altura serà la recta r2 que passa per B i és perpendicular a la cara del
tetraedre que formen els tres vèrtexs que no són B, és a dir, la recta r2 que passa
per B i és perpendicular a la cara ACD. Per tant, la recta r2 que passa per B i és
perpendicular al pla 2 : z = 0 .
r2 perpendicular a  2 → vr = n → vr = ( 0,0,1)
2 2 2

Per tant, r2 passa pel punt B = (1,1,1) i té com a vector director vr2 = ( 0,0,1) :
1 ( x − 1) = 0  ( z − 1)  x − 1 = 0
x −1 y −1 z −1  x = 1
r2 : = = → → → r2 : 
0 0 1 1 ( y − 1) = 0  ( z − 1)  y − 1 = 0
 y =1

c) Comproveu que les dues altures anteriors es tallen en un punt P.

x = y x = 1
Tenim que r1 :  i r2 :  . Tenim el sistema:
x = z y =1

 x = y 1 = y
x = z 1 = z
 
 → → ( x, y, z ) = (1,1,1) → P = (1,1,1)
x = 1 x = 1
 y = 1  y = 1

d) Comproveu si la recta que uneix qualsevol vèrtex del tetràedre amb P és


perpendicular a la cara oposada (i és, per tant, una altura del tetràedre).

• Comprovem que la recta que uneix el vèrtex A amb P és perpendicular a la cara


BCD:
Això és el mateix que comprovar que la recta r1 trobada en l’apartat b) que per
construcció uneix A amb P és perpendicular a la cara BCD. Però aquesta recta sí
que és perpendicular a la cara BCD per construcció. (Era l’altura que passava per
A)
• Anàlogament la recta que uneix el vèrtex B amb P també és perpendicular a la
cara ACD.
• Resta per comprovar que la recta que passa per C i P és perpendicular a la cara
ABD i que la recta que passa per D i P és perpendicular a la cara ABC.
Comprovem la 1a:
Per comprovar que la recta que passa C i P és perpendicular a la cara ABD
solament cal comprovar que el vector director d’aquesta recta CP és
perpendicular als vectors AB i AD . Tenim que:
CP = P − C = (1,1,1) − ( 3,0,0 ) = ( −2,1,1)
AB = B − A = (1,1,1) − ( 0,0,0 ) = (1,1,1)
AD = D − A = ( 0,3,0 ) − ( 0,0,0 ) = ( 0,3,0 )

C/ Pasqual i Batlle, 1-15 08790 Gelida Telèfon: 93 779 04 50 Pàg. 23


e-mail: a8035246@xtec.cat https://agora.xtec.cat/iesgelida
Generalitat de Catalunya
Departament d’Educació
Institut Gelida Departament de Matemàtiques

Ara tenim dos possibilitats, comprovar que els productes escalars CP  AB i


CP  AD són tots dos zero o també que el vector CP és proporcional al producte
vectorial AB  AD .
Anem a fer-ho mitjançant el producte escalar:
CP  AB = ( −2,1,1)  (1,1,1) = −2 + 1 + 1 = 0
CP  AD = ( −2,1,1)  ( 0,3,0 ) = 0 + 3 + 0 = 3  0
Per tant, la recta que passa per C i per D no és perpendicular a la cara ABD i
podem assegurar que en general no existeix cap punt on es troben totes les altures
d’un tetraedre.
Tornar a l’enunciat

2) PAU LOGSE 1999 Sèrie 2 Qüestió 2:


x−3 z +1
Considereu la recta r de l'espai d'equacions =y=
2 2
Trobeu l'equació cartesiana del pla que conté r i que passa pel punt P = (1, 1, 1)
(equació cartesiana vol dir la de la forma ax + by + cz = d).

A = ( 3,0, −1) és un punt de la recta r i u = ( 2,1, 2 ) és el seu vector director.

AP = P − A = (1,1,1) − ( 3,0, −1) = ( −2,1, 2 ) .


El pla  que busquem serà el pla que passa pel punt P = (1,1,1) i té com a vectors

directors u = ( 2,1, 2 ) i AP = ( −2,1, 2 ) . Per tant:


x − 1 2 −2
 : y −1 1 1 = 0 →  : 2 y − z = 1
z −1 2 2
Tornar a l’enunciat

3) PAU LOGSE 1999 Sèrie 2 Problema 2:


Donats els punts de l'espai A = (2, 1, 0), B = (0, 2, 0), C = (–3, 0, 0) i D = (0, –1, 0)
a) Són coplanaris?

A, B, C i D coplanaris sii ho són els vectors AB , AC i AD .


AB = B − A = ( 0, 2,0 ) − ( 2,1,0 ) = ( −2,1,0 )
AC = C − A = ( −3,0,0 ) − ( 2,1,0 ) = ( −5, −1,0 )
AD = D − A = ( 0, −1,0 ) − ( 2,1,0 ) = ( −2, −2,0 )

−2 −5 −2
 AB, AC , AD  = 1 −1 −2 = 0 → Coplanaris
 
0 0 0

Formen un paral·lelogram?
C/ Pasqual i Batlle, 1-15 08790 Gelida Telèfon: 93 779 04 50 Pàg. 24
e-mail: a8035246@xtec.cat https://agora.xtec.cat/iesgelida
Generalitat de Catalunya
Departament d’Educació
Institut Gelida Departament de Matemàtiques

Un paral·lelogram és un quadrilàter de costats paral·lels dos a dos. Per tant hem de


comprovar si els vectors són paral·lels dos a dos.
AB = B − A = ( 0, 2,0 ) − ( 2,1,0 ) = ( −2,1,0 )
BC = C − B = ( −3,0,0 ) − ( 0, 2,0 ) = ( −3, −2,0 )
CD = D − C = ( 0, −1,0 ) − ( −3,0,0 ) = ( 3, −1,0 )
AD = D − A = ( 0, −1,0 ) − ( 2,1,0 ) = ( −2, −2,0 )
Es veu clarament que no hi ha parells de vectors proporcionals i per tant el
quadrilàter no és un paral·lelogram.

b) Calculeu l'àrea del polígon ABCD.


En l’apartat A hem comprovat que els punts A, B, C i D són coplanaris, però si ens
fixem això era obvi donat que tots 4 tenen la tercera component igual a 0. Per tant,
aquestos punts no solament són coplanaris sinó que tots 4 estan en el pla z = 0 . Per
tant, si ens dibuixem les altres dos components, és a dir, A = ( 2,1) , B = ( 0,2) ,
C = ( −3,0 ) i D = ( 0, −1) tenim el següent esquema:

Podem dividir el polígon amb dos triangles


BDA i BDC.
El triangle BDA té la base BD de longitud
igual a 3 unitats i l’altura relativa a
aquesta base de longitud 2. Per tant, la
seva àrea és de 3.
El triangle BDC té la base BD de 3 unitats i
l’altura relativa a aquesta base també de
3 unitats, per tant, la seva àrea és de 9
2
unitats.
Sumant: 3 + 92 = 6
2 + 92 = 15
2

c) Calculeu el punt simètric del punt E = (1, 1, 2) respecte del pla que determinen A, B i
C.

Podem calcular-lo però el pla que determinen els punts A, B i C és el pla  : z = 0 .


Per tant, calcularem el simètric del punt E respecte d’aquest pla. Per fer-ho
calculem la recta r que passa per E i és perpendicular al pla  : z = 0 .

r ⊥  → vr proporcional a n → v r proporcional a ( 0,0,1) .


x −1 y −1 z − 2
Així r : ( x, y, z ) = (1,1, 2 ) +  ( 0, 0,1) → = = →
0 0 1
1 ( x − 1) = 0  ( z − 2 )  x − 1 = 0
 x = 1
→ → → r:
1 ( y − 1) = 0  ( z − 2 )  y − 1 = 0
 y =1
Ara fem el punt de tall de la recta r amb el pla  obtenint el següent sistema:

C/ Pasqual i Batlle, 1-15 08790 Gelida Telèfon: 93 779 04 50 Pàg. 25


e-mail: a8035246@xtec.cat https://agora.xtec.cat/iesgelida
Generalitat de Catalunya
Departament d’Educació
Institut Gelida Departament de Matemàtiques

z = 0

 x = 1 que òbviament té la solució P = (1,1,0)
y =1

EP = P − E = (1,1,0 ) − (1,1, 2 ) = ( 0,0, −2 )
La solució al nostre problema serà el punt:
Q = P + EP = (1,1, 0 ) + ( 0, 0, −2 ) = (1,1, −2 )

d) Calculeu la distància entre la recta que passa per E i A i la recta que passa per B i C.

• Primer que res hem de calcular les equacions d’aquestes rectes i determinar la
seva posició relativa.

• Recta r que passa pels punts E = (1, 1, 2) i A = (2, 1, 0):


EA = A − E = ( 2,1,0 ) − (1,1, 2 ) = (1,0, −2 ) , aleshores:
x − 1 y − 1 z − 2 0 = y − 1 y =1
r: = = → 
1 0 −2 −2 ( x − 1) = z − 2 2 x + z = 4
• Recta s que passa pels punts B = (0, 2, 0) i C = (–3, 0, 0):
BC = C − B = ( −3,0,0 ) − ( 0, 2,0 ) = ( −3, −2,0 ) → vs = ( 3, 2,0 ) , aleshores:
x+3 y z 2 ( x + 3) = 3 y  z = 0
s: = = → 
3 2 0 0 = 3 z 2 x − 3 y = −6
• Posició relativa de r i s:
Donat que el sistema que surt és molt fàcil el resoldrem directament:
y =1 y =1 y =1
2 x + z = 4 z = 0 
  z = 0
     → Sistema incompatible, i com les dues
 z = 0  2 x = 4  x = 2

 2 x − 3 y = −6 
 2 x − 3 = −6  x = −23
rectes r i s no són paral·leles perquè els seus vectors directors no són proporcionals
podem afirmar que r i s es creuen.

Utilitzant la fórmula per la distància entre dues rectes que es creuen tenim que:
−2 1 3
1 0 2
AB, vr , vs 0 −2 0 −14 14
d ( r, s ) = = = = =
vr  vs (1, 0, −2 )  ( 3, 2, 0 ) ( 4, −6, 2 ) 42 + ( −6 ) + 22
2

14 14
= =
56 2
Tornar a l’enunciat

C/ Pasqual i Batlle, 1-15 08790 Gelida Telèfon: 93 779 04 50 Pàg. 26


e-mail: a8035246@xtec.cat https://agora.xtec.cat/iesgelida
Generalitat de Catalunya
Departament d’Educació
Institut Gelida Departament de Matemàtiques

4) PAU LOGSE 1999 Sèrie 5 Qüestió 2:


4 x − y − z = 0 y
Donades les rectes r1 :  i r2 : x = = z
2 x + y − 2 z − 1 = 0 3
Calculeu l'equació del pla paral·lel a les dues rectes que passa per l'origen.

Si un pla  és paral·lel a una recta r aleshores el vector director de la recta,


també ho serà del pla. Per tant, els dos vectors directors d’aquestes rectes v1 i v2
seran vectors directors del pla  que finalment quedarà completament
determinant afegint que passa per l’origen de coordenades O = ( 0,0,0) .
Per tant, calculem els vectors directors de les rectes:
4 x − y − z = 0 4 x − y = z z = 4 x − y = 
r1 :  → ⎯⎯ ⎯→
2 x + y − 2 z − 1 = 0 2 x + y = 2 z + 1 2 x + y = 2 + 1
Sumant les equacions tenim que:
 1
6 x = 3 + 1 → x = +
2 6
 1 2
4x − y =  → y = 4x −  = 4  +  −  =  +
 2 6 3
 1 2  1 2  1 
Així: ( x, y, z ) =  + ,  + ,   → r1 : ( x, y, z ) =  , , 0  +   ,1,1 →
2 6 3  6 3  2 
1 2 
→ r1 : ( x, y, z ) =  , , 0  +  (1, 2, 2 ) → v1 = (1, 2, 2 )
6 3 
v2 = (1,3,1)
Per tant  serà el pla que passa pel punt O = ( 0,0,0) i té com a vectors directors

v1 = (1,2,2) i v2 = (1,3,1) . És a dir:


x−0 1 1 x 1 1
 : y − 0 2 3 = 0 → y 2 3 = 0 →  : 4x − y − z = 0
z −0 2 1 z 2 1
Tornar a l’enunciat

5) PAU LOGSE 1999 Sèrie 6 Qüestió 4:


x −1  x − 2z = 5
Considereu les rectes r : = y = z −2 i s: :
2  x − 2 y = 11
Comproveu que aquestes dues rectes són paral·leles i calculeu l'equació del pla que
les conté.

Per comprovar que les dues rectes són paral·leles podem veure que els seus vectors
directors són proporcionals o treballar per rangs.
Com desprès hem de calcular el pla que conté les dues rectes, necessitarem un punt
de cada recta i el vector director. Així que deixarem de banda els rangs i treballarem
per vectors.

C/ Pasqual i Batlle, 1-15 08790 Gelida Telèfon: 93 779 04 50 Pàg. 27


e-mail: a8035246@xtec.cat https://agora.xtec.cat/iesgelida
Generalitat de Catalunya
Departament d’Educació
Institut Gelida Departament de Matemàtiques

x −1
r: = y = z − 2 → r : ( x, y, z ) = (1, 0, 2 ) +  ( 2,1,1)
2
x − 2z = 5 x = 2z + 5 z =  x = 2 + 5
s:  ⎯⎯ ⎯→ Restant les equacions:
 x − 2 y = 11  x − 2 y = 11  x − 2 y = 11
→ 2 y = 2 − 6 → y =  − 3 Així:
( x, y, z ) = ( 2 + 5,  − 3,  ) → s : ( x, y, z ) = ( 5, −3, 0 ) +  ( 2,1,1)
Podem observar que vr = vs → r i s paral·leles o coincidents.

A = (1,0, 2)  r i B = ( 5, −3,0)  s AB = B − A = ( 5, −3,0 ) − (1,0, 2 ) = ( 4, −3, −2 ) que no


és proporcional a vr , per tant r i s paral·leles:
Per calcular el pla  que conté les dues rectes farem el pla que conté els vectors
vr = ( 2,1,1) AB = ( 4, −3, −2 ) i que passa pel punt (1,0, 2) . Aquest pla serà:
x −1 2 4
: y 1 −3 = 0 →  : x + 8 y − 10 z = −19
z − 2 1 −2
Tornar a l’enunciat

6) PAU LOGSE 2000 Sèrie 1 Problema 2:


x = 2z − 3
Donats el pla  d'equació x + 2y + 3z – 1 = 0, la recta r d'equacions  i el
y = z + 4
punt P = (2, 1, 1), calculeu:
a) Unes equacions de la recta que passa per P i és perpendicular a  .

Anomenem a la nostra recta s . Aleshores s ⊥  → vs proporcional a n = (1, 2,3) →


vs = (1,2,3) .
Aleshores la recta s passa pel punt P = ( 2,1,1) i té com a vector director vs = (1,2,3)
per tant, la seva equació vectorial és: ( x, y, z ) = ( 2,1,1) +  (1, 2,3)

b) L'equació del pla que passa per P i és perpendicular a la recta r.


Anomenem al pla que busquem  ' . Com  ' és perpendicular a r, aleshores el vector
normal a ' serà el vector director de r. És a dir, n ' = vr . Calculem aquest últim:
 x = 2 z − 3 z =  x = 2 − 3
r: ⎯⎯⎯ → → ( x, y, z ) = ( 2 − 3,  + 4,  ) →
y = z + 4 y =  + 4
→ ( x, y, z ) = ( −3,4,0) +  ( 2,1,1) → vr = ( 2,1,1)
Ara sabem que el nostre pla ' passa pel punt P = ( 2,1,1) i que té com a vector

normal n ' = ( 2,1,1) . Així:


P =( 2,1,1) '
 ' : 2 x + y + z + D = 0 ⎯⎯⎯⎯→ 2  2 +1+1+ D = 0 → 6 + D = 0 →
→ D = −6 →  ' : 2 x + y + z − 6 = 0

C/ Pasqual i Batlle, 1-15 08790 Gelida Telèfon: 93 779 04 50 Pàg. 28


e-mail: a8035246@xtec.cat https://agora.xtec.cat/iesgelida
Generalitat de Catalunya
Departament d’Educació
Institut Gelida Departament de Matemàtiques

c) Unes equacions de la recta que passa per P i talla perpendicularment r.


Per trobar l’equació de la recta t que passa pel punt P i talla perpendicularment a r
farem el següent:
1) Calculem l’equació del pla que passa per P i és perpendicular a r. (Calculat a
l’apartat b)
2) Calculem el punt P’ on aquest pla talla la recta r.
3) Finalment calculem l’equació de la recta que passa pels punts P i P’.
Així:
1) Equació del pla que passa per P i és perpendicular a r:  ' : 2 x + y + z − 6 = 0 .
2) Punt de tall P’ on aquest pla ' talla la recta r.
2 x + y + z − 6 = 0

x = 2z − 3 → 2  ( 2 z − 3) + z + 4 + z − 6 = 0 → 4 z − 6 + z + 4 + z − 6 = 0 →
y = z + 4

→ 6 z − 8 = 0 → 6 z = 8 → z = 43
x = 2 z − 3 = 2  43 − 3 = 83 − 3 = −31
y = z + 4 = 43 + 4 = 163 → ( x, y, z ) = ( −31 , 163 , 34 ) → P ' = ( −31 , 163 , 34 ) .

3) Equació de la recta que passa per P = ( 2,1,1) i P ' = ( −31 , 163 , 34 ) .


PP ' = P '− P = ( −31 , 163 , 34 ) − ( 2,1,1) = ( −37 , 133 , 13 ) proporcional a ( −7,13,1) .
Així la recta que busquem passa per P = ( 2,1,1) i té com a vector director ( −7,13,1) .
Per tant, l’equació vectorial d’aquesta recta és: ( x, y, z ) = ( 2,1,1) +  ( −7,13,1)

d) Unes equacions de la recta que passa per P, és paral·lela al pla  i tal que el seu
vector director és perpendicular al de r.
Anomenem a la nostra recta t. Si la recta t és paral·lela al pla  , aleshores el vector
director de la recta vt , serà perpendicular al vector normal del pla n = (1, 2,3) .
Per altra banda, l’enunciat diu que la nostra recta t també ha de ser perpendicular a
la recta r. És a dir, vt serà perpendicular a vr = ( 2,1,1) .
Però tenim un mecanisme molt fàcil per trobar un vector vt perpendicular a
n = (1, 2,3) ia vr = ( 2,1,1) . Aquest és el producte vectorial. És a dir:
i j k
vt = n  vr = (1, 2,3)  ( 2,1,1) = 1 2 3 = ( −1,5, −3)
2 1 1
Per tant, la nostra recta és: t : ( x, y, z ) = ( 2,1,1) +  ( −1,5, −3)
Tornar a l’enunciat

C/ Pasqual i Batlle, 1-15 08790 Gelida Telèfon: 93 779 04 50 Pàg. 29


e-mail: a8035246@xtec.cat https://agora.xtec.cat/iesgelida
Generalitat de Catalunya
Departament d’Educació
Institut Gelida Departament de Matemàtiques

7) PAU LOGSE 2000 Sèrie 2 Problema 2:


2 x − 5 y − z − 3 = 0
Considereu la recta r: i el pla  : 2 x − y + az + 2 = 0 , on a és un
x − 3y − z − 2 = 0
paràmetre.
a) Per a quin valor de a la recta i el pla són paral·lels? Quina serà llavors la distància
entre el punt P = (1, 0, –1) de la recta i el pla?

Anem a estudiar la posició relativa de la recta r i el pla  en funció del paràmetre a:


2 x − 5 y − z = 3  2 −5 −1  2 −5 −1 3 
    
 x − 3 y − z = 2 → M =  1 −3 −1 i M ' =  1 −3 −1 2 
2 x − y + az = −2    
  2 −1 a   2 −1 a −2 

Aprofitant que M és una matriu quadrada podem treballar per determinants.


(Evidentment també podem optar per fer-ho pel mètode de Gauss)
2 −5 −1
M = 1 −3 −1 = 3 − a = 0 → a = 3
2 −1 a
Sabem que per al valor a = 3 la matriu M tindrà rang 2 però i la matriu M’? Anem a
calcular el rang de la matriu M’ per aquest valor.
 2 −5 −1 3  F  F  1 −3 −1 2  F →F −2 F  1 −3 −1 2   1 −3 −1 2 
  1 2  2 2 1   
M ' =  1 −3 −1 2    2 −5 −1 3    0 1 1 −1    0 1 1 −1
F3 → F3 − 2 F1
 2 −1 3 −2   2 −1 3 −2   0 5 5 −6   0 0 0 −1
       
Per tant, quan a=3 aleshores Rang ( M ) = 2  3 = Rang ( M ') → S.I . → r i  paral·lels.

Ara ens demanen calcular la distància del punt P = (1,0, −1) al pla
 : 2 x − y + 3z + 2 = 0 .
Sabem que la distància entre el punt P = ( p1 , p2 , p3 ) i el pla  : Ax + By + Cz + D = 0
Ap1 + Bp2 + Cp3 + D
és d ( P,  ) = . En el nostre cas:
A2 + B 2 + C 2
2 1 − 0 + 3  ( −1) + 2 2−0−3+ 2 1 14
d ( P,  ) = = = =
22 + ( −1) + 32
2
4 +1+ 9 14 14

b) Existeix algun valor de a per al qual la recta i el pla siguin perpendiculars?


La recta r i el pla  seran perpendiculars sii els vectors vr i n apunten en la mateixa
direcció.
n = ( 2, −1, a ) Calculem vr .
i j k
2 x − 5 y − z − 3 = 0
r: → vr = ( 2, −5, −1)  (1, −3, −1) = 2 −5 −1 = ( 2,1, −1)
x − 3y − z − 2 = 0 1 −3 −1

C/ Pasqual i Batlle, 1-15 08790 Gelida Telèfon: 93 779 04 50 Pàg. 30


e-mail: a8035246@xtec.cat https://agora.xtec.cat/iesgelida
Generalitat de Catalunya
Departament d’Educació
Institut Gelida Departament de Matemàtiques

Aleshores:
2 1 −1
r ⊥   vr proporcional n  ( 2,1, −1) proporcional a ( 2, −1, a )  = =
2 −1 a
2 1
Però donat que  aleshores mai poden ser perpendiculars. Per tant no existeix
2 −1
cap valor de a pel qual la recta i el pla siguin perpendiculars.

c) Determineu el valor de a perquè la recta i el pla formin un angle de 30º.

Sabem que aquest angle complirà la següent equació:

vr  n ( 2,1, −1)  ( 2, −1, a ) 2  2 + 1 ( −1) − 1 a


sin  = → sin  = → sin  =
vr n ( 2,1, −1) ( 2, −1, a ) 22 + 12 + ( −1)  22 + ( −1) + a 2
2 2

4 −1 − a  =30º 1 4 −1 − a
sin  = ⎯⎯⎯ → sin 30º = = → 6  5 + a2 = 2 4 −1 − a →
6  5 + a2 2 6  5+ a 2

→ 6  ( 5 + a2 ) = 4  ( 3 − a ) → 30 + 6a2 = 4 (9 − 6a + a2 ) → 30 + 6a2 = 36 − 24a + 4a 2 →


2

→ 2a2 + 24a − 6 = 0 → a 2 + 12a − 3 = 0 → a = −6  39


Tornar a l’enunciat

8) PAU LOGSE 2000 Sèrie 3 Problema 2:


Un quadrat de l'espai té tres dels seus vèrtexs consecutius situats en els punts de
coordenades enteres P = (3, –2, 4), Q = (a, –1, a + 1) i R = (2, –3, 0).
a) Tenint en compte que els vectors QP i QR han de ser perpendiculars, calculeu el
valor del nombre enter a.

QP = P − Q = ( 3, − 2, 4 ) − ( a, − 1, a + 1) = ( 3 − a, − 1, 3 − a )
QR = R − Q = ( 2, −3,0 ) − ( a, − 1, a + 1) = ( 2 − a, − 2, − a − 1)
QP ⊥ QR  QP  QR = 0  ( 3 − a, − 1, 3 − a )  ( 2 − a, − 2, − a − 1) = 0 
 ( 3 − a )( 2 − a ) −1( −2) + (3 − a )( −a − 1) = 0 → 6 − 3a − 2a + a2 + 2 − 3a − 3 + a2 + a = 0 →
a = 1
→ 2a 2 − 7 a + 5 = 0 → 
a = 2
5

Si P, Q i R són vèrtex d’un quadrat, aleshores els costats han de mesurar el mateix, per
tant, QP = QR . Comprovem aquesta condició pels valors obtinguts anteriorment:

• a = 1 → QP = ( 2, −1, 2 ) i QR = (1, −2, −2 ) → QP = QR = 5


• a= 5
2 → QP = ( 12 , −1, 12 ) i QR = ( −21 , −2, −27 ) → QP  QR → En aquest cas no formen
part d’un quadrat. Per tant, el valor buscat és : a = 1

C/ Pasqual i Batlle, 1-15 08790 Gelida Telèfon: 93 779 04 50 Pàg. 31


e-mail: a8035246@xtec.cat https://agora.xtec.cat/iesgelida
Generalitat de Catalunya
Departament d’Educació
Institut Gelida Departament de Matemàtiques

b) Calculeu l'equació del pla que conté aquest quadrat.

P = (3, –2, 4), Q = (a, –1, a + 1) i R = (2, –3, 0).

QP = ( 2, −1, 2 )

a =1→ 
QR = (1, −2, −2 )

El pla  que passa pels punts P, Q i R serà el pla que passa pel punt P i té com a
vectors directors QP i QR . Calculem-lo:
x −3 2 1
 : y + 2 −1 −2 = 0 →  : 2 x + 2 y − z + 2 = 0
z − 4 2 −2

c) Calculeu el quart vèrtex d'aquest quadrat.

Podem observar en el dibuix adjunt que si anomenem T al quart vèrtex


del quadrat (en gris) tenim que T = P + QR .
P = (3, –2, 4), Q = (1, –1, 2) i R = (2, –3, 0).
Aleshores QR = R − Q = ( 2, −3,0 ) − (1, −1, 2 ) = (1, −2, −2 ) .
Finalment: T = P + QR = ( 3, −2, 4 ) + (1, −2, −2 ) = ( 4, −4, 2 )

d) Calculeu l'àrea d'aquest quadrat.

( 1 + (−2) + (−2) ) =
2
Àrea quadrat = Costat = QR = (1, −2, −2 ) =
2 2 2 2
2 2

= 12 + ( −2 ) + ( −2 ) = 1 + 4 + 4 = 9 u 2
2 2

Tornar a l’enunciat

9) PAU LOGSE 2000 Sèrie 6 Qüestió 3:


Calculeu el peu de la recta perpendicular a la recta r : ( x, y, z ) = (1, −1,1) +  ( 0,1,1)
traçada des del punt P=(1, 0, –1).
L’estratègia per resoldre aquest problema serà la següent:
1) Calculem el pla  perpendicular a r i que
passa pel punt P = (1,0, −1) .
2) Calculem el punt Q on el pla  i la recta r
es tallen.
3) La solució al problema serà la recta s que
passarà pels punts P i Q (en vermell).

1) Càlcul del pla  perpendicular a r i que passa


pel punt P = (1,0, −1) .
 ⊥ r → n = vr → n = ( 0,1,1) →  : y + z + D = 0 ⎯⎯⎯
P
→ 0 −1+ D = 0 →
→ D = 1 →  : y + z +1 = 0
C/ Pasqual i Batlle, 1-15 08790 Gelida Telèfon: 93 779 04 50 Pàg. 32
e-mail: a8035246@xtec.cat https://agora.xtec.cat/iesgelida
Generalitat de Catalunya
Departament d’Educació
Institut Gelida Departament de Matemàtiques

2) Càlcul del punt Q on el pla  i la recta r es tallen.


x −1 y + 1 z −1  x −1 = 0 x = 1
r : ( x, y, z ) = (1, −1,1) +  ( 0,1,1) → = = → 
0 1 1  y +1 = z −1  y = z − 2
Per tant, el sistema que he de resoldre és:

 y + z +1 = 0

x = 1 → ( z − 2) + z + 1 = 0 → 2z −1 = 0 → z = 1
2
y = z − 2

y = z − 2 → y = 12 − 2 → y = −23 → ( x, y, z ) = (1, −23 , 12 ) → Q = (1, −23 , 12 )

3) Càlcul de la recta que passa pels punts P = (1,0, −1) i Q = (1, −23 , 12 ) .

PQ = Q − P = (1, −23 , 12 ) − (1,0, −1) = ( 0, −23 , 32 ) proporcional a u = ( 0, −1,1)


Per tant la recta buscada serà: s : ( x, y, z ) = (1, 0, −1) +  ( 0, −1,1)
Tornar a l’enunciat

10) PAU LOGSE 2000 Sèrie 6 Problema 1:


Considereu la recta r de l'espai que passa pel punt P = (1, 1, 3) i té per vector director
v = (1 − a, a,1) . Sigui  el pla que té per equació 2x + y – z = 1.
a) Determineu per a cada valor del paràmetre a la posició relativa de la recta r
respecte al pla  (paral·lela, continguda o amb un punt d'intersecció).

Sempre tenim l’opció de procedir mitjançant matrius


però procuraré fer un mètode més ràpid.

En el dibuix adjunt es veu que r i  paral·lels o r


continguda en  sii vr i n ortogonals  vr  n = 0 .
Comprovem açò últim:
vr  n = (1 − a, a,1)  ( 2,1, −1) = 2 (1 − a ) + a −1 = 2 − 2a + a −1 = −a + 1
vr  n = 0 → −a + 1 = 0 → a = 1

No hem acabat, sabem que pel valor a=1 la recta r estarà continguda en el pla o serà
paral·lela a aquest però, en quin dels dos casos ens trobem? Serà suficient comprovar
si el punt P que pertany a r també pertany al pla o no.
P = (1,1,3) i  : 2 x + y − z = 1 substituint: 2 1 + 1 − 3 = 2 + 1 − 3 = 0  1 → P   → r 
Per tant: Si a = 1 aleshores r paral·lela a  i si a  1 aleshores r i  secants.

C/ Pasqual i Batlle, 1-15 08790 Gelida Telèfon: 93 779 04 50 Pàg. 33


e-mail: a8035246@xtec.cat https://agora.xtec.cat/iesgelida
Generalitat de Catalunya
Departament d’Educació
Institut Gelida Departament de Matemàtiques

b) Hi ha alguna de les rectes r que sigui perpendicular al pla  ?

Evidentment la recta r serà perpendicular al pla  sii els vectors


de r i de  apunten en la mateixa direcció, és a dir, si aquestos
vectors són proporcionals. Aleshores:
vr = (1 − a, a,1) i n = ( 2,1, −1)
1− a a 1 1 − a = 2a 1 = 3a a = 13
= = →  
2 1 −1 −a = 1 a = −1 a = −1
Per tant no hi ha cap valor del paràmetre a que faci que la
recta r sigui perpendicular al pla  .

c) Calculeu la distància que hi ha entre el punt P i el pla  .

Donat un punt P = ( p1 , p2 , p3 ) i un pla  : Ax + By + Cz + D = 0 tenim que:


Ap1 + Bp2 + Cp3 + D
d ( P,  ) = . En el nostre cas P = (1,1,3) i  : 2 x + y − z − 1 = 0 i
A2 + B 2 + C 2
2 1 + 1 − 3 − 1 2 + 1 − 3 −1 −1 1 6
per tant: d ( P,  ) = = = = =
22 + 12 + ( −1)
2
4 +1+1 6 6 6
Tornar a l’enunciat

11) PAU LOGSE 2001 Sèrie 2 Qüestió 3:


Donats els punts de l'espai A = (2, 0, 0), B = (0, 1, 0) i C = (0, 0, 3).
a) Determineu l'equació del pla  que els conté.

• Per calcular l’equació del pla  que passa pels punts A, B i C necessitem un punt i
dos vectors directors.
• Calculem dos vectors:
AB = B − A = ( 0, 1, 0 ) − ( 2, 0, 0 ) = ( −2, 1, 0 )
AC = C − A = ( 0, 0, 3) − ( 2, 0, 0 ) = ( −2, 0, 3)
• Finalment calculem l’equació del pla  que passa pel punt A = ( 2, 0, 0) i amb

vectors directors AB = ( −2, 1, 0 ) i AC = ( −2, 0, 3) .


x − 2 −2 −2
: y 1 0 = 0 →  : 3 ( x − 2 ) + 0 + 0 + 2 z − 0 + 6 y = 0 →  : 3x + 6 y + 2 z − 6 = 0
z 0 3

b) Calculeu l'equació de la recta r perpendicular al pla  i que passa per l'origen.

• Si la recta r és perpendicular al pla  , aleshores el vector director de la recta vr


coincidirà amb el vector normal al pla  que anomenarem n .
• Càlcul del vector normal al pla  :

C/ Pasqual i Batlle, 1-15 08790 Gelida Telèfon: 93 779 04 50 Pàg. 34


e-mail: a8035246@xtec.cat https://agora.xtec.cat/iesgelida
Generalitat de Catalunya
Departament d’Educació
Institut Gelida Departament de Matemàtiques

i j k
n = AB  AC = ( −2, 1, 0 )  ( −2, 0, 3) = −2 1 0 =
−2 0 3
= 3i − 0k − 0 j + 2k − 0i + 6 j = 3i + 6 j + 2k = ( 3, 6, 2 )

• Per tant, busquem la recta r que té vector director vr = ( 3, 6, 2) i que passa per

l’origen O = ( 0, 0, 0) . Aquesta recta és: r : ( x, y, z ) = ( 0,0,0 ) +  ( 3,6,2 )


Tornar a l’enunciat

12) PAU LOGSE 2001 Sèrie 4 Qüestió 1:


Determineu per a quins valors del paràmetre a el pla  : ax + 2y + z = a és paral·lel a la
 x − ay + z = 1
recta r : 
ax + z = a + 1

• El pla  i la recta r seran paral·lels sempre i quan el sistema que formin tots dos sigui
incompatible. És a dir, ens estan demanant els valors del paràmetre a que facin que el
ax + 2 y + z = a

sistema  x − ay + z = 1 sigui incompatible.
ax + z = a + 1

• Aleshores hem de discutir el sistema anterior en funció dels valors del paràmetre a .
 a 2 1 a  F  F  1 −a 1 1  F → F − aF  1 −a 1 1
    
A ' =  1 −a 1 1    a 2 1 a    0 2 + a 1 − a 0 
2 1 2 2 1
2
F → F − aF
 a 0 1 a + 1  a 0 1 a + 1 3 3 1  0 a2 1 − a 1 
    

La matriu A tindrà rang diferent de 3 sii les dues últimes files són proporcionals.
Aleshores:
2 + a2 1 − a
= → ( 2 + a 2 ) (1 − a ) = a 2 (1 − a ) → (1 − a ) ( 2 + a 2 − a 2 ) = 0 →
a 2
1− a
→ (1 − a )  2 = 0 → 1 − a = 0 → a = 1
Fins ara sabem que si a =1 aleshores Rang ( A)  3 i que si a 1 aleshores

Rang ( A) = Rang ( A' ) = 3 = nº incògnites → S.C.D. →  i r es tallaran en un punt.


Per tant, el que ens queda per estudiar és si en el cas a = 1 el sistema és compatible
indeterminat o incompatible. Així:
• Estudi del sistema quan a = 1 :

En aquest cas:

C/ Pasqual i Batlle, 1-15 08790 Gelida Telèfon: 93 779 04 50 Pàg. 35


e-mail: a8035246@xtec.cat https://agora.xtec.cat/iesgelida
Generalitat de Catalunya
Departament d’Educació
Institut Gelida Departament de Matemàtiques

1 2 1 1  F →F − F  1 2 1 1 
A ' =  1 −1 1 1    0 −3 0 0  → Podem observar que les equacions 2 i 3
2 2 1

F →F − F
 1 0 1 2  3 3 1  0 −2 0 1 
   
són incompatibles, és a dir, Rang ( A) = 2  3 = Rang ( A' ) → S.I .
Per tant, l’únic valor de a per al que la recta i el pla són paral·lels és a =1

NOTA: Com en tota discussió d’un sistema d’equacions lineals, donat que la matriu de
coeficients era una matriu quadrada, era més ràpid calcular el seu determinar i
estudiar directament els valors de a que fessin nul aquest determinant.
Tornar a l’enunciat

13) PAU LOGSE 2001 Sèrie 4 Problema 2:


Sigui  el pla d'equació x – y + 2z = 3 i P el punt (1, 1, 0).
a) Calculeu la distància d de P a  .

Donat un punt P = ( p1 , p2 , p3 ) i un pla  : Ax + By + Cz + D = 0 tenim que:


Ap1 + Bp2 + Cp3 + D
d ( P,  ) = . En el nostre cas P = (1,1,0) i  : x − y + 2 z − 3 = 0 i
A2 + B 2 + C 2
1−1+ 2 0 − 3 −3 −3 3 3 6 6
per tant: d ( P,  ) = = = = = =
12 + ( −1) + 22
2
1+1+ 4 6 6 6 2

b) Determineu l'equació de l'altre pla  ' paral·lel a  que també dista d del punt P.

Sabem que qualsevol pla ' paral·lel a  : x − y + 2 z − 3 = 0 té una equació del tipus
 ' : x − y + 2z + k = 0 .
6
Per tant, cal determinar k per a que d ( P,  ' ) =
. Aleshores:
2
6 1−1+ 2 0 + k k 6 6 6 6
d ( P,  ' ) = → = = →k = → k = = 3 → k = 3
2 12 + ( −1) + 22 2 2 2
2
6
Com pel pla  el paràmetre k ja valia −3 , el valor de k pel pla ' és k = 3 i per tant
tenim que la resposta al problema és el pla  ' : x − y + 2z + 3 = 0

c) Determineu l'equació de la recta r perpendicular a  que passa per P.

Per ser la recta r perpendicular al pla  tenim que el vector director de r serà el
vector normal al pla  , és a dir, vr = n = (1, − 1, 2) .

Així, la recta r és la recta que passa pel punt P = (1, 1, 0) i té vector director

C/ Pasqual i Batlle, 1-15 08790 Gelida Telèfon: 93 779 04 50 Pàg. 36


e-mail: a8035246@xtec.cat https://agora.xtec.cat/iesgelida
Generalitat de Catalunya
Departament d’Educació
Institut Gelida Departament de Matemàtiques

vr = (1, − 1, 2 ) , per tant: r : ( x, y, z ) = (1,1,0) +  (1, −1,2 ) o si volem escriure-la amb

x −1 y −1 z − 0
l’equació continua seria r: = =
1 −1 2

d) Calculeu la intersecció de la recta r amb el pla  .

Per calcular el punt de intersecció entre r i  solament cal resoldre el sistema


d’equacions que formen tots dos.
x − 1 y − 1 z − 0  x1−1 = −1 → − x + 1 = y − 1 → − x − y = −2 → x + y = 2
y −1

r: = = →  x −1 →
1 −1 2  1 2 = z
→ 2 x − 2 = z → 2 x − z = 2
x + y = 2
→r:
2 x − z = 2
x + y = 2

Per tant, els sistema d’equacions format per r i  és: 2 x − z = 2
 x − y + 2z = 3

 1 1 0 2  F →F −2 F  1 1 0 2  F →F − F  1 1 0 2 
A ' =  2 0 −1 2    0 −2 −1 −2    0 −2 −1 −2  →
2 2 1 3 3 2

F →F − F
 1 −1 2 3  3 3 1  0 −2 2 1  0 0 3 3 
     
3z = 3 → z = 1
1
−2 y − z = −2 → −2 y − 1 = −2 → −2 y = −1 → y =
2
1 1 3
x+ y =2→ x+ =2→ x =2− → x =
2 2 2
3 1 
Així, el punt entre la recta r i el pla  és: P ' =  , , 1
2 2 
Tornar a l’enunciat

14) PAU LOGSE 2001 Sèrie 5 Problema 1:


x − 2 y + 3 z +1
Considereu a l'espai la recta r d'equacions = = i la recta s
2 −3 −1
x + 4 y −1 z + 4
d'equacions = =
−2 3 1

a) Determineu el punt de tall de la recta r amb el pla z = 0.

−3 ( x − 2 ) = 2 ( y + 3) −3x + 6 = 2 y + 6 3x + 2 y = 0


x − 2 y + 3 z +1 
r: = = → → →
2 −3 −1 
 −1 ( x − 2 ) = 2 ( z + 1)  − x + 2 = 2 z + 2 x + 2z = 0
Així el punt de tall entre la recta r i el pla z =0 serà la solució del sistema:

C/ Pasqual i Batlle, 1-15 08790 Gelida Telèfon: 93 779 04 50 Pàg. 37


e-mail: a8035246@xtec.cat https://agora.xtec.cat/iesgelida
Generalitat de Catalunya
Departament d’Educació
Institut Gelida Departament de Matemàtiques

3x + 2 y = 0

 x + 2 z = 0 que té com a única solució el punt ( x, y, z ) = ( 0, 0, 0 )
z = 0

b) Comproveu que les rectes r i s són paral·leles i calculeu la distància entre elles.

Per comprovar que r i s són paral·leles mirarem que els seus vectors directors vr i v s
siguin proporcionals i que un punt d’una recta no pertanyi a l’altra.
2 −3 −1
Tenim que vr = ( 2, −3, −1) i vs = ( −2,3,1) . = = → vr i vs proporcionals r i s
−2 3 1
paral·leles o coincidents.
P = ( 2, −3, −1)  r anem a comprovar si aquest punt també pertany a l’altra recta s.
x + 4 y − 1 z + 4 P =( 2,−3,−1) 2 + 4 −3 − 1 −1 + 4 6 −4 3
s: = = ⎯⎯⎯⎯→ = = → = = FALS
−2 3 1 −2 3 1 −2 3 1
Per tant, P  s i r i s paral·leles.
• Càlcul de la distància entre les dues rectes:
Per calcular la distància entre dues rectes paral·leles és suficient calcular la distància
d’un punt d’una a l’altra recta. Així agafem P = ( 0,0,0)  r Q = ( −4,1, −4)  s i

v = ( 2, −3, −1) vector director de les rectes i tenim:

PQ  v ( −4,1, −4 )  ( 2, −3, −1) ( −13, −12,10 )


d ( r , s ) = d ( Q, r ) = = = =
v ( 2, −3, −1) 22 + ( −3) + ( −1)
2 2

413 118
= =
14 2

c) Quina és l'equació del pla que conté les dues rectes?

Com r i s paral·leles tindran el mateix vector director vr = ( 2, −3, −1) .


El pla  que contindrà aquestes dues rectes serà el pla que té com a vector director
vr = ( 2, −3, −1) i que passa pels punts P = ( 2, −3, −1)  r i Q = ( −4,1, −4 ) , és a dir:
El pla  que passa per P = ( 2, −3, −1)  r i té com a vectors directors vr = ( 2, −3, −1) i

PQ . Però PQ = Q − P = ( −4,1, −4 ) − ( 2, −3, −1) = ( −6, 4, −3) , per tant:


x − 2 2 −6
 : y + 3 −3 4 = 0 → 13x + 12 y − 10 z = 0
z + 1 −1 −3
(Nota, també haguéssim arribat al mateix resultat i amb càlculs relativament més fàcils
agafant com a punt P de la primera recta P = ( 0,0,0 ) .

C/ Pasqual i Batlle, 1-15 08790 Gelida Telèfon: 93 779 04 50 Pàg. 38


e-mail: a8035246@xtec.cat https://agora.xtec.cat/iesgelida
Generalitat de Catalunya
Departament d’Educació
Institut Gelida Departament de Matemàtiques

d) Calculeu la distància del pla anterior a l'origen de coordenades.


És evident que el pla anterior passa per l’origen de coordenades, per tant aquesta
distància és zero.
Tornar a l’enunciat

15) PAU LOGSE 2002 Sèrie 1 Qüestió 3:


Comproveu que la recta que passa pels punts A = (4, 0, 0) i B = (0, 2, 2) és paral·lela al
pla d'equació x – 3y + 5z = 2, i calculeu la distància entre la recta i el pla.

• La recta que r passa pels punts A i B tindrà vector director:


AB = B − A = ( 0,2,2 ) − ( 4,0,0 ) = ( −4,2,2 )
El vector normal al pla  : x − 3 y + 5z = 2 és n = (1, −3,5)
• Si aquests dos vectors AB = ( −4,2,2 ) i n = (1, −3,5) fossin perpendiculars, aleshores
ja estaríem en condicions d’afirmar que la recta i el pla o bé son paral·lels o bé la
recta està continguda en el pla.
AB  n = ( −4,2,2 )  (1, −3,5) = −4 − 6 + 10 = 0 → AB ⊥ n → r i  paral·lels o r està
continguda en  .
Finalment, per decidir si estem en el cas en que r i  són paral·lels o bé estem en el
cas en que r està continguda en  , agafem un punt de la recta r i el substituïm en
l’equació de  .
A = ( 4,0,0)  r i 4 − 3  0 + 5  0 = 4  2 → A → r paral·lela a 

NOTA: Una altra manera de resoldre aquest problema seria trobar l’equació de la
recta i estudiar la posició relativa entre la recta i el pla. Quedaria així:
La recta r que passa per A = ( 4,0,0 ) i B = ( 0, 2, 2 ) té

vector director AB = ( −4, 2, 2 ) i passa per A = ( 4,0,0 ) , per


x−4 y−0 z −0
tant, l’equació d’aquesta recta és r : = = ,
−4 2 2
d’on obtenim que:
 x−4
−4
y
2
2
 = → 2 x − 8 = −4 y → 2 x + 4 y = 8 ⎯⎯→ x + 2 y = 4
 x−4 z 2
 −4 = 2 → 2 x − 8 = −4 z → 2 x + 4 z = 8 ⎯⎯→ x + 2 z = 4

Així, el sistema format per la recta i el pla és:

 x − 3 y + 5z = 2 1 −3 5 2   1 −3 5 2 
    
x + 2 y = 4 → A ' = 1 2 0 4    0 5 −5 2 
x + 2z = 4    
 1 0 2 4   0 3 −3 2 

Podem observar que les dues últimes equacions són incompatibles entre si, per tant, la
recta i el pla no tenen cap punt en comú, és a dir, la recta és paral·lela al pla.
Tornar a l’enunciat

C/ Pasqual i Batlle, 1-15 08790 Gelida Telèfon: 93 779 04 50 Pàg. 39


e-mail: a8035246@xtec.cat https://agora.xtec.cat/iesgelida
Generalitat de Catalunya
Departament d’Educació
Institut Gelida Departament de Matemàtiques

16) PAU LOGSE 2002 Sèrie 1 Problema 2:


Considerem el cub de vèrtexs A, B, C, D, E, F, G, H que té
l'aresta de longitud 4 dm.

a) Determineu l'equació del pla inclinat EHBC si prenem com


a origen de coordenades el vèrtex D i com a eixos de
coordenades DA, DC i DH en aquest ordre, tenint en compte
que el sentit positiu de cada un d'ells és el que sortint de
l'origen D va cap a A, C i H, respectivament.

• Si el cub té aresta igual a 4, aleshores tenim que:


B = ( 4,4,0) , C = ( 0,4,0) , E = ( 4,0,4 ) i H = ( 0,0,4)
• Determinem l’equació del pla  que passa pels punts, per exemple B, C i D:
BC = C − B = ( 0,4,0 ) − ( 4,4,0 ) = ( −4,0,0 )
BE = E − B = ( 4,0,4 ) − ( 4,4,0 ) = ( 0, −4,4 )
Calculem  com el pla que passa pel punt B = ( 4,4,0) i té com a vectors directors

BC i BD .
x − 4 −4 −4
y − 4 −4 y − 4 −1
 : y−4 0 −4 = 0 → ( −1)  ( −4 )  = 0 → 44 =0→
z 4 z 1
z −0 0 4
y − 4 −1
→ =0→ y −4+ z =0→ y + z =4
z 1

b) Calculeu les equacions de les diagonals CE i AG i utilitzeu-les per calcular les


coordenades del seu punt d'intersecció.

• Diagonal que passa per C = ( 0,4,0) i E = ( 4,0,4 ) : (En


blau)

CE = E − C = ( 4,0,4 ) − ( 0,4,0) = ( 4, −4,4 ) → CE (1, −1,1)


x − 0 y − 4 z − 0  1x =
y −4
→ −x = y − 4 → x + y = 4
−1
r: = = → x
1 −1 1  1 = 1z → x = z → x − z = 0

x + y = 4
→r:
x − z = 0
• Diagonal que passa per A = ( 4,0,0) i G = ( 0,4,4 ) : (En vermell)
AG = G − A = ( 0,4,4 ) − ( 4,0,0 ) = ( −4,4,4 ) → AG ( −1,1,1)

x−4 y −0 z−0  = → x − 4 = −y → x + y = 4 x + y = 4
x −4 y

s: = = →  y−1 1 → s:
−1 1 1 1 = 1 → y = z → y − z = 0
 z
y − z = 0

C/ Pasqual i Batlle, 1-15 08790 Gelida Telèfon: 93 779 04 50 Pàg. 40


e-mail: a8035246@xtec.cat https://agora.xtec.cat/iesgelida
Generalitat de Catalunya
Departament d’Educació
Institut Gelida Departament de Matemàtiques

• Resolem el sistema format per les dues rectes r i s :

 x + y = 4
r :  x + y = 4 1 1 0 4 1 1 0 4 
 x − z = 0 
   x − z = 0 → A ' =  1 0 −1 0    0 −1 −1 −4  
s :  x + y = 4  y − z = 0  0 1 −1 0   0 1 −1 0 
  y − z = 0     

1 1 0 4 
  0 −1 −1 −4  → −2 z = −4 → z = 2
 0 0 −2 −4 
 
− y − z = −4 → − y − 2 = −4 → − y = −2 → y = 2
x+ y =4→ x+2=4→ x =2

Per tant, el punt de tall entre les dues rectes és ( x, y, z ) = ( 2,2,2 )


NOTA: Per construcció, cadascuna de les dues rectes va des d’un vèrtex del cub al
vèrtex oposat. Això fa que aquestes dues rectes s’hagin de tallar en el centre del cub
que evidentment és el punt ( 2,2,2) .
Tornar a l’enunciat

17) PAU LOGSE 2002 Sèrie 2 Qüestió 4:

Calculeu l'angle que forma el pla x – 2y + z = 1 amb la recta determinada per les
x = t

equacions  y = 1 + t
z = 2

Sigui el pla  : x − 2 y + z = 1 , aleshores el seu vector normal és n = (1, −2,1) .


x = t

Sigui la recta r :  y = 1 + t , aleshores el seu vector director és vr = (1,1,0 ) .
z = 2

Podem calcular l’angle agut (*) entre aquests dos vectors mitjançant el seu producte
escalar:
n  vr (1, −2,1)  (1,1,0 )
( )
n  vr = n  vr cos n , vr → cos n , vr = ( ) n  vr
=
(1, −2,1) (1,1,0 )
=

 3
=
1− 2 + 0
6 2
=
−1
12
=
2 3
1
=
6
3
(
→ cos n , vr =
6
3
)→ n , vr = ar cos 
 6
 73,22º

Aleshores, l’angle que formen el pla  i la recta r és 90º −73,22º = 16,78º (FIGURA)

C/ Pasqual i Batlle, 1-15 08790 Gelida Telèfon: 93 779 04 50 Pàg. 41


e-mail: a8035246@xtec.cat https://agora.xtec.cat/iesgelida
Generalitat de Catalunya
Departament d’Educació
Institut Gelida Departament de Matemàtiques

(*) Per a forçar que l’angle sigui agut, és a dir, menor de


90º, hem d’agafar el producte escalar en valor absolut.

En el cas de l’angle entre dos vectors, que pot ser obtús,


aleshores prendríem la fórmula

(
n  vr = n  vr cos n , vr ).
Tornar a l’enunciat

18) PAU LOGSE 2002 Sèrie 2 Problema 1:


Considereu les rectes r i s amb les equacions següents:
x − y + 3 = 0  y + 13 = 0
r:  s: 
2 x − z + 2 = 0  x − 2 z − 3 = 0
a) Calculeu, de cada una de les rectes, un punt i un vector director.

Per aconseguir-ho cal ficar dues incògnites qualsevol de les ( x, y, z ) en funció de la


3a.
x − y + 3 = 0 x = y − 3
r: →
2 x − z + 2 = 0 2 ( y − 3) − z + 2 = 0 → 2 y − 6 − z + 2 = 0 →→ z = 2 y − 4
( x, y, z ) = ( y − 3, y,2 y − 4 ) ⎯⎯⎯
y =
→ ( x, y, z ) = ( −3,0, −4 ) +  (1,1,2 )

 y + 13 = 0  y = −31
s: → → ( x, y, z ) = ( 2 z + 3, −31 , z ) ⎯⎯⎯
z =

 x − 2z − 3 = 0  x = 2z + 3
→ ( x, y, z ) = ( 3, −31 ,0) +  ( 2,0,1)

b) Determineu si existeix cada un dels objectes següents i en cas afirmatiu calculeu la


seva equació:
i) El pla paral·lel a la recta s que conté la recta r.

Sigui  aquest pla.


Per ser paral·lel a la recta s, el vector director de s serà vector director de  . Així
vs = ( 2,0,1) vector director de  .

Si  conté la recta r aleshores el punt P = ( −3,0, −4 ) que pertany a r també pertany a

 . I el vector director de r, vr = (1,1,2 ) també serà vector director de  .


Així:
x+3 2 1
: y 0 1 = 0 → y + 2 ( z + 4 ) − ( x + 3) − 4 y = y + 2 z + 8 − x − 3 − 4 y =
z+4 1 2

= − x − 3y + 2z + 5 = 0 →  : x + 3y − 2z − 5 = 0

C/ Pasqual i Batlle, 1-15 08790 Gelida Telèfon: 93 779 04 50 Pàg. 42


e-mail: a8035246@xtec.cat https://agora.xtec.cat/iesgelida
Generalitat de Catalunya
Departament d’Educació
Institut Gelida Departament de Matemàtiques

ii) El pla perpendicular a la recta s que conté la recta r.

En la figura podem observar que per a que existeixi aquest pla  ' , les
rectes r i s han de ser perpendiculars.
Però vr  vs = (1,1,2)  ( 2,0,1) = 2 + 0 + 2 = 4  0 → r i s no perpendiculars,
per tant, aquest pla no existeix.

iii) La recta perpendicular a les rectes r i s que passa per (0, 0, 0).
Si anomenem t a la recta buscada, aleshores sabem que el seu vector director vt serà
perpendicular als vectors directors de r i de s. És a dir:
i j k
vt = vr  vs = (1,1,2 )  ( 2,0,1) = 1 1 2 = i + 0k + 4 j − 2k − 0i − j =
2 0 1
= i + 3 j − 2k = (1, 3, − 2 )

Per tant, la recta buscada és ( x, y, z ) = ( 0,0,0 ) +  (1,3, −2 )


Tornar a l’enunciat

19) PAU LOGSE 2002 Sèrie 3 Qüestió 3:


Considereu els plans d'equacions: 1 : x + 2 y − z = 3 i  2 : ax + ( a − 2) y + 2 z = 4 .
a) Hi ha algun valor del paràmetre a per al qual la intersecció dels plans  1 i  2 no és
una recta?

Dos plans o bé es tallen formant una recta o bé són


paral·lels o coincidents. És a dir, no es poden tallar en un
punt.
Si 1 i  2 no es tallen formant una recta aleshores són
paral·lels o coincidents. Però en aquests dos casos
(paral·lels o coincidents) els seus vectors normals
apuntaran en la mateixa direcció i per tant seran
proporcionals.
Així, estudiem el cas en que els vectors n1 = (1,2, −1) i

n2 = ( a, a − 2, 2 ) són proporcionals.

−1  a = 2 → 2 = −a → a = −2
1 −1
1 2
= = →  2 −1 → Per tant, si que hi ha un valor del
a a−2 2 =
 a −2 2 → 4 = − a + 2 → a = −2
paràmetre a pel qual els dos plans no formen una recta, aquest valor és a = −2

C/ Pasqual i Batlle, 1-15 08790 Gelida Telèfon: 93 779 04 50 Pàg. 43


e-mail: a8035246@xtec.cat https://agora.xtec.cat/iesgelida
Generalitat de Catalunya
Departament d’Educació
Institut Gelida Departament de Matemàtiques

b) Calculeu un vector director de la recta que s'obté quan es fa la intersecció de 1 i


2 per al valor del paràmetre a = 0.

Quan tenim una recta donada com a intersecció de dos plans. Podem calcular el
vector director de la recta amb el producte vectorial dels vectors normals dels plans.
Així:
 1 : x + 2 y − z = 3 a =0  : x + 2 y − z = 3  n1 = (1,2, −1)
 ⎯⎯ ⎯ → 1 →
 2 : ax + ( a − 2 ) y + 2 z = 4  2 : −2 y + 2 z = 4 n2 = ( 0, −2,2 )

i j k
n1  n2 = (1,2, −1)  ( 0, −2,2 ) = 1 2 −1 = 4i − 2k + 0 j − 0k − 2i − 2 j = ( 2, −2, −2 )
0 −2 2
Tornar a l’enunciat

20) PAU LOGSE 2002 Sèrie 3 Qüestió 4:


y −5 z −7
Considereu la recta r d'equacions: x − 1 = = . Calculeu els punts d'aquesta
−3 −4
recta situats a una distància 3 del punt A = (1, 0, 1).

En el gràfic podem observar que generalment hi ha


dos punts d’una recta que estan a una determinada
distància d’un tercer.
y −5 z −7
r : x −1 = = → ( x, y, z ) = (1,5,7 ) +  (1, −3, −4 )
−3 −4
Per tant, tots els punts de la recta r són de la forma: (1 + , 5 − 3, 7 − 4 ) .
Sigui un punt P de la recta r, P tindrà la forma: (1 +  , 5 − 3 , 7 − 4 )

Aleshores la distància entre P i A serà: d ( A, P ) = AP

AP = P − A = (1 +  , 5 − 3 , 7 − 4 ) − (1, 0, 1) = (  , 5 − 3 , 6 − 4 )

d ( A, P ) = AP = (  , 5 − 3 , 6 − 4 ) =  2 + (5 − 3 ) + ( 6 − 4 ) =
2 2

=  2 + 25 − 30 + 9 2 + 36 − 48 + 16 2 = 26 2 − 78 + 61


d ( A, P ) = 3 → 26 2 − 78 + 61 = 3 → 26 2 − 78 + 61 = 9 → 26 2 − 78 + 52 = 0 ⎯⎯→
26

 = 1
→  2 − 3 + 2 = 0 → 
 = 2
 = 1 → P = ( 2,2,3)

P = (1 +  , 5 − 3 , 7 − 4 ) → 
 = 2 → P ' = ( 3, −1, −1)

Tornar a l’enunciat

C/ Pasqual i Batlle, 1-15 08790 Gelida Telèfon: 93 779 04 50 Pàg. 44


e-mail: a8035246@xtec.cat https://agora.xtec.cat/iesgelida
Generalitat de Catalunya
Departament d’Educació
Institut Gelida Departament de Matemàtiques

21) PAU LOGSE 2003 Sèrie 2 Qüestió 3:


x −1 y +1 z
Considereu el punt P = (5, –2, 9) i la recta r : = =
−2 −3 6
a) Calculeu l’equació de la recta s que talla perpendicularment r i que passa per P.
b) Calculeu el punt de tall Q entre les rectes r i s.

Resoldrem tots dos apartats a la vegada.


• Calculem el pla  perpendicular a la recta r i que passa per
P:
P
 ⊥ r →  : −2 x − 3 y + 6 z + k = 0 ⎯⎯⎯ →
→ −2  5 − 3  ( −2 ) + 6  9 + k = 0 →
→ −10 + 6 + 54 + k = 0 → 50 + k = 0 → k = −50 →
→  : −2 x − 3 y + 6 z − 50 = 0
• Calculem el punt de tall Q entre el pla  i la recta r:

x − 1 y + 1 z  −2 = −3 → −3x + 3 = −2 y − 2 → −3x + 2 y = −5
x −1 y +1

r: = = →
−2 −3 6  x−−21 = 6z → 6 x − 6 = −2 z → 6 x + 2 z = 6 ⎯⎯2
→ 3x + z = 3
 −2 x − 3 y + 6 z − 50 = 0  −2 −3 6 50  F →3F  −6 −9 18 150  F → F − F

→ A ' =  −3 2 0 −5    −6 4 0 −10  
1 1 2 2 1

 −3x + 2 y = −5 F →2 F F →F + F
3 x + z = 3  3 0 1 3  2 2 6 0 2  3 3 1
    6 
 −6 −9 18 150 
F2 →9 F2
 −6 −9 18 150 
F3 → F3 + F2
  0  
13 −18 −160    0 117 −162 −1440  
F3 →13 F3
0 −9 20 156   0 −117 260 2028 
  
 −6 −9 18 150 
  0 117 −162 −1440 
0 588 
 0 98
588
98 z = 588 → z = →z=6
98
117 y − 162 z = −1440 → 117 y − 162  6 = −1440 → 117 y = 972 − 1440 → 117 y = −468 →
−468
→y= → y = −4
117
−6 x − 9 y + 18z = 150 → −6x − 9  ( −4 ) + 18  6 = 150 → −6 x + 36 + 108 = 150 →
→ −6x = 6 → x = −1
Així, la recta r i el pla  es tallen en el punt Q = ( −1, −4,6) .

• Finalment, la recta buscada, és la recta que passa per P = (5, −2,9 ) i

Q = ( −1, −4,6) :
PQ = Q − P = ( −1, −4,6 ) − (5, −2,9 ) = ( −6, −2, −3)

s : ( x, y, z ) = ( 5, −2,9 ) +  ( 6,2,3)
Tornar a l’enunciat

C/ Pasqual i Batlle, 1-15 08790 Gelida Telèfon: 93 779 04 50 Pàg. 45


e-mail: a8035246@xtec.cat https://agora.xtec.cat/iesgelida
Generalitat de Catalunya
Departament d’Educació
Institut Gelida Departament de Matemàtiques

22) PAU LOGSE 2003 Sèrie 3 Qüestió 4:


Considereu els punts de l’espai A = (0, –2a – 1, 4a – 2), B = (1, –3, 4), C = (3, –5, 3).
a) Comproveu que el triangle de vèrtexs A, B i C és rectangle en B per a qualsevol
valor de a.

El triangle de vèrtex A, B i C serà rectangle en B si els vectors BA i


BC són perpendiculars.
Aquests vectors seran perpendiculars si el seu producte escalar és
zero.
Aquesta perpendicularitat no dependrà del valor del paràmetre a,
si el producte escalar anterior és sempre zero i no depèn de a.
BA = A − B = ( 0, − 2a − 1, 4a − 2 ) − (1, − 3, 4 ) = ( −1, − 2a + 2, 4a − 6 )
BC = C − B = ( 3, − 5, 3) − (1, − 3, 4 ) = ( 2, − 2, − 1)
BA  BC = ( −1, − 2a + 2, 4a − 6 )  ( 2, − 2, − 1) = −2 + 4a − 4 − 4a + 6 = 0
Per tant, el triangle de vèrtex A, B i C és rectangle en B per a qualsevol valor de a.

b) Calculeu els valors de a que fan que aquest triangle sigui isòsceles.

Per a que un triangle de vèrtex A, B i C rectangle en B sigui isòsceles


s’ha de complir que els dos catets siguin iguals, és a dir, que els
vectors BA i BC mesurin el mateix.

BA = ( −1, − 2a + 2, 4a − 6 ) = ( −1) + ( −2a + 2 ) + ( 4a − 6 ) =


2 2 2

= 1 + 4a 2 − 8a + 4 + 16a 2 − 48a + 36 = 20a 2 − 56a + 41


BC = ( 2, − 2, − 1) = 22 + ( −2 ) + ( −1) = 4 + 4 + 1 = 9
2 2

BA = BC → 20a 2 − 56a + 41 = 9 → 20a 2 − 56a + 41 = 9 →


 a=2

→ 20a − 56a + 32 = 0 → 
2

 a = 45

Tornar a l’enunciat

C/ Pasqual i Batlle, 1-15 08790 Gelida Telèfon: 93 779 04 50 Pàg. 46


e-mail: a8035246@xtec.cat https://agora.xtec.cat/iesgelida
Generalitat de Catalunya
Departament d’Educació
Institut Gelida Departament de Matemàtiques

23) PAU LOGSE 2003 Sèrie 3 Problema 2:


Un segment d’extrems A = (5, 3, 1) i B = (4, 2, –1) es divideix en tres parts iguals
mitjançant dos plans perpendiculars a aquest segment. Calculeu les equacions dels
dos plans i la distància entre ells.

• Pel fet de que els plans  i ' siguin perpendiculars al segment AB tindrem que el
vector normal a aquests plans serà el vector AB .
És a dir, n = n ' = AB = B − A = ( 4, 2, − 1) − (5, 3, 1) = ( −1, − 1, − 2 )
• Calculem ara, els punts C i D que divideixen el segment AB en tres parts iguals.
1 1  −1 −1 −2   14 8 1 
C = A+ AB = ( 5, 3, 1) + ( −1, − 1, − 2 ) = ( 5, 3, 1) +  , , = , , 
3 3  3 3 3   3 3 3
2 2  −2 −2 −4   13 7 −1 
D = A + AB = ( 5, 3, 1) + ( −1, − 1, − 2 ) = ( 5, 3, 1) +  , , = , , 
3 3  3 3 3   3 3 3 
• Finalment calculem l’equació dels plans  i  ' :
 14 8 1 
 passa pel punt C =  , ,  i amb vector director AB = ( −1, − 1, − 2 ) . Així:
 3 3 3
C −14 8 1 −24
 : − x − y − 2 z + k = 0 ⎯⎯⎯ → − − 2 + k = 0 → + k = 0 → −8 + k = 0 →
3 3 3 3
→ k = 8 →  : − x − y − 2z + 8 = 0

 13 7 −1 
' passa pel punt D =  , ,  i amb vector director AB = ( −1, − 1, − 2 ) . Així:
 3 3 3 
D ' −13 7 −1 −18
 ' : − x − y − 2 z + k ' = 0 ⎯⎯⎯ → − − 2 + k ' = 0 → + k ' = 0 → −6 + k ' = 0 →
3 3 3 3
→ k ' = 6 →  ' : − x − y − 2z + 6 = 0
La distància entre els dos plans  i  ' serà la mateixa que la distància entre els punts
C i D que a la vegada serà un terç de la distància entre els punts A i B. Així:
1 1 1
d ( ,  ' ) = d ( C , D ) = d ( A, B ) = AB = ( −1, − 1, − 2 ) =
3 3 3
1 6
= ( −1) + ( −1) + ( −2 ) =
2 2 2

3 3
Tornar a l’enunciat

C/ Pasqual i Batlle, 1-15 08790 Gelida Telèfon: 93 779 04 50 Pàg. 47


e-mail: a8035246@xtec.cat https://agora.xtec.cat/iesgelida
Generalitat de Catalunya
Departament d’Educació
Institut Gelida Departament de Matemàtiques

24) PAU LOGSE 2003 Sèrie 5 Qüestió 2:


y
Determineu l'equació del pla que conté a la recta x − 1 = = z + 1 i passa per l'origen
2
de coordenades.

Si trobem dos punts A i B de la recta. Com el pla també passa per l’origen de
coordenades, ja tindrem 3 punts que determinaran el nostre pla.
y
• Trobem aleshores dos punts de la recta x − 1 = = z +1
2
Evidentment un punt pot ser A = (1,0, −1) .
Si a aquest punt A li sumem el vector director de la recta, obtindrem un altre punt de
la recta. Així, B = A + v = (1,0, −1) + (1,2,1) = ( 2,2,0)
• Finalment calculem l’equació del pla que passa pels punts A = (1,0, −1) , B = ( 2,2,0)
i O = ( 0,0,0) .
OA = A − O = (1,0, −1) − ( 0,0,0 ) = (1,0, −1)
OB = B − O = ( 2,2,0 ) − ( 0,0,0 ) = ( 2,2,0 )

x−0 1 2 x 1 2
 : y − 0 0 2 = 0 → y 0 2 = 0 → −2 y + 2 z + 2 x = 0 → 2 x − 2 y + 2 z = 0 ⎯⎯
2

z − 0 −1 0 z −1 0

→ x− y+z=0
Tornar a l’enunciat

25) PAU LOGSE 2004 Sèrie 1 Qüestió 2:


Considereu els punts de l’espai A(0, 0, 1), B(1, 1, 2) i C(0, –1, –1).
a) Trobeu l’equació del pla ABC.

• Per calcular l’equació del pla utilitzarem un punt i dos vectors. Així:
AB = B − A = (1,1,2 ) − ( 0,0,1) = (1,1,1)
AC = C − A = ( 0, −1, −1) − ( 0,0,1) = ( 0, −1, −2 )
• El pla  que passa pel punt A = ( 0,0,1) i té com a vectors directors AB = (1,1,1) i
AC = ( 0, −1, −2 ) és:

x−0 1 0 x 1 0
 : y − 0 1 −1 = 0 → y 1 −1 = 0 → −2 x − z + 1 + x + 2 y = 0 →
z − 1 1 −2 z − 1 1 −2

→  : −x + 2 y − z + 1 = 0

C/ Pasqual i Batlle, 1-15 08790 Gelida Telèfon: 93 779 04 50 Pàg. 48


e-mail: a8035246@xtec.cat https://agora.xtec.cat/iesgelida
Generalitat de Catalunya
Departament d’Educació
Institut Gelida Departament de Matemàtiques

b) Si D és el punt de coordenades (k, 0, 0), quant ha de valer k per tal que els quatre
punts A, B, C i D siguin coplanaris?

Els 4 punts A, B, C i D seran coplanaris sempre i quan el punt D estigui en el pla que
determinen A, B i C. Per tant, solament hem d’obligar a que el punt D passi pel pla
anterior.

( ) D = k ,0,0
→  : − x + 2 y − z + 1 = 0 ⎯⎯⎯⎯ → −k + 2  0 − 0 + 1 = 0 → −k + 1 = 0 → k = 1
Tornar a l’enunciat

26) PAU LOGSE 2004 Sèrie 3 Problema 2:


Tenim quatre punts a l’espai: A(0, 0, 0); B(0, 0, 2); C(0, 2, 0) i D(2, 0, 0). Es demana:
a) representeu gràficament els quatre punts;

b) calculeu el volum del tetràedre (piràmide de base triangular) ABCD;

Si calculem els vectors AB , AC i AD , sabem que el mòdul del producte mixte


d’aquestos vectors és el volum del paral·lelepípede que formen. Per altra part, el
volum del tetràedre o piràmide de base triangular que formen tres vectors és la sisena
part del volum del paral·lelepípede, per tant:

AB = B − A = ( 0, 0, 2 ) − ( 0, 0, 0 ) = ( 0, 0, 2 )
AC = C − A = ( 0, 2,0 ) − ( 0,0,0 ) = ( 0, 2,0 )
AD = D − A = ( 2,0,0 ) − ( 0,0,0 ) = ( 2,0,0 )

0 0 2
1 1 1 1 1 8 4
VPiràmide = VParal ·lelepípede =   AB, AC , AD  =  0 2 0 =  −8 =  8 = =
6 6 6 6 6 6 3
2 0 0

C/ Pasqual i Batlle, 1-15 08790 Gelida Telèfon: 93 779 04 50 Pàg. 49


e-mail: a8035246@xtec.cat https://agora.xtec.cat/iesgelida
Generalitat de Catalunya
Departament d’Educació
Institut Gelida Departament de Matemàtiques

c) trobeu l’equació del pla que passa per B, C i D;

Podem trobar l’equació del pla a partir d’un punt i dos vectors.
BC = C − B = ( 0, 2,0 ) − ( 0,0, 2 ) = ( 0, 2, −2 )
BD = D − B = ( 2,0,0 ) − ( 0,0, 2 ) = ( 2,0, −2 )
Finalment l’equació general del pla  que passa pel punt B = ( 0,0, 2 ) i té com a

vectors directors BC = ( 0, 2, −2 ) i BD = ( 2, 0, −2 ) és:


x−0 0 2
 : y − 0 2 0 = 0 → −4 x − 4 y − 4 ( z − 2 ) = 0 ⎯⎯⎯
( −4 )
→ x+ y+ z−2 =0
z − 2 −2 −2

d) calculeu la distància de l’origen al pla de l’apartat anterior.

Sabem que donat un punt P = ( p1 , p2 , p3 ) i un pla  : Ax + By + Cz + D = 0 podem


Ap1 + Bp2 + Cp3 + D
calcular la distància entre el punt i el pla com: d ( P,  ) = . En el
A2 + B 2 + C 2
nostre cas:
0+0+0−2 −2
P = ( 0,0,0 ) ,  : x + y + z − 2 = 0 i d ( P,  ) =
2 2 3
= = =
1 +1 +1
2 2 2
3 3 3
Tornar a l’enunciat

27) PAU LOGSE 2004 Sèrie 4 Qüestió 1:


Considereu els punts de l’espai A(1, 1, 2), B(0, 1, 1) i C(k, 1, 5).
a) Trobeu l’equació de la recta que passa per A i B.

AB = B − A = ( 0,1,1) − (1,1, 2 ) = ( −1,0, −1) proporcional a v = (1,0,1)


Finalment la recta que passa per A i per B serà la recta que passa pel punt A = (1,1, 2)
amb vector director v = (1,0,1) , és a dir: r : ( x, y, z ) = (1,1, 2 ) +  (1, 0,1)

Altres equacions de la recta r serien:


x −1 y −1 z − 2
Equacions continues: r : = =
1 0 1
x −1 y −1 z − 2 0  ( x − 1) = 1 ( y − 1)  y − 1 = 0 y =1
r: = = → → → r:
1 0 1 1 ( x − 1) = 1 ( z − 2 )  x − 1 = z − 2
  x − z = −1

b) Per a quins valors de k els punts A, B i C formen un triangle?

Tres punts formen triangle sii no estan alineats, és a dir, tres punts A, B i C formen triangle
sii els vectors AB i AC són proporcionals.
C/ Pasqual i Batlle, 1-15 08790 Gelida Telèfon: 93 779 04 50 Pàg. 50
e-mail: a8035246@xtec.cat https://agora.xtec.cat/iesgelida
Generalitat de Catalunya
Departament d’Educació
Institut Gelida Departament de Matemàtiques

Estudiem dons la proporcionalitat o no dels vectors AB i AC .


AB = B − A = ( 0,1,1) − (1,1, 2 ) = ( −1,0, −1)
AC = C − A = ( k ,1,5) − (1,1, 2 ) = ( k − 1,0,3)

AB = ( −1,0, −1) i AC = ( k − 1,0,3) proporcionals sii


k −1 0 3
= = → −1 ( k − 1) = −1 3 → k − 1 = 3 → k = 4
−1 0 −1

Per tant, donat que els tres punts A, B i C estan alineats sii k =4 podem afirmar que
aquestos tres punts formaran triangle sii k  4
Tornar a l’enunciat

28) PAU LOGSE 2004 Sèrie 4 Problema 2:


 x = −3 + 2t

Considereu la recta r d’equació  y = 5 − 2t i el punt M (2, 3, 7).
z = 3 + t

a) Trobeu, en funció de t, la distància de M a un punt qualsevol de la recta r.
Qualsevol punt P de la recta r serà de la forma P = ( −3 + 2t , 5 − 2t , 3 + t )
La distància entre els punts P i M serà el mòdul del vector que formen.
MP = P − M = ( −3 + 2t , 5 − 2t , 3 + t ) − ( 2,3,7 ) = ( −5 + 2t , 2 − 2t , − 4 + t )
Finalment:

d ( M , P ) = MP = ( −5 + 2t , 2 − 2t , − 4 + t ) = ( −5 + 2t ) + ( 2 − 2t ) + ( −4 + t ) =
2 2 2

= 25 − 20t + 4t 2 + 4 − 8t + 4t 2 + 16 − 8t + t 2 = 9t 2 − 36t + 45 =
= 9 ( t 2 − 4t + 5) = 3 t 2 − 4t + 5

b) Trobeu les coordenades dels punts A i B de r situats a distància 3 2 del punt M.

d ( P, M ) = 3 2 → 3 t 2 − 4t + 5 = 3 2 → t 2 − 4t + 5 = 2 →
t = 1
→ t 2 − 4t + 5 = 2 → t 2 − 4t + 3 = 0 → 
t = 3
Substituint en l’expressió del punt P : P = ( −3 + 2t , 5 − 2t , 3 + t ) tenim que:
Per tant:
Quan t = 1 tenim: A = ( −3 + 2 1, 5 − 2 1, 3 + 1) → A = ( −1,3, 4 )
I quan t = 3 : B = ( −3 + 2  3, 5 − 2  3, 3 + 3) → B = ( 3, −1, 6 )

C/ Pasqual i Batlle, 1-15 08790 Gelida Telèfon: 93 779 04 50 Pàg. 51


e-mail: a8035246@xtec.cat https://agora.xtec.cat/iesgelida
Generalitat de Catalunya
Departament d’Educació
Institut Gelida Departament de Matemàtiques

c) El triangle AMB, és rectangle en M?


Tenim que M = ( 2,3,7 ) , A = ( −1,3,4) i B = ( 3, −1,6)
El triangle AMB serà rectangle en M sii els vectors MA i MB són ortogonals. És a dir, si
el seu producte escalar és zero. Anem a comprovar aquesta última condició.
MA = A − M = ( −1,3, 4 ) − ( 2,3,7 ) = ( −3,0, −3)
MB = B − M = ( 3, −1,6 ) − ( 2,3,7 ) = (1, −4, −1)
MA  MB = ( −3,0, −3)  (1, −4, −1) = −3 + 0 + 3 = 0 per tant el triangle AMB és rectangle
en M.

d) Els punts A i B formen part d’un paral·lelogram de vèrtexs ABCD que té el centre de
simetria en el punt M. Calculeu les coordenades de C i D.

Evidentment, abans de començar necessitem un dibuix de


la situació.
Podem observar que el punt C es pot calcular com:
C = M + AM = ( 2,3,7 ) + ( 3,0,3) = ( 5,3,10 )
Anàlogament:
D = M + BM = ( 2,3,7 ) + ( −1, 4,1) = (1,7,8)
Tornar a l’enunciat

29) PAU LOGSE 2004 Sèrie 5 Problema 2:


 x = 1 + 3t
x − 2 y +1 z 
Considereu les rectes r : = = i s :  y = −1 − 4t
−2 1 −2 z = 5 + t

a) Estudieu la seva posició relativa.

De les equacions de les rectes podem extreure que:


vr = ( −2,1, −2) , vs = ( 3, −4,1) A = ( 2, −1,0)  r i B = (1, −1,5)  s

Evidentment vr i v s no proporcionals. Per tant, les rectes r i s no poden ser ni paral·leles


ni coincidents. Així r i s o bé són secants en un punt o bé es creuen.
Per saber en quin dels dos casos ens trobem utilitzarem un argument que ha sortit
diversos cops durant el curs. És el següent: Si les dues rectes r i s es tallen en un punt
aleshores els vectors vr , v s i el vector AB són coplanaris i per tant la matriu que es
forma amb tots 3 té rang 2 mentre que si les rectes r i s es creuen aleshores aquestos
vectors no són coplanaris i per tant aquesta matriu té rang 3.
AB = B − A = (1, −1,5) − ( 2, −1,0 ) = ( −1,0,5 )
−2 3 −1
vr , vs , AB = 1 −4 0 = 32  0 → r i s es creuen.
−2 1 5

C/ Pasqual i Batlle, 1-15 08790 Gelida Telèfon: 93 779 04 50 Pàg. 52


e-mail: a8035246@xtec.cat https://agora.xtec.cat/iesgelida
Generalitat de Catalunya
Departament d’Educació
Institut Gelida Departament de Matemàtiques

b) Trobeu l’equació del pla que conté s i és paral·lel a r.

El pla  que conté a la recta s i és paral·lel a la recta r té com a vectors directors els
dos vectors de les dues rectes ( vr i vs ) i per contenir a la recta s també podem
assegurar que aquest pla passa pel punt B = (1, −1,5)  s . Així:
vr = ( −2,1, −2) i vs = ( 3, −4,1) vectors directors de  i B = (1, −1,5)  s . Per tant:
x − 1 −2 3
 : y + 1 1 −4 = 0 → ( x − 1) − 6 ( y + 1) + 8 ( z − 5 ) − 3 ( z − 5 ) − 8 ( x − 1) + 2 ( y + 1) = 0 →
z − 5 −2 1
→ −7 ( x −1) − 4 ( y + 1) + 5 ( z − 5) = 0 → −7 x + 7 − 4 y − 4 + 5z − 25 = 0 →
→  : −7 x − 4 y + 5 z − 22 = 0 →  : 7 x + 4 y − 5 z + 22 = 0

c) Calculeu la distància entre r i s.

Podem calcular la distància entre dues rectes r i s que es creuen mitjançant la fórmula:
vr , vs , AB 
 
d ( r, s ) = .
vr  vs
Cal notar que el numerador de la fracció anterior és el valor absolut del producte mixt
dels vectors vr , v s i AB que hem calculat en l’apartat a). Queda per calcular dons el
denominador.
i j k
vr  vs = ( −2,1 − 2 )  ( 3, −4,1) = −2 1 −2 = 1i + 8k − 6 j − 3k − 8i + 2 j = ( −7, −4,5 )
3 −4 1
Finalment:
vr , vs , AB 
  32 32 32
d ( r, s ) = = = = =
vr  vs ( −7, −4,5) ( −7 ) + ( −4 )
2 2
+ 52 49 + 16 + 25

32 16 10
= =
90 15
Tornar a l’enunciat

30) PAU LOGSE 2005 Sèrie 1 Qüestió 3:


x − 3 y −1 z + 2
Trobeu la distància entre la recta r : = = i el pla  : 3x + 4y + 7 = 0.
4 −3 3

Sabem que les posicions relatives entre una recta i un pla són: 1) Què la recta sigui
secant al pla en un punt. 2) Que la recta i el pla siguin paral·lels. i 3) Que la recta
estigui continguda en el pla.

C/ Pasqual i Batlle, 1-15 08790 Gelida Telèfon: 93 779 04 50 Pàg. 53


e-mail: a8035246@xtec.cat https://agora.xtec.cat/iesgelida
Generalitat de Catalunya
Departament d’Educació
Institut Gelida Departament de Matemàtiques

Al calcular la distància entre la recta i el pla tindrem que si la recta és secant al pla en
un punt o la recta està continguda en el pla aleshores la distància entre recta i pla
serà 0.
Per tant, l’única possibilitat per a que la distància entre r i  sigui no nul·la és que r i
 siguin paral·lels.

Comencem dons estudiant la posició relativa entre la recta r i el pla  . Aquesta


posició relativa la podem trobar per Rangs o també de la manera següent:

Tal i com es pot observar en la figura adjunta si la recta r i


el pla  són paral·lels, aleshores el vector director de r , vr i
el vector normal de  , n són ortogonals. És a dir, el seu
producte escalar ha de ser zero. Comprovem-ho.
vr  n = ( 4, −3,3)  (3,4,0) = 12 −12 + 0 = 0 → vr ⊥ n i per
tant la recta r o bé és paral·lela al pla  o bé està continguda en aquest.

Per sortir d’aquesta disjuntiva, agafem un punt de r i comprovem si pertany o no a  .


Sigui P = ( 3,1, −2)  r , la pregunta és ¿ P   ?
Dons substituïm el punt P en l’equació de  i tenim:
3  3 + 4 1 + 7 = 9 + 4 + 7 = 20  0 → P   → r i  paral·lels.

Finalment, una vegada comprovat que r i  són paral·lels, la distància entre la recta
r i el pla  serà la de qualsevol punt P de r a  i es podrà calcular mitjançant la
fórmula:
Ap1 + Bp2 + Cp3 + D
P = ( p1 , p2 , p3 ) i  : Ax + By + Cz + D = 0 aleshores d ( P,  ) =
A2 + B 2 + C 2
En el nostre cas, agafant P = ( 3,1, −2)  r i  : 3x + 4 y + 7 = 0 tenim que:

3  3 + 4 1 + 7 9+4+7 20 20
d ( P,  ) = = = = = 4
32 + 42 + 02 25 5 5
Tornar a l’enunciat

31) PAU LOGSE 2005 Sèrie 3 Qüestió 3:


x − 3 y −1 z + 2
Trobeu la distància entre la recta r : = = i el pla  : 2x – 3y + 3z + 5 = 0.
2 −3 3

Ja hem treballat en diferents ocasions i ho hem repetit en el


problema anterior que abans de ficar-nos a calcular
distàncies entre objectes cal estudiar la posició relativa
perquè si aquestos objectes tenen algun punt en comú
aleshores la distància serà zero.
Per tant, el primer que hem de fer és estudiar la posició relativa
entre la recta r i el pla  .
Sempre tenim l’opció de calcular la posició relativa raonant
per Rangs però en aquest cas hi ha un mètode molt més
ràpid. És el següent:

C/ Pasqual i Batlle, 1-15 08790 Gelida Telèfon: 93 779 04 50 Pàg. 54


e-mail: a8035246@xtec.cat https://agora.xtec.cat/iesgelida
Generalitat de Catalunya
Departament d’Educació
Institut Gelida Departament de Matemàtiques

Podem observar que en aquest cas el vector director de la recta r , vr = ( 2, −3,3) i el

vector normal al pla  , n = ( 2, −3 − 3) coincideixen.


Això voldrà dir, tal i com s’aprecia en la figura adjunta, que la recta r i el pla  són
perpendiculars, per tant secants, i per tant la distància entre ells serà zero.
Tornar a l’enunciat

32) PAU LOGSE 2005 Sèrie 3 Qüestió 4:


Donats els punts A = (1, 0, 0) i B (0, 0, 1):
x = 1

a) Trobeu un punt C sobre la recta d’equació paramètrica  y = 1 +  que faci que el
z = 1+ 

triangle ABC sigui rectangle en C.

El triangle ABC serà rectangle en C si els vectors CA i


CB són ortogonals. És a dir, si el seu producte
escalar és zero.

Com C és un punt de la recta, aleshores C tindrà la


forma: C = (1, 1 + , 1 +  ) i per tant els vectors CA i

CB seran:

CA = A − C = (1,0,0 ) − (1, 1 +  , 1 +  ) = ( 0, − 1 −  , − 1 −  )
CB = B − C = ( 0,0,1) − (1, 1 +  , 1 +  ) = ( −1, − 1 −  , −  )
CA  CB = 0 → ( 0, − 1 −  , − 1 −  )  ( −1, − 1 −  , −  ) = 0 →
→ 0 + ( −1 −  )  ( −1 −  ) + ( −1 −  )  ( − ) = 0 → 1 + 2 +  2 +  +  2 = 0 →
 = −1

→ 2 + 3 + 1 = 0 → 
2
−1
 = 2

Per tant, en aquest cas APARENTMENT, hi ha dos punts que compleixen el requisit que
són:
• Quan  = −1 aleshores C = (1,1 −1,1 −1) = (1,0,0) que és igual al punt A i per tant en
aquest cas els punts A, B i C no formen triangle.
−1  1 1  1 1
• Quan = aleshores C =  1,1 − ,1 −  =  1, ,  diferent de A i de B, per tant,
2  2 2  2 2
en aquest cas els punts A, B i C si que formen triangle.
 1 1
Així la solució al problema és C =  1, , 
 2 2

C/ Pasqual i Batlle, 1-15 08790 Gelida Telèfon: 93 779 04 50 Pàg. 55


e-mail: a8035246@xtec.cat https://agora.xtec.cat/iesgelida
Generalitat de Catalunya
Departament d’Educació
Institut Gelida Departament de Matemàtiques

b) Trobeu l’àrea del triangle ABC.

Donat que el triangle ABC és rectangle en C tenim que els


catets CA i CB formen 90º (és a dir, són ortogonals) i per tant
els podem utilitzar com a base i com a altura. Per tant:
 1 1   −1 −1 
CA = A − C = (1, 0, 0 ) − 1, ,  =  0, ,  →
 2 2  2 2 
 −1   −1 
2 2
1 1 2 2
→ CA = 02 +   +   == 0 + + = =
 2   2  4 4 4 2

−1 1  −1
2 2
 1 1 
( −1) +   +   =
1
CB = B − C = ( 0, 0,1) − 1, ,  =  −1, ,  → CB =
2

 2 2  2 2  2  2
1 1 6 3
= 1+ + = =
4 4 4 2

Base x Altura 2
2
 3
3
3
Finalment: Àrea triangle = = = =
2 2
unitats d’àrea.
2 2 2 4
Tornar a l’enunciat

33) PAU LOGSE 2005 Sèrie 4 Problema 2:


Una piràmide de base quadrada té el vèrtex en el pla d’equació z = 3. Tres dels vèrtexs
de la base són els punts del pla OXY: A = (1, 0, 0), B = (1, 1, 0) i C = (0, 1, 0).
a) Feu un gràfic dels elements del problema. Quines són les coordenades del quart
vèrtex de la base, D?

Si heu fet el dibuix bé s’aprecia clarament que el quart


vèrtex de la base de la piràmide és l’origen de
coordenades, és a dir, el punt D és D = ( 0,0,0)

 àrea base  altura 


b) Quin és el volum de la piràmide? Volum = 
 3

La base és un quadrat de costat 1, per tant, l’àrea de la base és A = 1 1 = 1 . L’altura


de la piràmide és la distància entre els plans OXY (d’equació z = 0 ) i el pla z = 3 , per
tant, l’altura és 3. Finalment el volum de la piràmide serà:
àrea base  altura 1 3
Volum = = = 1u 3
3 3

C/ Pasqual i Batlle, 1-15 08790 Gelida Telèfon: 93 779 04 50 Pàg. 56


e-mail: a8035246@xtec.cat https://agora.xtec.cat/iesgelida
Generalitat de Catalunya
Departament d’Educació
Institut Gelida Departament de Matemàtiques

c) Si el vèrtex de la piràmide és el punt V = (a, b, 3), quina és l’equació de la recta que


conté l’altura sobre la base?

Del vèrtex de la piràmide solament ens diuen que està en


el pla z = 3 , per tant, aquest vèrtex serà un punt de la
forma V = ( a, b,3) on no podem precisar ni a ni b.
L’altura de la piràmide és el segment perpendicular a la
base, és a dir, al pla z = 0 i que passa pel vèrtex
V = ( a, b,3) . Per tant, la recta que conté a l’altura sobre la
base serà la recta perpendicular al pla z = 0 i que passa
pel punt V = ( a, b,3) (en vermell).
Però si una recta r és perpendicular a un pla  , aleshores
el vector director de la recta coincideix amb el vector
normal del pla, és a dir, vr = n .
Però  : z = 0 → n = ( 0,0,1) → n = ( 0,0,1) .
Així la recta r és la recta que passa pel punt V = ( a, b,3) amb vector director

vr = ( 0,0,1) i en conseqüència tindrà les següents equacions:

• Equació vectorial: r : ( x, y, z ) = ( a, b,3) +  ( 0,0,1)


x −a y −b z −3
• Equació continua: r : = =
0 0 1
• Equacions generals:
1 ( x − a ) = 0  ( z − 3)  x − a = 0
x −a y −b z −3  x = a
r: = = → → → r:
0 0 1 1 ( y − b ) = 0  ( z − 3)  y − b = 0
 y = b
Tornar a l’enunciat

34) PAU LOGSE 2006 Sèrie 1 Qüestió 1:


Trobeu les coordenades dels punts situats sobre la recta d’equació
( x, y, z ) = ( −1,1,1) + t (1, 2,1) que estan a distància 1 del pla 2 x + 2 y + z = 5 .

Sabem que la distància entre un punt P = ( p1 , p2 , p3 ) i un


pla  : Ax + By + Cz + D = 0 es pot calcular mitjançant la
Ap1 + Bp2 + Cp3 + D
fórmula: d ( P,  ) = .
A2 + B 2 + C 2

En el nostre cas, els punts de la recta


r : ( x, y, z ) = ( −1,1,1) + t (1, 2,1) tindran la forma

P = ( −1 + t , 1 + 2t, 1 + t ) i per tant la distància entre aquest punt i el pla


 : 2 x + 2 y + z − 5 = 0 serà:

C/ Pasqual i Batlle, 1-15 08790 Gelida Telèfon: 93 779 04 50 Pàg. 57


e-mail: a8035246@xtec.cat https://agora.xtec.cat/iesgelida
Generalitat de Catalunya
Departament d’Educació
Institut Gelida Departament de Matemàtiques

Ap1 + Bp2 + Cp3 + D 2 ( −1 + t ) + 2 (1 + 2t ) + (1 + t ) − 5


d ( P,  ) = = =
A2 + B 2 + C 2 22 + 22 + 12
−2 + 2t + 2 + 4t + 1 + t − 5 7t − 4
= =
9 3
t = 1
7t − 4 7t − 4 = 3 
d ( P,  ) = 1 → = 1 → 7t − 4 = 3 → 7t − 4 = 3 →  → 1
3 7t − 4 = −3 t =
 7
Per tant els dos punts de la recta que estan a distància 1 del pla són:
( x, y, z ) = ( −1,1,1) + t (1, 2,1) ⎯⎯
t =1
→ ( x, y, z ) = ( −1,1,1) + (1, 2,1) = ( 0,3, 2 )

( x, y, z ) = ( −1,1,1) + t (1, 2,1) ⎯⎯ → ( x, y, z ) = ( −1,1,1) + ( 17 , 72 , 17 ) = ( −76 , 97 , 87 )


t= 1
7

Com es pot veure en el gràfic que acompanya a aquest problema hi haurà dos punts
de la recta r que distaran 1 del pla  , un per sobre i l’altre per sota de  . (Són els que
estan pintats de vermell)
Tornar a l’enunciat

35) PAU LOGSE 2006 Sèrie 1 Problema 1:


Una recta r passa pel punt A = (3,0,2) i té la direcció del vector (–1,1,4).
a) Trobeu quin angle forma r amb el pla horitzontal.

L’angle que formarà la recta r i el pla horitzontal XY serà


l’angle que forma el vector director de la recta
vr = ( −1,1,4) amb el pla XY.
En la figura adjunta es veu que el pla horitzontal (pintat de
groc) és el pla que conté tots els punts de coordenada z
(altura) igual a zero. Per tant, l’equació general del pla
horitzontal és: z = 0 .

Recordem que l’angle entre una recta r de vector director vr i un pla  de vector
 vr  n 
normal n es pot calcular mitjançant la fórmula: ( r ,  ) = arcsin  
 vr  n 
En el nostre cas: vr = ( −1,1,4) , n = ( 0,0,1) vr  n = ( −1,1,4)  ( 0,0,1) = 4
vr = ( −1,1, 4 ) = ( −1) + 12 + 42 = 18 i n = ( 0, 0,1) = 02 + 02 + 12 = 1 = 1
2

 vr  n   4   4 
( r ,  ) = arcsin   = arcsin   = arcsin   70,53º
 vr  n   18 1   18 

C/ Pasqual i Batlle, 1-15 08790 Gelida Telèfon: 93 779 04 50 Pàg. 58


e-mail: a8035246@xtec.cat https://agora.xtec.cat/iesgelida
Generalitat de Catalunya
Departament d’Educació
Institut Gelida Departament de Matemàtiques

b) Comproveu que no passa pel punt B = (1,3,10).

Per comprovar que la recta r no passa pel punt B = (1,3,10)


serà suficient demostrar que aquest punt B no compleix les
equacions de r : ( x, y, z ) = ( 3,0, 2) +  ( −1,1, 4) però hi ha una
manera més ràpida. És la següent:
Sabem que A = ( 3,0, 2) és un punt de la recta r . Si B = (1,3,10) fos un altre punt de

r aleshores el vector AB hauria de ser proporcional al vector director de la recta r


que és vr = ( −1,1,4) . (Mirar figura adjunta)
AB = B − A = (1,3,10 ) − ( 3,0, 2 ) = ( −2,3,8)
−2 3 8
AB i vr proporcionals sii = = que òbviament és fals donat que la primera
−1 1 4
fracció dóna 2 mentre que la segona dóna 3 i per tant el punt B no és de la recta r .

Un altra manera que podríem dir tradicional de demostrar que el punt B no és un punt
de r és substituir-lo en l’equació de r i comprovar que dóna una contradicció, en
aquest cas tindríem: B = (1,3,10) i r : ( x, y, z ) = ( 3,0, 2) +  ( −1,1, 4) aleshores:
(1,3,10) = (3,0,2) +  ( −1,1,4) → (1,3,10) − (3,0,2) =  ( −1,1,4) → ( −2,3,8) =  ( −1,1, 4) →
−2 = − →  = 2

→ 3 =  →  = 3 que és una contradicció donat que  no pot valer
8 = 4

simultàniament 2 i 3.

c) Trobeu l’equació de la recta que passa per A i B.

A = ( 3,0, 2) , B = (1,3,10) AB = B − A = (1,3,10 ) − ( 3,0, 2 ) = ( −2,3,8) per tant, la recta

que passa per A i per B es pot escriure com: s : ( x, y, z ) = ( 3, 0, 2 ) +  ( −2,3,8 )


Tornar a l’enunciat

36) PAU LOGSE 2006 Sèrie 3 Qüestió 3:


x − y −1 = 0
Determineu l’equació del pla perpendicular a la recta r: que passa pel
x + z + 2 = 0
punt (1,1,2). Quina distància hi ha d’aquest pla a l’origen de coordenades?

Anomenem  al pla que busquem. Si el pla  és perpendicular a


la recta r , aleshores el vector normal a  , n serà el vector
director de la recta r , vr tal i com es pot observar en el dibuix.

Per tant, necessitem calcular el vector director de

C/ Pasqual i Batlle, 1-15 08790 Gelida Telèfon: 93 779 04 50 Pàg. 59


e-mail: a8035246@xtec.cat https://agora.xtec.cat/iesgelida
Generalitat de Catalunya
Departament d’Educació
Institut Gelida Departament de Matemàtiques

x − y −1 = 0
r: .
x + z + 2 = 0
Recordem que quan una recta bé donada com a intersecció de dos plans (com ara)
el vector director de la recta és el producte vectorial dels vectors normals d’aquests
plans. En el nostre cas:
i j k
x − y −1 = 0
r: → vr = (1, −1, 0 )  (1, 0,1) = 1 −1 0 = −i + k − j = ( −1, −1,1)
x + z + 2 = 0 1 0 1

n = vr = ( −1, −1,1) →  : − x − y + z + D = 0 ⎯⎯⎯⎯→−


( )
P = 1,1,2 
Per tant, 1−1+ 2 + D = 0 →
→ D = 0 →  : −x − y + z = 0 →  : x + y − z = 0

Quina distància hi ha d’aquest pla a l’origen de coordenades?

O = ( 0,0,0)  perquè compleix l’equació de  , per tant, d ( O,  ) = 0

NOTA: Un altra manera d’aconseguir el vector director de la recta r i per tant el


vector normal al pla  seria trobar l’equació vectorial de la recta r a partir de les
seves equacions generals aïllant dues incògnites en funció de l’altra. El raonament
seria:
 x − y − 1 = 0 z =  x − y = 1 −2 −  − y = 1  y = −3 − 
r: ⎯⎯⎯ → → → →
x + z + 2 = 0  x = −2 −   x = −2 −   x = −2 − 
→ ( x, y, z ) = ( −2 − , −3 − ,  ) → ( x, y, z ) = ( −2, −3,0) +  ( −1, −1,1) → vr = ( −1, −1,1)
Tornar a l’enunciat

37) PAU LOGSE 2006 Sèrie 3 Problema 2:


2 x − 5 y − z − 3 = 0
Considereu la recta r: i el pla  : 2x – y + az + 2 = 0 on a és un
x − 3y − z − 2 = 0
paràmetre.
a) Trobeu un vector director de la recta i un vector perpendicular al pla.

A l’igual que abans per trobar el vector director d’una recta que ve donada
mitjançant les seves equacions generals, és a dir, com a intersecció de dos plans  ' i
 '' . És suficient calcular el producte vectorial dels vectors normals a aquestos plans. En
el nostre cas:
n ' = ( 2, −5, −1)
 ' : 2 x − 5 y − z − 3 = 0 
r =  '  '' on  → → vr = n '  n '' =
 '' : x − 3 y − z − 2 = 0 n '' = (1, −3, −1)

i j k
= ( 2, −5, −1)  (1, −3, −1) = 2 −5 −1 = 5i − 6k − j + 5k − 3i + 2 j = 2i + j − k = ( 2,1, −1)
1 −3 −1

C/ Pasqual i Batlle, 1-15 08790 Gelida Telèfon: 93 779 04 50 Pàg. 60


e-mail: a8035246@xtec.cat https://agora.xtec.cat/iesgelida
Generalitat de Catalunya
Departament d’Educació
Institut Gelida Departament de Matemàtiques

Per tant, el vector director de la recta r és vr = ( 2,1, −1)


NOTA: No cal dir que també podem trobar el vector director de la recta r calculant la
seva equació vectorial a partir de les generals. En aquest cas el que hem de fer és
anomenar  a una de les tres incògnites x , y o z i ficar les altres dues en funció de 
. En el nostre cas seria:
2 x − 5 y − z − 3 = 0 y = 2 x − 5 − z − 3 = 0 2 x − z = 3 + 5
r: ⎯⎯⎯ → →
x − 3y − z − 2 = 0  x − 3 − z − 2 = 0  x − z = 2 + 3

Restant les dues equacions tenim que: x = 1 + 2


Finalment substituint x en la segona equació:
x − z = 2 + 3 →1+ 2 − z = 2 + 3 → z = 1+ 2 − 2 − 3 → z = −1− 
Per tant:
( x, y, z ) = (1 + 2, , −1 −  ) → r : ( x, y, z ) = (1,0, −1) +  ( 2,1, −1) → vr = ( 2,1, −1)
Per altra banda, el vector normal al pla  : 2 x − y + az + 2 = 0 és n = ( 2, −1, a )

b) Quin ha de ser el valor de a per tal que la recta i el pla siguin paral·lels?

Cal notar que per saber si la recta r i el pla  són paral·lels o no podem treballar amb
les seves posicions relatives mitjançant els rangs de la matriu
de coeficients i la matriu ampliada però podem utilitzar un
altre procediment que serà substancialment més ràpid
perquè utilitza els resultats de l’apartat anterior. És el
següent:
Si la recta r , i el pla  són paral·lels, aleshores els vectors vr
i n han de ser ortogonals. És a dir, el seu producte escalar ha de donar zero. Anem a
imposar aquesta condició.
vr  n = 0 → ( 2,1, −1)  ( 2, −1, a ) = 0 → 4 − 1 − a = 0 → 3 − a = 0 → a = 3

Aquí hem comès un petit error. Perquè imposar que el vector director de la recta i el
vector normal al pla siguin perpendiculars no ens assegura que el pla i la recta siguin
paral·lels perquè també podria passar que la recta estigués continguda dintre del pla.
Per descartar aquesta possibilitat agafem un punt de la recta i mirarem que no
compleix l’equació del pla. Sigui P = (1,0, −1)  r al substituir en l’equació del pla
quan a = 3 tenim que:
→ 2 1 − 0 + 3  ( −1) + 2 = 1  0 → P   → r
P =(1,0, −1)
 : 2 x − y + 3z + 2 = 0 ⎯⎯⎯⎯ no
continguda en el pla  → r  . Per tant, aclarida esta observació ara si que podem
assegurar que pel valor del paràmetre a = 3 la recta r i el pla  són paral·lels.

C/ Pasqual i Batlle, 1-15 08790 Gelida Telèfon: 93 779 04 50 Pàg. 61


e-mail: a8035246@xtec.cat https://agora.xtec.cat/iesgelida
Generalitat de Catalunya
Departament d’Educació
Institut Gelida Departament de Matemàtiques

c) Esbrineu si existeixen valors de a per als quals la recta i el pla siguin perpendiculars.
En cas afirmatiu, calculeu-los.

Basant-nos en el mateix dibuix que en l’apartat anterior girant la recta 90º podem
veure que la recta r i el pla  són perpendiculars sii els
vectors vr i n apunten en la mateixa direcció, és a dir, si són
proporcionals. Anem dons a imposar aquesta condició:
vr i n proporcionals → ( 2,1, −1) i ( 2, −1, a ) proporcionals
2 1 −1
→ = = . Veiem que aquest cas és impossible dons la
2 −1 a
primera fracció sempre val 1 mentre que la segona sempre
val -1.

d) Esbrineu si existeixen valors de a per als quals la recta i el pla formin un angle de 30º.
En cas afirmatiu, calculeu-los.

Sabem que per calcular l’angle agut que formen una recta de vector director vr i un
vr  n
pla de vector normal n podem utilitzar la fórmula: sin  = on  és l’angle
vr  n
buscat.
En el nostre cas:
( 2,1, −1)  ( 2, −1, a ) 1 2  2 + 1 ( −1) − 1 a
sin ( 30º ) = → = →
( 2,1, −1)  ( 2, −1, a ) 2 22 + 12 + ( −1)  22 + ( −1) + a 2
2 2

1 4 −1 − a 1 3− a
→ = → = → 6  5 + a2 = 2 3 − a
2 4 +1+1  4 +1+ a 2 2 6  5+ a 2

Però elevant al quadrat tenim que:

( ) = ( 2 3 − a ) → 6  ( 5 + a 2 ) = 22 ( 3 − a ) → 30 + 6a 2 = 4 (9 − 6a + a 2 ) →
2 2
6  5 + a2
2

2
→ 30 + 6a2 = 36 − 24a + 4a 2 → 2a 2 + 24a − 6 = 0 ⎯⎯ → a 2 + 12a − 3 = 0 → a = −6  39
Tornar a l’enunciat

38) PAU LOGSE 2006 Sèrie 4 Qüestió 3:


x + y − z = 0
Calculeu l’equació de la recta paral·lela a la recta r: i que passa pel
2 x − y + z = 1
punt (0,1,0).

Anomenem r ' a aquesta recta paral·lela. És evident que r ' tindrà el mateix vector
director que r i a més passarà pel punt P = ( 0,1,0) .
Anem a calcular el vector director de r . Per fer-ho tenim dos mètodes.

• Mètode 1: El vector director d’una recta definida com a intersecció de dos plans és el
producte vectorial dels vectors normals d’aquestos plans. En el nostre cas:

C/ Pasqual i Batlle, 1-15 08790 Gelida Telèfon: 93 779 04 50 Pàg. 62


e-mail: a8035246@xtec.cat https://agora.xtec.cat/iesgelida
Generalitat de Catalunya
Departament d’Educació
Institut Gelida Departament de Matemàtiques

x + y − z = 0  : x + y − z = 0 n = (1,1, −1)

r: → r =    ' on  →
2 x − y + z = 1 n ' = ( 2, −1,1)
 ' : 2 x − y + z = 1 
i j k
Finalment vr = n  n ' = (1,1, −1)  ( 2, −1,1) = 1 1 −1 = ( 0, −3, −3 ) proporcional a
2 −1 1
v = ( 0,1,1) . Per tant, la recta buscada és: r ' : ( x, y, z ) = ( 0,1, 0 ) +  ( 0,1,1)

• Mètode 2: Calculem l’equació vectorial de la recta r a partir de les seves equacions


generals.
x + y − z = 0 z = x + y = 
r: ⎯⎯ ⎯→
2 x − y + z = 1 2 x − y = 1 − 
1
Sumant les dues equacions tenim que: 3x = 1 → x =
3
1 1
Substituint en la primera equació + y =  → y =  − així:
3 3
r : ( x, y, z ) = ( 3 ,  − 3 ,  ) → r : ( x, y, z ) = ( 3 , 3 ,0 ) +  ( 0,1,1) → vr = ( 0,1,1)
1 1 1 −1
obtenint
com és obvi el mateix vector director que amb l’altre mètode.
Una vegada sabem que el vector director de r ' és vr = ( 0,1,1) i que la recta r ' passa
pel punt P = ( 0,1,0) trobaríem l’equació vectorial de la recta r ' igual que ho hem fet
abans.

Per altra banda crec que la manera més ràpida de donar amb la recta que ens
demanen és:
x + y − z = 0
r:
La recta r ' es paral·lela a la recta per tant, la part vectorial de les
2 x − y + z = 1
equacions de les rectes r i r ' serà la mateixa. És a dir, la recta r ' serà de la forma
x + y − z = a
r ':  . Però com sabem que P = ( 0,1,0 )  r ' substituint en l’equació
2 x − y + z = b
tenim que:

0 + 1 − 0 = a a = 1 x + y − z = 1 F2 → F2 + F1 x + y − z = 1
r ':  → → r ':  ⎯⎯⎯⎯ →r ':  →
2  0 − 1 + 0 = b b = −1 2 x − y + z = −1 3x = 0

x + y − z = 1 0 + y − z = 1 y − z =1
→ r ':  → r ':  → r ': 
3x = 0 x = 0 x = 0
Tornar a l’enunciat

C/ Pasqual i Batlle, 1-15 08790 Gelida Telèfon: 93 779 04 50 Pàg. 63


e-mail: a8035246@xtec.cat https://agora.xtec.cat/iesgelida
Generalitat de Catalunya
Departament d’Educació
Institut Gelida Departament de Matemàtiques

39) PAU LOGSE 2006 Sèrie 4 Qüestió 4:


Determineu els extrems d’un segment AB sabent que el punt A pertany al pla
x −1 y − 2 z
2 x + y + z = 0 , el punt B pertany a la recta = = i el punt mitjà del segment
2 −1 3
és ( 0,0,0) .
En aquest cas, com en molts altres davant d’un
problema geomètric el més important és fer un
bon dibuix de la situació.
Anomenem O a l’origen de coordenades, és a
dir O = ( 0,0,0) . Sigui A = ( p1 , p2 , p3 ) l’extrem del
segment AB que pertany al pla  .

Donat que O és el punt mitjà del segment AB tenim que els vectors OA i BO són
iguals. És a dir OA = BO .

Però OA = A − O = ( p1 , p2 , p3 ) − ( 0,0,0 ) = ( p1 , p2 , p3 )

OA = BO
Tenim dons que:  → BO = ( p1 , p2 , p3 ) → O − B = ( p1 , p2 , p3 ) →
OA = ( p1 , p2 , p3 )

→ ( 0,0,0) − B = ( p1, p2 , p3 ) → B = ( 0,0,0) − ( p1, p2 , p3 ) = ( − p1, − p2 , − p3 )

Per tant, imposant que O = ( 0,0,0) és el punt mitjà del segment AB hem arribat a la
conclusió de que siA = ( p1 , p2 , p3 ) aleshores B = ( − p1 , − p2 , − p3 ) . (En realitat això era
obvi perquè B era el simètric de A respecte de l’origen de coordenades i per tant les
coordenades dels punts A i B són les mateixes però canviades de signe)

x −1 y − 2 z
Finalment imposem que A   : 2 x + y + z = 0 i que Br : = = .
2 −1 3
A = ( p1 , p2 , p3 )  : 2x + y + z = 0 → 2 p1 + p2 + p3 = 0
x −1 y − 2 z − p − 1 − p2 − 2 − p3
B = ( − p1 , − p2 , − p3 )  r :
= = → 1 = = →
2 −1 3 2 −1 3
 p1 + 1 = −2 p2 − 4  p1 + 2 p2 = −5
→ →
 1
3 p + 3 = 2 p3 3 p1 − 2 p3 = −3

Si ajuntem la primera equació amb les altres dues tenim el sistema:


2 p1 + p2 + p3 = 0

 p1 + 2 p2 = −5 que té com a solució ( p1 , p2 , p3 ) = ( 13 , −38 , 2 )
3 p − 2 p = −3
 1 3
Per tant la resposta al problema és:
A = ( p1 , p2 , p3 ) = ( 13 , −38 , 2 ) i B = ( − p1 , − p2 , − p3 ) = ( −31 , 83 , −2 )
Tornar a l’enunciat
C/ Pasqual i Batlle, 1-15 08790 Gelida Telèfon: 93 779 04 50 Pàg. 64
e-mail: a8035246@xtec.cat https://agora.xtec.cat/iesgelida
Generalitat de Catalunya
Departament d’Educació
Institut Gelida Departament de Matemàtiques

40) PAU LOGSE 2007 Sèrie 1 Qüestió 2:


Considereu els punts de l’espai P = (–1, a – 1, 3), Q = (0, a – 2, 1 – a) i R = (2, –1, 6 – 6a).
a) Trobeu el valor de a per al qual els tres punts estan alineats.

Els punts P, Q i R estaran alineats sii els vectors PQ i PR


PQ = Q − P = ( 0, a − 2, 1 − a ) − ( −1, a − 1, 3) = (1, − 1, − a − 2 )
PR = R − P = ( 2, − 1, 6 − 6a ) − ( −1, a − 1, 3) = ( 3, − a, 3 − 6a )
1 −1 − a − 2
PQ i PR proporcionals sii = = →
3 − a 3 − 6a
 13 = −−a1 → −a = −3 → a = 3
→  1 − a −2
 3 = 3−6 a → 3 − 6a = −3a − 6 → 9 = 3a → a = 3
Per tant, la solució del problema és a = 3

b) Quan els tres punts estan alineats, quina és l’equació de la recta que els conté?

Per saber la recta necessitem un punt i un vector director:


Punt: P = ( −1, a −1,3) ⎯⎯→
a =3
P = ( −1,3 −1,3) → P = ( −1, 2,3)
Vector director: PQ = (1, − 1, − a − 2 ) ⎯⎯→
a =3
PQ = (1, − 1, − 5)
Per tant, l’equació vectorial de la recta és: r : ( x, y, z ) = ( −1, 2,3) +  (1, −1, −5 )
Tornar a l’enunciat

41) PAU LOGSE 2007 Sèrie 1 Qüestió 4:


Trobeu l’equació de la recta continguda en el pla  : x + 2 y + 6 z − 2 = 0 , que talla els
eixos OY i OZ.

La part més difícil d’aquest problema és imaginar-se que


cal fer.
En la figura de l’esquerra tenim un pla  i els eixos de
coordenades dibuixats. En la següent figura dibuixo en
color vermell intermitent la recta r que estarà continguda
en el pla  i que tallarà l’eix OY en un punt A i l’eix OZ en
un punt B.

Una vegada imaginada la situació, resulta evident que la


recta r quedarà determinada pels punts A i B. Per tant, el
problema queda resolt si trobo els punts A i B.

Però això és força fàcil perquè el punt A és la intersecció


del pla  amb l’eix OY i el punt B és la intersecció del pla
 amb l’eix OZ.

C/ Pasqual i Batlle, 1-15 08790 Gelida Telèfon: 93 779 04 50 Pàg. 65


e-mail: a8035246@xtec.cat https://agora.xtec.cat/iesgelida
Generalitat de Catalunya
Departament d’Educació
Institut Gelida Departament de Matemàtiques

Càlcul del punt A:

Recordant que els punts de l’eix OY són els que tenen coordenades x i z igual a 0 tenim
que:
 x + 2 y + 6 z − 2 = 0 0 + 2 y + 6  0 − 2 = 0  x = 0
  
A =   OY →  x = 0 → x = 0 →  y = 1 → A = ( 0,1,0)
z = 0 z = 0 z = 0
  

Anàlogament, donat que els punts de l’eix OZ són els que tenen coordenades x i y
igual a 0 tenim que:
 x + 2 y + 6 z − 2 = 0 0 + 2  0 + 6 z − 2 = 0  x = 0
  
B =   OY →  x = 0 → x = 0 →  y = 0 → B = ( 0,0, 13 )
y = 0 y = 0 z = 1
   3

Finalment calculem l’equació de la recta que passa pels punts A i B:


Vector director: AB = B − A = ( 0,0, 13 ) − ( 0,1,0 ) = ( 0, −1, 13 )

r : ( x, y, z ) = ( 0,1,0 ) +  ( 0, −1, 13 )

Hi ha un altre mètode molt més ràpid per calcular l’equació de la recta r . En el segon
dibuix, es veu que la recta r , pel fet de tallar els eixos OY i OZ estarà continguda en el
pla YZ.
Per tant, la recta r , per una banda estarà continguda en el pla de l’enunciat
 : x + 2 y + 6 z − 2 = 0 i per l’altra en el pla YZ que té com a equació x = 0 .

Finalment podem expressar la recta r com la intersecció d’aquestos dos plans. És a


dir:
x + 2 y + 6z − 2 = 0 2 y + 6 z − 2 = 0 2  y + 3z − 1 = 0
r =   YZ →  →r: ⎯⎯→ r : 
x = 0 x = 0 x = 0
Tornar a l’enunciat

42) PAU LOGSE 2007 Sèrie 1 Problema 1:


y − 2 z −1
Considereu la recta d’equació r:x= = .
2 2
a) Expresseu el quadrat de la distància d’un punt qualsevol (x, y, z) de la recta al punt
P = (1, 2, 5) com una funció de la coordenada x.

D’entrada anem a expressar les coordenades y i z de qualsevol punt de la recta r en


funció de la coordenada x.
y − 2 z − 1 2 x = y − 2  y = 2 x + 2
r:x= = → →
2 2 2 x = z − 1 z = 2x +1
Per tant, un punt genèric de la recta r es pot expressar com Pr = ( x, 2 x + 2, 2 x + 1)
Ara, la distància entre el punt Pr de la recta r i el punt P = (1, 2,5) serà el mòdul del
vector que formen.
C/ Pasqual i Batlle, 1-15 08790 Gelida Telèfon: 93 779 04 50 Pàg. 66
e-mail: a8035246@xtec.cat https://agora.xtec.cat/iesgelida
Generalitat de Catalunya
Departament d’Educació
Institut Gelida Departament de Matemàtiques

PPr = Pr − P = ( x, 2 x + 2, 2 x + 1) − (1, 2,5) = ( x − 1, 2 x, 2 x − 4 )

( P, Pr ) = ( ( x − 1, 2 x, 2 x − 4 ) ) ( )=
2
( x − 1) + ( 2 x ) + ( 2 x − 4 )
2
=
2 2 2 2
d

= ( x − 1) + ( 2 x ) + ( 2 x − 4 ) = x 2 − 2 x + 1 + 4 x 2 + 4 x 2 − 16 x + 16 = 9 x 2 − 18 x + 17
2 2 2

Per tant, la solució a aquest apartat és: d 2 ( P, Pr ) = 9 x 2 − 18 x + 17

b) Trobeu quin valor de x fa mínima aquesta funció, deduïu quin punt Q de la recta és
el més proper a P i calculeu la distància del punt a la recta.
Hem de minimitzar la funció f ( x ) = 9 x2 − 18x + 17 .
Per fer-ho igualem a zero la primera derivada.
f ( x ) = 9x2 −18x + 17 → f ' ( x ) = 18x −18
f ' ( x ) = 0 → 18x −18 = 0 → 18x = 18 → x = 1
f '' ( x ) = 18 → f '' (1) = 18  0 → x = 1 és un mínim relatiu de la funció.
Recordem que un punt genèric de la recta r era Pr = ( x, 2 x + 2, 2 x + 1) i ara sabem
que de tots els punts de la recta r el punt Q que minimitza la distància (o el quadrat
de la distància) al punt P = (1, 2,5) és el que s’obté pel valor x =1 , és a dir:

Pr = ( x, 2 x + 2, 2 x + 1) ⎯⎯
x =1

→ Q = (1, 4,3)

Finalment ens demanen la distància entre el punt P i la recta


r . Però per construcció, aquesta distància és la distància
entre els punts P i Q .
P = (1, 2,5) , Q = (1, 4,3)
PQ = Q − P = (1, 4,3) − (1, 2,5) = ( 0, 2, −2 )

d ( P, r ) = d ( P, Q ) = PQ = ( 0, 2, −2 ) = 02 + 22 + ( −2 ) = 8 = 2 2
2

c) Escriviu l’equació de la recta que passa per P i Q i comproveu que és perpendicular


a r.
Per calcular la recta r ' que passa pels punts P i Q necessitem el vector PQ . Aquest
l’hem calculat en l’apartat anterior i és: PQ = ( 0, 2, −2 ) proporcional a v ' = ( 0,1, −1) .
Així la recta r ' passa pel punt P = (1, 2,5) i amb vector director v ' = ( 0,1, −1) , per tant:
r ' : ( x, y, z ) = (1, 2,5 ) +  ( 0,1, −1)
Finalment ens queda per comprovar que les rectes r i r ' són perpendiculars.
Però r i r ' perpendiculars sii el producte escalar dels seus vectors directors és zero.
Comprovem-ho.
v  v ' = (1, 2, 2)  ( 0,1, −1) = 1 0 + 2 1 + 2  ( −1) = 0 + 2 − 2 = 0 per tant r ⊥ r ' .
Tornar a l’enunciat
C/ Pasqual i Batlle, 1-15 08790 Gelida Telèfon: 93 779 04 50 Pàg. 67
e-mail: a8035246@xtec.cat https://agora.xtec.cat/iesgelida
Generalitat de Catalunya
Departament d’Educació
Institut Gelida Departament de Matemàtiques

43) PAU LOGSE 2007 Sèrie 2 Qüestió 1:


x + y + z = 1
Trobeu l’equació del pla perpendicular a la recta  que passa per l’origen
2 x + y = 3
de coordenades.

Anomenem  al pla que estem buscant.


 ⊥ r → n = vr
 x + y + z = 1 x =  + y + z = 1  y = 3 − 2  y = 3 − 2
r: ⎯⎯⎯ → → → →
2 x + y = 3 2 + y = 3  + ( 3 − 2 ) + z = 1  z = −2 + 
r : ( x, y, z ) = ( ,3 − 2, −2 +  ) → r : ( x, y, z ) = ( 0,3, −2 ) +  (1, −2,1) → vr = (1, −2,1)
n = vr = (1, −2,1) →  : x − 2 y + z + D = 0 ⎯⎯⎯⎯
( ) 0,0,0 
→ D = 0 →  : x − 2y + z = 0
Tornar a l’enunciat

44) PAU LOGSE 2007 Sèrie 2 Qüestió 4:


Trobeu els punts de la recta r : x − 1 = y + 2 = z que equidisten dels plans
 1 : 4 x − 3z − 1 = 0 i  2 : 3x + 4 y − 1 = 0 .

El que farem serà primer ficar un punt genèric P de la recta


r en funció d’un paràmetre. (Per exemple en funció de la
seva coordenada z ). Desprès calcularem les distàncies
d’aquest punt als plans  1 i  2 .

x −1 = z x = z +1
r : x −1 = y + 2 = z →  → → P = ( z + 1, z − 2, z )
y + 2 = z y = z − 2
Sabem que la distància entre un punt P = ( p1 , p2 , p3 ) i un pla  : Ax + By + Cz + D = 0
Ap1 + Bp2 + Cp3 + D
es pot calcular mitjançant la fórmula: d ( P,  ) = .
A2 + B 2 + C 2
En el nostre cas, P = ( z + 1, z − 2, z ) 1 : 4 x − 3z − 1 = 0 i  2 : 3x + 4 y − 1 = 0 :
,
4 ( z + 1) − 3z − 1 4 z + 4 − 3z − 1 z + 3
d ( P, 1 ) = = =
4 + ( −3)
2 2
16 + 9 5

3 ( z + 1) + 4 ( z − 2 ) − 1 3z + 3 + 4 z − 8 − 1 7z − 6
d ( P,  2 ) = = =
32 + 42 9 + 16 5

z + 3 7 z − 6 x5
d ( P,  1 ) = d ( P,  2 ) → = ⎯⎯→ z + 3 = 7 z − 6 → z + 3 =  ( 7 z − 6 ) →
5 5

C/ Pasqual i Batlle, 1-15 08790 Gelida Telèfon: 93 779 04 50 Pàg. 68


e-mail: a8035246@xtec.cat https://agora.xtec.cat/iesgelida
Generalitat de Catalunya
Departament d’Educació
Institut Gelida Departament de Matemàtiques

 9 3
 6z = 9 → z = → z =
z + 3 = 7z − 6  6 2
→ →
 z + 3 = −7 z + 6 8 z = 3 → z = 3

 8

P = ( z + 1, z − 2, z ) ⎯⎯→ P = ( 52 , −21 , 32 )
z= 3
2

P = ( z + 1, z − 2, z ) ⎯⎯→ Q = ( 118 , −813 , 83 )


z=3
8

Tornar a l’enunciat

45) PAU LOGSE 2007 Sèrie 2 Problema 2:


A l’espai es consideren els tres plans d’equacions:
1 : x + 2 y + z = 1  2 : px + y + pz = 1  3 : px + y + 2 z = 1 , on p és un paràmetre real.
a) Esbrineu per a quins valors de p els tres plans es tallen en un únic punt. Trobeu aquest
punt quan p = 1.

Evidentment els tres plans es tallaran en únic punt si el sistema format per les seves
equacions és compatible determinat. És a dir, si
Rang ( M ) = Rang ( M ') = nº incògnites = 3 .
El sistema és el següent:
x + 2 y + z = 1  1 2 1 1 1 2 1  1 2 1 1
      
 px + y + pz = 1 →  p 1 p 1 → M =  p 1 p  i M ' =  p 1 p 1
 px + y + 2 z = 1  p 1 2 1    
    p 1 2  p 1 2 1

Donat que la matriu de coeficients M és una matriu quadrada, treballarem amb el


seu determinant en detriment del mètode de Gauss. (Evidentment per Gauss el resultat
seria el mateix)
1 2 1 1 2 1
 
M =  p 1 p  → M = p 1 p = 2 + p + 2 p2 − p − p − 4 p = 2 p2 − 5 p + 2
 p 1 2
  p 1 2
p = 2
M = 0 → 2 p2 − 5 p + 2 = 0 → 
p = 2
1

Per tant, els únics valors del paràmetre p que anul·len el determinant de la matriu M
1  1
són p = 2 i p= . Així, si p  − 2,  aleshores rang ( M ) = 3
2  2
Però 3 = Rang ( M )  Rang ( M ')  nº incògnites = 3 →
→ Rang ( M ) = Rang ( M ') = 3 = nº incògnites → S.C.D. → els tres plans es tallen en un
únic punt.
 1
Per tant, la resposta al problema és: p − 2, 
 2

C/ Pasqual i Batlle, 1-15 08790 Gelida Telèfon: 93 779 04 50 Pàg. 69


e-mail: a8035246@xtec.cat https://agora.xtec.cat/iesgelida
Generalitat de Catalunya
Departament d’Educació
Institut Gelida Departament de Matemàtiques

El problema també ens demana trobar el punt de tall quan p = 1 .


 1 2 1 1 1 2 1 1 F → F − F  1 2 1 1 
  p =1   2 2 1 
M ' =  p 1 p 1 ⎯⎯→ M ' = 1 1 1 1   0 −1 0 0 
F →F − F
 p 1 2 1 1 1 2 1 3 3 1  0 −1 1 0 
     
−y = 0 → y = 0
y =0
− y + z = 0 ⎯⎯→ z=0
y =0
x + 2 y + z = 1 ⎯⎯→
z =0
x =1
Per tant, el punt de tall en aquest cas és: P = (1, 0, 0 )

b) Hi ha algun valor de p que faci que la intersecció comuna sigui una recta? Si és així,
escriviu l’equació vectorial d’aquesta recta.

Sabem que si els tres plans es tallen en una recta aleshores el sistema és compatible
indeterminat. Això fa que per a que els tres plans es tallen en una recta els rangs de les
matrius han de ser: Rang ( M ) = Rang ( M ') = 2 .
Però per a que Rang ( M ) sigui 2 es necessita que M = 0 . És a dir, si hi ha algun valor
del paràmetre p per al qual els tres plans es tallen en una recta comuna aquest valor
ha d’anul·lar el determinant de la matriu M i per tant, segons els càlculs de l’apartat
1
a) solament hi ha dos possibilitats. Que p sigui 2 o que p sigui . Anem a estudiar
2
per separat aquestos dos casos:

• Cas p = 2 :
1 2 1 1  1 2 1 1
  p =2    1 2 1 1 F2 →F2 −2 F1
M '= p 1 p 1 ⎯⎯⎯ → M ' =  2 1 2 1    
p 1 2     2 1 2 1
 1  2 1 2 1

F2 → F2 − 2 F1 1 2 1 1
   → Rang ( M ) = Rang ( M ') = 2  3 = nº incògnites → S .C.I .
 0 −3 0 −1
amb un grau de llibertat → els tres plans es tallen en una recta.

• Cas p = 12 :
 1 2 1 1  1 2 1 1 F →2 F  1 2 1 1  F → F − F
  p = 12   2 2  2 2 1
M ' =  p 1 p 1 ⎯⎯⎯ → M ' =  12 1 12 1  1 2 1 2  
F →2 F F →F − F
 p 1 2 1  1 1 2 1 3 3  1 2 4 2  3 3 1
  2   

F2 → F2 − F1
 1 2 1 1
 
 0 0 0 1
F3 → F3 − F1 
 0 0 3 1
 

C/ Pasqual i Batlle, 1-15 08790 Gelida Telèfon: 93 779 04 50 Pàg. 70


e-mail: a8035246@xtec.cat https://agora.xtec.cat/iesgelida
Generalitat de Catalunya
Departament d’Educació
Institut Gelida Departament de Matemàtiques

Podem observar que l’equació de la fila 2 és impossible, per tant, el sistema és


incompatible i en aquest cas el que passa és que els tres plans mai es tallen a la
vegada. És a dir, no tenen tots tres cap punt en comú. Per tant, l’únic valor del
paràmetre p que fa que els tres plans es tallen en una recta és p = 2 .

• Equació vectorial de la recta on es tallen els tres plans en el cas p = 2 :


1 2 1 1 
En el cas p = 2 hem arribat a que el sistema és equivalent a   que en
 0 −3 0 −1
forma d’equacions seria
x + 2 y + z = 1  y = 1
y = z = y =1 1
→ → ⎯⎯ ⎯→
3 3 3

−3 y = −1 x + 2  3 + z = 1 x + z = 3 x = 3 − 
1 1 1

Per tant, el sistema format pels tres plans equival a la recta:


r : ( x, y, z ) = ( 13 −  , 13 ,  ) → r : ( x, y, z ) = ( 13 , 13 ,0 ) +  ( −1,0,1)

c) Trobeu quina és la posició relativa dels tres plans quan p = 1/2.

1
En l’apartat b) quan hem estudiat el cas p= hem arribat a la conclusió que el
2
sistema format pels tres plans era incompatible i per tant els tres plans no tenien cap
punt en comú. A partir de les matrius a les quals hem arribat també es veu que en
aquest cas Rang ( M ) = 2 i Rang ( M ') = 3 , per tant ja sabem que els tres plans no
tenen cap punt en comú i que ens trobem o bé en el cas 7 o bé en el cas 8 del llibre.

Cas 7 Cas 8

Els tres plans no tenen cap punt en comú Els tres plans no tenen cap punt en comú
i no existeixen plans paral·lels i hi ha dos plans paral·lels

Però quan p= 1
2 el sistema que queda és:

 1 2 1 1
 
M ' =  12 1 12 1 d’on resulta evident multiplicant la segona fila per 2 que el pla
 1 1 2 1
2 
determinant per la primera fila i el pla determinat per la segona fila són paral·lels. Per
1
tant, estem en el cas 8, és a dir, quan p= els tres plans no tenen cap punt en comú i
2
dos d’ells són paral·lels secants al tercer.
Tornar a l’enunciat

C/ Pasqual i Batlle, 1-15 08790 Gelida Telèfon: 93 779 04 50 Pàg. 71


e-mail: a8035246@xtec.cat https://agora.xtec.cat/iesgelida
Generalitat de Catalunya
Departament d’Educació
Institut Gelida Departament de Matemàtiques

46) PAU LOGSE 2007 Sèrie 3 Qüestió 1:


Trobeu les equacions dels plans paral·lels a  : 2 x − y + 2 z = 3 situats a 6 unitats de
distància d’aquest.

Sabem que si un pla ' és paral·lel a  , aleshores els dos plans tindran el mateix
vector normal. És a dir, n = n ' i per tant, donat que l’equació del pla  és:
 : 2 x − y + 2 z − 3 = 0 l’equació de  ' serà de la forma  ' : 2 x − y + 2 z + k = 0 .
Per tant, l’únic que necessitem és determinar el valor del paràmetre k per a que la
distància entre els plans  : 2 x − y + 2 z − 3 = 0 i  ' : 2 x − y + 2 z + k = 0 sigui 6 unitats.

Però sabem que la distància entre dos plans paral·lels  : Ax + By + Cz + D = 0 i


 : Ax + By + Cz + D ' = 0 es pot calcular mitjançant la fórmula
D − D'
d ( ,  ') = .
A2 + B 2 + C 2
−3 − k −3 − k −3 − k
En el nostre cas: d ( ,  ' ) = = =
22 + ( −1) + 22 3
2
9

−3 − k −3 − k = 18 → k = −21
d ( ,  ' ) = 6 → = 6 → −3 − k = 18 → 
3 −3 − k = −18 → k = 15

I per tant els dos plans situats a una distància de 6 unitats del pla 
de l’enunciat són:
 ' : 2 x − y + 2 z + 15 = 0 i  '' : 2 x − y + 2 z − 21 = 0

Noteu que sempre hi haurà dos plans paral·lels situats a una determinada distància del
pla  donat, un per sobre i l’altre per sota.
Tornar a l’enunciat

47) PAU LOGSE 2007 Sèrie 3 Problema 1:


Una recta r és paral·lela a la recta s : x − 1 = y − 1 = z − 1 , talla en un punt A la recta
x −1 y x − 2 y −1 z
t: = = z + 1 , i en un punt B la recta l : = = .
3 2 2 2 3
a) Trobeu l’equació del pla determinat per les rectes r i t.

L’enunciat no ens dóna l’equació de la recta r , però ens diu que és paral·lela a la
recta s . Per tant, vr = vs = (1,1,1) .
Per a que dues rectes r i t determinin un pla han de ser paral·leles (que no és el cas
perquè vr = (1,1,1) i vt = ( 3, 2,1) no són proporcionals) o secants en un punt. Per tant,
abans de començar caldria comprovar que les rectes r i n són secants en un punt.
Però això ja ens ho assegura l’enunciat quan diu que la recta r talla en un punt A la
recta t.
Per tant, una vegada feta la comprovació de rigor anem a buscar l’equació del pla 
que determinen les rectes r i t.

C/ Pasqual i Batlle, 1-15 08790 Gelida Telèfon: 93 779 04 50 Pàg. 72


e-mail: a8035246@xtec.cat https://agora.xtec.cat/iesgelida
Generalitat de Catalunya
Departament d’Educació
Institut Gelida Departament de Matemàtiques

Sabem que el pla  contindrà les dues rectes r i t, per tant, els vectors directors
d’aquestes vr i vt seran també vectors directors del pla i qualsevol punt de les rectes
pertanyerà a  .
Agafant vr = (1,1,1) , vt = ( 3, 2,1) i P = (1,0, −1)  t tenim que:

x −1 1 3 x −1 1 3
 : y−0 1 2 = 0 → y 1 2 = 0 →
z − ( −1) 1 1 z +1 1 1

→ x −1 + 3 y + 2 ( z + 1) − 3 ( z + 1) − 2 ( x − 1) − y = 0 → − ( x − 1) + 2 y − ( z + 1) = 0 →
→ −x +1+ 2 y − z −1 = 0 → −x + 2 y − z = 0 →  : x − 2 y + z = 0

b) Trobeu el punt B calculant el punt d’intersecció del pla anterior amb la recta l.

En aquest apartat ens demanen calcular el punt d’intersecció B entre el pla


x − 2 y −1 z
 : x − 2 y + z = 0 de l’apartat anterior i la recta l : = = .
2 2 3
Per obtenir-lo solament necessitem resoldre el sistema format per l’equació general del
pla  i les dues equacions generals de la recta l .
x − 2 y −1 z 2 ( x − 2 ) = 2 ( y − 1)  x − 2 = y − 1  x − y = 1

l: = = → → →
2 2 3 ( y − 1) = 2 z
3  3 y − 3 = 2 z 3 y − 2 z = 3

Així el sistema queda:

 x − 2 y + z = 0  1 −2 1 0  F →F − F  1 −2 1 0  F →F −3 F  1 −2 1 0 
   2 2 1  3 3 2 
x − y = 1 →  1 −1 0 1    0 1 −1 1    0 1 −1 1 
3 y − 2 z = 3      
  0 3 −2 3   0 3 −2 3  0 0 1 0
z = 0 → y − 0 = 1 → y = 1 → x − 2 1 + 0 = 0 → x = 2 → ( x, y, z ) = ( 2,1, 0 ) → B = ( 2,1, 0 )

c) Trobeu l’equació de la recta r.

L’enunciat ens diu que la recta r és paral·lela a la recta s : x − 1 = y − 1 = z − 1 i que


talla a la rectal en un punt B .
Però r s → vr = vs = (1,1,1) i en l’apartat anterior hem calculat el punt B que ens ha

donat B = ( 2,1,0) .
Per tant la recta r és la recta que passa pel punt B = ( 2,1,0) i té com a vector

director vr = (1,1,1) . Per tant, la seva equació continua és:


x − 2 y −1 z − 0
r: = = → r : x − 2 = y −1 = z
1 1 1

C/ Pasqual i Batlle, 1-15 08790 Gelida Telèfon: 93 779 04 50 Pàg. 73


e-mail: a8035246@xtec.cat https://agora.xtec.cat/iesgelida
Generalitat de Catalunya
Departament d’Educació
Institut Gelida Departament de Matemàtiques

d) Trobeu el punt A.

Segons l’enunciat, el punt A és el punt d’intersecció entre les rectes r i t .


Però ara, ja sabem les equacions d’aquestes dues rectes.
x −1 y
r : x − 2 = y −1 = z i t : = = z +1
3 2
Per tant:
x − 2 = z x − z = 2
r : x − 2 = y −1 = z →  →
 y −1 = z y − z =1
x −1 y  x − 1 = 3 ( z + 1)  x − 1 = 3z + 3  x − 3z = 4

t: = = z +1 →  → →
3 2  y = 2 ( z + 1)
  y = 2z + 2  y − 2z = 2
I el sistema format per les dues rectes serà:

x − z = 2 1 0 −1 2 1 0 −1 2
y − z =1   F3 → F3 − F1  
 0 1 −1 1 0 1 −1 1
 →  
 x − 3z = 4 1 0 −3 4 0 0 −2 2
 y − 2 z = 2  0 1 −2

2

0 1 −2

2

De l’equació 3) tenim que −2z = 2 → z = −1 .


z =−1
Substituint en l’equació 4): y − 2 z = 2 ⎯⎯⎯ → y + 2 = 2 → y = 0.
z =−1
Substituint en l’equació 2): y − z = 1 ⎯⎯⎯ → y +1 = 1 → y = 0 .
Substituint y i z en l’equació 1):
z =−1
x − z = 2 ⎯⎯⎯ → x − ( −1) = 2 → x + 1 = 2 → x = 1
Per tant, A = (1, 0, −1)
Tornar a l’enunciat

48) PAU LOGSE 2008 Sèrie 2 Qüestió 4:


Trobeu l’equació de la recta perpendicular al pla  : 2 x − y + z + 3 = 0 , que passa pel
punt ( −1,3,a ) del pla.

Si una recta r és perpendicular a un pla  sabem que el


vector director de la recta vr serà el vector normal al pla  .
En el nostre cas vr = n = ( 2, −1,1) .
Per tant, per determinar la recta r solament ens falta un punt.
L’enunciat ens diu que la recta r passa pel punt P = ( −1,3, a )
i que aquest punt P és un punt del pla  .
Però si P és un punt del pla  , aleshores el punt P complirà
l’equació de  , és a dir:
2  ( −1) − 3 + a + 3 = 0 → −2 − 3 + a + 3 = 0 → −2 + a = 0 → a = 2 → P = ( −1,3, 2 ) i per

tant: r : ( x, y, z ) = ( −1,3, 2 ) + k ( 2, −1,1)


Tornar a l’enunciat

C/ Pasqual i Batlle, 1-15 08790 Gelida Telèfon: 93 779 04 50 Pàg. 74


e-mail: a8035246@xtec.cat https://agora.xtec.cat/iesgelida
Generalitat de Catalunya
Departament d’Educació
Institut Gelida Departament de Matemàtiques

49) PAU LOGSE 2008 Sèrie 2 Problema 2:


x − 2 y +1 z x −1 y + 7 z + 5
Donades les rectes r: = = i s: = = i el punt P = (1,1, −1)
1 2 −1 1 2 3
,
volem trobar l’equació de la recta que passa per P i que talla r i s. Per aconseguir-ho:
a) Trobeu l’equació general o cartesiana (és a dir, l’equació de la forma Ax + By + Cz +
D = 0) del pla  que conté la recta r i el punt P.

x − 2 y +1 z vr = (1, 2, −1)



r: = = →
1 2 −1  Q = ( 2, −1, 0 )  r
Com el pla  conté la recta r aleshores, el vector director de
r també serà vector director de  i el punt Q de r també
ho serà de  .
Per altra banda, donat que els punts P i Q seran punts del pla
 podem assegurar que el vector PQ serà vector director de
.
PQ = Q − P = ( 2, −1,0 ) − (1,1, −1) = (1, −2,1)
Així el pla  passa pel punt P = (1,1, −1) i té com a vectors directors vr = (1, 2, −1) i

PQ = (1, −2,1) , per tant:


x −1 1 1
 : y − 1 2 −2 = 0 → 2 ( x − 1) − ( y − 1) − 2 ( z + 1) − 2 ( z + 1) − 2 ( x − 1) − ( y − 1) = 0 →
z + 1 −1 1
→ −2 ( y − 1) − 4 ( z + 1) = 0 → −2 y + 2 − 4 z − 4 = 0 → −2 y − 4 z − 2 = 0 ⎯⎯⎯
( ) −2

→ y + 2z +1 = 0

b) Trobeu el punt M calculant el punt d’intersecció del pla  amb la recta s.


x − 1 y + 7 z + 5 2 x − 2 = y + 7 2 x − y = 9
s: = = → →
1 2 3 3 y + 21 = 2 z + 10 3 y − 2 z = −11
 : y + 2 z + 1 = 0 → y + 2 z = −1
2 x − y = 9

Per tant el sistema format per s i  és: 3 y − 2 z = −11 que expressat matricialment:
 y + 2 z = −1

 2 −1 0 9   2 −1 0 9   2 −1 0 9 
  F2  F3   F3 →F3 −3 F2  
 0 3 −2 −11   0 1 2 −1    0 1 2 −1 
 0 1 2 −1   0 3 −2 −11  0 0 −8 −8 
     
−8z = −8 → z = 1
y + 2 z = −1 → y + 2 1 = −1 → y + 2 = −1 → y = −3
2x − y = 9 → 2x − ( −3) = 9 → 2 x + 3 = 9 → 2 x = 6 → x = 3
Per tant: M = ( 3, −3,1)

C/ Pasqual i Batlle, 1-15 08790 Gelida Telèfon: 93 779 04 50 Pàg. 75


e-mail: a8035246@xtec.cat https://agora.xtec.cat/iesgelida
Generalitat de Catalunya
Departament d’Educació
Institut Gelida Departament de Matemàtiques

c) Trobeu l’equació de la recta que passa pels punts P i M.


P = (1,1, −1) i M = ( 3, −3,1) → PM = M − P = ( 3, −3,1) − (1,1, −1) = ( 2, −4, 2 )
proporcional a v = (1, −2,1) i per tant l’equació continua d’aquesta recta serà:

x −1 y −1 z +1
t: = =
1 −2 1

d) Comproveu que la recta trobada en l’apartat anterior és la que busquem.


Per construcció, la recta t trobada passa pel punt P = (1,1,−1) i
talla a la recta s en el punt M = (3,−3,1) . Per tant, solament resta
per comprovar que també talla a la recta r. Un dels mètodes
per a realitzar aquesta comprovació és veure que el sistema
format per les equacions d’ambdues rectes r i t és compatible
determinat,
x − 2 y +1 z  2 x − 4 = y + 1 2 x − y = 5
r: = = → →
1 2 −1 − y − 1 = 2 z  y + 2 z = −1
x − 1 y − 1 z + 1 −2 x + 2 = y − 1 2 x + y = 3
t: = = → →
1 −2 1  x −1 = z + 1 x − z = 2
Sumant la primera i la tercera equació tenim 4x = 8 , és a dir, x = 2 . D’aquí, a més a
més, y = −1. De la segona equació, z = 0 , valor que també compleix la quarta
equació. Per tant, la recta de l’apartat c) talla a la recta r en el punt (2,−1,0) .
Aquest apartat es pot raonar també sense buscar el punt d’intersecció amb r.
En efecte, la recta trobada a l’apartat c) passa per P i per M per construcció.
Com que, a més a més. està continguda al pla  , que també conté la recta r, la
intersecció entre les dues està assegurada, a menys que fossin paral·leles, que no és,
evidentment, el cas.
Tornar a l’enunciat

50) PAU LOGSE 2008 Sèrie 4 Qüestió 4:


3 x − 2 y + 2 z = 7
Donats el punt P = ( 7,5,1) , el pla  : x − 2 y − 3z = 10 i la recta r: :
 x − 6 y − 2z = 5
a) Trobeu la distància del punt P al pla  .

Donat un punt P = ( p1 , p2 , p3 ) i un pla  : Ax + By + Cz + D = 0 sabem que


Ap1 + Bp2 + Cp3 + D
d ( P,  ) = , en el nostre cas:
A2 + B 2 + C 2

 : x − 2 y − 3z = 10 →  : x − 2 y − 3 z − 10 = 0 i
7 − 2  5 − 3 1 − 10 7 − 10 − 3 − 10 −16 16 16 14 8 14
d ( P,  ) = = = = = =
12 + ( −2 ) + ( −3)
2 2
1+ 4 + 9 14 14 14 7

C/ Pasqual i Batlle, 1-15 08790 Gelida Telèfon: 93 779 04 50 Pàg. 76


e-mail: a8035246@xtec.cat https://agora.xtec.cat/iesgelida
Generalitat de Catalunya
Departament d’Educació
Institut Gelida Departament de Matemàtiques

b) Trobeu la distància del punt P a la recta r.


Recordem que la forma més ràpida de trobar la distància entre un punt P i una recta r
és: Si Q és un punt de la recta i vr és el seu vector director aleshores:

PQ  vr
d ( P, r ) =
vr
Però resulta evident que necessitem un punt i el vector director de r . Per fer-ho tenim
diferents mètodes.
Mètode 1: Expressar la recta mitjançant la seva equació vectorial resolent el sistema
que formen les seves dues equacions generals.

3x − 2 y + 2 z = 7 3x − 2 y = 7 − 2 z z = 3x − 2 y = 7 − 2


r: → ⎯⎯⎯ →
x − 6 y − 2z = 5 x − 6 y = 5 + 2z  x − 6 y = 5 + 2

7 − 2 −2
5 + 2 −6 −42 + 12 + 10 + 4 16 − 32
x= = = = − + 2
3 −2 −18 + 2 −16
1 −6

3 7 − 2
1 5 + 2 15 + 6 − 7 + 2 8 + 8
y= = = = −
− 12
3 −2 −18 + 2 −16 2

1 −6

( x, y, z ) = ( − + 2, −2 − 12 ,  ) → ( x, y, z ) = ( 2, −21 ,0) +  ( −1, −21 ,1) →


Així

→ ( x, y, z ) = ( 2, −21 ,0) +  ( 2,1, −2)

Mètode 2: Quan tenim una recta expressada amb les seves equacions generals (és a
dir, com a intersecció de dos plans) el vector director de la recta és el producte
vectorial dels vectors normals d’aquestos plans. En el nostre cas:

3x − 2 y + 2 z = 7
r: → vr = ( 3, −2, 2 )  (1, −6, −2 ) = (16,8, −16 ) proporcional a
x − 6 y − 2z = 5
vr = ( 2,1, −2 ) .
Ara solament necessitem aconseguir un punt de la recta r . Mirant el sistema amb les
dues equacions podem intuir que una forma ràpida d’aconseguir un punt serà agafar
y = 0 . Aleshores, al sumar les dues equacions tindrem: 4x = 12 → x = 3 Finalment,
substituint en la segona equació:
x − 6 y − 2 z = 5 → 2 z = x − 6 y − 5 = 3 − 6  0 − 5 → 2 z = −2 → z = −1 per tant, un punt de
la recta és: Q = ( 3,0, −1)

Ara ja estem en condicions d’utilitzar la fórmula de la distància:

C/ Pasqual i Batlle, 1-15 08790 Gelida Telèfon: 93 779 04 50 Pàg. 77


e-mail: a8035246@xtec.cat https://agora.xtec.cat/iesgelida
Generalitat de Catalunya
Departament d’Educació
Institut Gelida Departament de Matemàtiques

PQ = Q − P = ( 2, −21 ,0 ) − ( 7,5,1) = ( −5, −211 , −1) i vr = ( 2,1, −2 )


PQ  vr ( −5, −211 , −1)  ( 2,1, −2 ) (12, −12, 6 ) 122 + ( −12 ) + 62
2

d ( P, r ) = = = = =
vr ( 2,1, −2 ) ( 2,1, −2 ) 2 + 1 + ( −2 )
2 2 2

144 + 144 + 36 324 18


= = = = 6
4 +1+ 4 9 3

c) Trobeu la distància de la recta r al pla  .


Recordem que abans de calcular distàncies entre elements de
l’espai hem de mirar la seva posició relativa. En aquest cas,
solament podrem calcular la distància entre la recta i el pla si
aquests són paral·lels. Però per a que una recta r sigui
paral·lela a un pla  s’ha de complir que el vector director de
la recta vr i el vector normal al pla n siguin perpendiculars.
Comprovem si és aquest el cas:
vr  n = ( 2,1, −2)  (1, −2, −3) = 2 − 2 + 6 = 6  0 → vr i n no perpendiculars → r i  es
tallen en un punt → d ( r ,  ) = 0
Tornar a l’enunciat

51) PAU LOGSE 2008 Sèrie 4 Problema 2:


x − a y z +1 x+2 y −b z−4
Les rectes r1 : = = i r2 : = = són coplanàries (és a dir,
2 1 4 1 2 −1
estan incloses en un mateix pla).
a) Expliqueu, raonadament, quina és la posició relativa d’aquestes rectes.

Si dues rectes són coplanàries no es poden creuar. Per tant, han de tallar-se o ser
paral·leles. Però els seus vectors directors són v1 = ( 2,1, 4) i v2 = (1, 2, −1) que
evidentment no són proporcionals. Per tant, les dues rectes no són paral·leles i per tant
es tallen.

b) Trobeu la relació que hi ha entre els paràmetres a i b.


x − a y z +1 x − a = 2 y x − 2 y = a
r1 : = = → → i
2 1 4 4 y = z + 1 4 y − z = 1
x + 2 y − b z − 4 2 x + 4 = y − b 2 x − y = −4 − b
r2 : = = → →
1 2 −1 − x − 2 = z − 4  x + z = 2
Per tant, el sistema format per les quatre equacions escrit en forma matricial és:
 1 −2 0 a   1 −2 0 a   1 −2 0 a 
  F3 → F3 − 2 F1   F4 → F4 + F2  
 0 4 −1 1 
  0 4 −1 1    0 4 −1 1 
 2 −1 0 −4 − b  F4 → F4 − F1  0 3 0 −4 − b − 2a   0 3 0 −4 − b − 2a 
     
1 0 1 2  0 2 1 2−a  0 6 0 3−a 

Si les rectes r1 i r2 es tallen aleshores s’ha de complir que rang ( M ) = rang ( M ') = ,
C/ Pasqual i Batlle, 1-15 08790 Gelida Telèfon: 93 779 04 50 Pàg. 78
e-mail: a8035246@xtec.cat https://agora.xtec.cat/iesgelida
Generalitat de Catalunya
Departament d’Educació
Institut Gelida Departament de Matemàtiques

= nº incògnites = 3 , per tant, les files 3 i 4 de la matriu anterior han de ser proporcionals
(Pensa que si no fossin proporcionals aleshores tindríem que rang ( M ') = 4 )
Així (0 3 0 −4 − b − 2a ) i (0 6 0 3 − a) proporcionals, i per tant:
3 −4 − b − 2a
= → 3 ( 3 − a ) = 6 ( −4 − b − 2a ) → 3 − a = 2 ( −4 − b − 2a ) →
6 3− a
→ 3 − a = −8 − 2b − 4a → 4a − a + 2b = −8 − 3 → 3a + 2b = −11

c) Trobeu els valors de a i b si el pla que les conté passa pel punt P = (2, 4, 6).

Sigui  el pla que conté les dues rectes r1 i r2 . Aleshores, donat que tota la recta r1
està continguda en  , el punt Q = ( a,0, −1)  r1 serà un punt de  i els vectors

directors de les rectesr1 i r2 que són respectivament v1 = ( 2,1, 4) i v2 = (1, 2, −1) seran
vectors directors de  . Amb aquestes dades podem calcular l’equació del pla  .
Aquesta serà:
x−a 2 1
: y 1 2 = 0 → 3x − 2 y − z − 3a − 1 = 0
z + 1 4 −1
Però P = ( 2, 4,6 )   → 3  2 − 2  4 − 6 − 3a − 1 = 0 → −3a − 9 = 0 → 3a = −9 → a = −3
I de la condició de l’apartat anterior:
3a + 2b = −11 → 3  ( −3) + 2b = −11 → −9 + 2b = −11 → 2b = −2 → b = −1
Tornar a l’enunciat

52) PAU LOGSE 2008 Sèrie 5 Qüestió 4:


x −1 y +1 z + 2
Donats el pla  : 3x − 2 y + 5 z = 6 i la recta r : = = , busqueu el punt de
2 1 −3
tall, si existeix.

x −1 y +1 z + 2  1( x − 1) = 2 ( y + 1)  x − 1 = 2 y + 2 x − 2 y = 3
r: = = →  
2 1 −3 −3 ( y + 1) = z + 2
 −3 y − 3 = z + 2 −3 y − z = 5
Per tant, el sistema format per la recta r i el pla  és:
x − 2 y = 3  1 −2 0 3  F →F −3 F  1 −2 0 3  F →4 F
   3 3 1  2 2
−3 y − z = 5 →  0 −3 −1 5    0 −3 −1 5  
F →3 F
3x − 2 y + 5 z = 6  3 −2 5 6   0 4 5 −3  3 3
    
F2 →4 F2
 1 −2 0 3  F → F + F  1 −2 0 3 
  3 3 2 
  0 −12 −4 20    0 −12 −4 20 
F3 →3 F3
 0 12 15 −9   0 0 11 11 
   
11z = 11 → z = 1
z =1
−12 y − 4 z = 20 ⎯⎯ →−12 y − 4 = 20 → −12 y = 24 → y = −2
y =−2
x − 2 y = 3 ⎯⎯⎯ → x + 4 = 3 → x = −1

C/ Pasqual i Batlle, 1-15 08790 Gelida Telèfon: 93 779 04 50 Pàg. 79


e-mail: a8035246@xtec.cat https://agora.xtec.cat/iesgelida
Generalitat de Catalunya
Departament d’Educació
Institut Gelida Departament de Matemàtiques

Per tant, el punt de tall entre la recta i el pla és: P = ( x, y, z ) = ( −1, −2,1)
NOTA: Per comprovar si la solució que hem trobat és correcta, donat que aquest punt
a de pertànyer a la vegada a la recta r i al pla  podem substituir en les seves
equacions. Evidentment el punt P ha de complir tant les equacions de r com
l’equació de  .
Tornar a l’enunciat

53) PAU LOGSE 2009 Sèrie 1 Qüestió 2:


3
Considereu en l’espai les rectes r i s, les equacions respectives de les quals són:
 x + 2 y + mz = 0
r : ( x, y, z ) = ( 4,1,0) +  ( m,1,1) i s :  en què m és un paràmetre real.
x + y + z = 1
Estudieu si hi ha cap valor d’aquest paràmetre per al qual les rectes siguin
perpendiculars i es tallin.

Les rectes r i s seran perpendiculars sii o són els seus vectors directors vr i vs sii
vr  vs = 0
r : ( x, y, z ) = ( 4,1,0) +  ( m,1,1) → vr = ( m,1,1)
 x + 2 y + mz = 0  x + 2 y = −mz z =  x + 2 y = −m
s: → ⎯⎯⎯ →
x + y + z = 1 x + y = 1− z x + y = 1− 
Ara podem resoldre el sistema per Cramer o també restant les dues equacions.
Restant-les tenim: y = −m −1 +  → y = −1 +  (1 − m)
x + y = 1 −  → x −1 +  (1 − m) = 1 −  → x = 1 −  + 1 −  + m → x = 2 +  ( m − 2)
Així
s : ( x, y, z ) = ( 2 +  ( m − 2) , −1 +  (1 − m) ,  ) → ( x, y, z ) = ( 2, −1,0 ) +  ( m − 2,1 − m,1) →
→ vs = ( m − 2,1 − m,1)
Finalment vr  vs = 0 → ( m,1,1)  ( m − 2,1 − m,1) = 0 → m ( m − 2) + 1 − m + 1 = 0 →
m = 1
→ m2 − 2m + 1 − m + 1 = 0 → m2 − 3m + 2 = 0 → 
m = 2
Per tant sabem que hi ha dos valors per als quals les dues rectes són perpendiculars.
Ara ens queda per saber si en algun d’aquestos valors es compleix també que les dues
rectes es tallen.
Sabem que dues rectes tenen 4 posicions relatives (Coincidents, Paral·leles, es tallen , i
es creuen) però en els casos que estem estudiant (m = 1 i m = 2) aquestes rectes són
perpendiculars, per tant no poden ser ni coincidents ni paral·leles. Així ens queda per
determinar, per a cada valor, m = 1 i m = 2 si r i s es tallen o es creuen.

CAS m =1:
vr = ( m,1,1) ⎯⎯→
m=1
vr = (1,1,1)
vs = ( m − 2,1 − m,1) ⎯⎯→
m=1
vs = ( −1,0,1)

C/ Pasqual i Batlle, 1-15 08790 Gelida Telèfon: 93 779 04 50 Pàg. 80


e-mail: a8035246@xtec.cat https://agora.xtec.cat/iesgelida
Generalitat de Catalunya
Departament d’Educació
Institut Gelida Departament de Matemàtiques

Sigui P = ( 4,1,0)  r i Q = ( 2, −1,0 )  s → PQ = ( −2, −2,0 ) proporcional a

PQ ' = (1,1,0 )
1 −1 1
r i s es tallen  vr , vs , PQ  = 0  vr , vs , PQ ' = 0  1 0 1 = 0
   
1 1 0
1 −1 1
Però 1 0 1 = −1  0 → r i s es creuen.
1 1 0

CAS m=2:
vr = ( m,1,1) ⎯⎯⎯
m= 2
→ vr = ( 2,1,1)
vs = ( m − 2,1 − m,1) ⎯⎯⎯
m= 2
→ vs = ( 0, −1,1)
Sigui P = ( 4,1,0)  r i Q = ( 2, −1,0 )  s → PQ = ( −2, −2,0 ) proporcional a

PQ ' = (1,1,0 )
2 0 1
r i s es tallen  vr , vs , PQ  = 0  vr , vs , PQ ' = 0  1 −1 1 = 0
   
1 1 0
2 0 1
En aquest cas 1 −1 1 = 0 → r i s es tallen.
1 1 0
Per tant la resposta al problema era m=2 .
NOTA: També haguéssim pogut començar el problema a l’inrevés, imposant primer
que les rectes r i s es tallen i desprès comprovar que pel valor obtingut ( m = 2) les
rectes són a més perpendiculars.
En aquest el raonament seria, vr = ( m,1,1) , vs = ( m − 2,1 − m,1) i donat P  r i Q  s

sabem que PQ proporcional a (1,1,0 )


m m−2 1
Aleshores r i s es tallen  vr , vs , PQ  = 0  1 1 − m 1 = 0 → m − 2 = 0 → m = 2
 
1 1 0
Una vegada sabem que l’únic valor per al qual r i s es tallen és m = 2 solament
resta per comprovar que a més ho fan perpendicularment. Per mirar-ho cal comprovar
simplement si vr  vs = 0 .
Però:
vr = ( m,1,1) ⎯⎯⎯
m= 2
→ vr = ( 2,1,1) , vs = ( m − 2,1 − m,1) ⎯⎯⎯
m= 2
→ vs = ( 0, −1,1)
i vr  vs = ( 2,1,1)  ( 0, −1,1) = 0 → vr ⊥ vs → r ⊥ s
Tornar a l’enunciat

C/ Pasqual i Batlle, 1-15 08790 Gelida Telèfon: 93 779 04 50 Pàg. 81


e-mail: a8035246@xtec.cat https://agora.xtec.cat/iesgelida
Generalitat de Catalunya
Departament d’Educació
Institut Gelida Departament de Matemàtiques

54) PAU LOGSE 2009 Sèrie 3 Qüestió 4:


Donats el pla  : x + 2 y − z = 0 i el punt P = ( 3,2,1)
a) Calculeu l’equació contínua de la recta r que passa per P i és perpendicular a  .

r ⊥  → vr = n = (1, 2, −1) → r : ( x, y, z ) = (3, 2,1) + k (1, 2, −1) i

x − 3 y − 2 z −1
l’equació contínua serà: r: = =
1 2 −1

b) Calculeu el punt simètric del punt P respecte del pla  .

Per trobar el simètric del punt P respecte del pla  fem el


següent:
1) Trobem el punt d’intersecció Q entre la recta r i el pla  .
2) Finalment P ' = Q + PQ

1) Trobem el punt d’intersecció Q entre la recta r i el pla  .

x − 3 y − 2 z −1  2 ( x − 3) = y − 2  2 x − 6 = y − 2 2 x − y = 4
r: = = →  
1 2 −1 
 − ( x − 3 ) = z − 1  − x + 3 = z − 1 x + z = 4
Per tant, el sistema format per  i r serà:

 x + 2 y − z = 0  1 2 −1 0  F →F −2 F  1 2 −1 0  F → F − F  1 2 −1 0 
   2 2 1  3 3 2 
2 x − y = 4 →  2 −1 0 4    0 −5 2 4    0 −5 2 4 
F →F − F
x + z = 4   3 3 1  0 −2 2 4   
  1 0 1 4   0 3 0 0

3y = 0 → y = 0
y =0
−5 y + 2 z = 4 ⎯⎯→ 2z = 4 → z = 2
y =0
x + 2 y − z = 0 ⎯⎯→
z =2
x − 2 = 0 → x = 2 → Q = ( 2,0, 2)

2) Calculem P ' = Q + PQ :

PQ = Q − P = ( 2,0, 2 ) − ( 3, 2,1) = ( −1, −2,1)

P ' = Q + PQ = ( 2, 0, 2 ) + ( −1, −2,1) = (1, −2,3)


Tornar a l’enunciat

C/ Pasqual i Batlle, 1-15 08790 Gelida Telèfon: 93 779 04 50 Pàg. 82


e-mail: a8035246@xtec.cat https://agora.xtec.cat/iesgelida
Generalitat de Catalunya
Departament d’Educació
Institut Gelida Departament de Matemàtiques

55) PAU LOGSE 2009 Sèrie 3 Problema 2:


3
Siguin P = (3 – 2a, b, –4), Q = (a – 1, 2 + b, 0) i R = (3, –2, –2) tres punts de l’espai .
a) Calculeu el valor dels paràmetres a i b per als quals aquests tres punts estiguin
alineats.

P , Q i R alineats  PQ i PR proporcionals.
PQ = Q − P = ( a − 1, 2 + b, 0 ) − ( 3 − 2a, b, − 4 ) = ( 3a − 4, 2, 4 )
PR = R − P = ( 3, − 2, − 2 ) − ( 3 − 2a, b, − 4 ) = ( 2a, − 2 − b, 2 )
4  2 a = 24  3a2−a 4 = 2
3a − 4
3a − 4 2
PQ i PR proporcionals  = = → → 2 →
2a −2 − b 2  −22−b = 42  −2−b = 2
3a − 4 = 4a 4a − 3a = −4   a = −4
 → →
2 = −4 − 2b 2b = −4 − 2  b = −3

b) Trobeu l’equació contínua de la recta que els conté quan estan alineats.

Sabem que els punts P , Q i R estan alineats quan a = −4 i b = −3 .


Substituint per aquestos valors tenim que P = (11, −3, −4) , Q = ( −5, −1,0) i el vector

PQ és PQ = ( 3a − 4, 2, 4 ) ⎯⎯⎯
a =−4
→ PQ = ( −16, 2, 4 ) proporcional a v = ( −8,1, 2 ) .
Així, la recta r que conté a aquestos punts és la recta que passa pel punt
P = (11, −3, −4) amb vector director v = ( −8,1, 2 ) amb equació continua:
x − 11 y − ( −3) z − ( −4 ) x − 11 y + 3 z + 4
r: = = → r: = =
−8 1 2 −8 1 2

c) Quan b = 0, trobeu els valors del paràmetre a perquè la distància entre els punts P i
Q sigui la mateixa que la distància entre els punts P i R.
P = ( 3 − 2a, b, − 4) , Q = ( a − 1, 2 + b, 0) i R = ( 3, −2, −2)
Si b=0 aleshores els punts queden: P = ( 3 − 2a, 0, − 4) , Q = ( a −1, 2, 0) i

R = ( 3, −2, −2) i per tant:

PQ = Q − P = ( a − 1, 2, 0 ) − ( 3 − 2a, 0, − 4 ) = ( 3a − 4, 2, 4 )
PR = R − P = ( 3, − 2, − 2 ) − ( 3 − 2a, 0, − 4 ) = ( 2a, − 2, 2 )
d ( P, Q ) = d ( P, R )  PQ = PR  ( 3a − 4, 2, 4 ) = ( 2a, − 2, 2 ) →

→ (3a − 4) + 22 + 42 = ( 2a ) + ( −2) + 22 → ( 3a − 4 ) + 22 + 42 = ( 2a ) + ( −2 ) + 22 →
2 2 2 2 2 2

 a = 2
→ 9a 2 − 24a + 16 + 4 + 16 = 4a 2 + 4 + 4 → 5a 2 − 24a + 28 = 0 → 
 a = 5
14

C/ Pasqual i Batlle, 1-15 08790 Gelida Telèfon: 93 779 04 50 Pàg. 83


e-mail: a8035246@xtec.cat https://agora.xtec.cat/iesgelida
Generalitat de Catalunya
Departament d’Educació
Institut Gelida Departament de Matemàtiques

d) Si b = 0, calculeu el valor del paràmetre a perquè els punts P, Q i R determinin un


triangle equilàter.

Si els punts P , Q i R determinen un triangle equilàter, aleshores per


definició de triangle equilàter, els seus tres costats mesuraran el
mateix. És a dir: d ( P, Q ) = d ( P, R ) = d ( R, Q ) Però per l’apartat c)
d’aquest mateix problema sabem que si b = 0 per a que es doni la
primera igualtat, és a dir, ( d ( P, Q) = d ( P, R )) s’ha de complir que
a=2 o a = 145 .
Per tant, ara es tractarà de determinar quin d’aquests dos valors fa que es doni també
la 2a igualtat.
Sabem que quan b=0 aleshores: P = ( 3 − 2a, 0, − 4) , Q = ( a −1, 2, 0) i

R = ( 3, −2, −2)
I per tant:
PQ = Q − P = ( a − 1, 2, 0 ) − ( 3 − 2a, 0, − 4 ) = ( 3a − 4, 2, 4 )
PR = R − P = ( 3, − 2, − 2 ) − ( 3 − 2a, 0, − 4 ) = ( 2a, − 2, 2 )
RQ = Q − R = ( a − 1, 2, 0 ) − ( 3, − 2, − 2 ) = ( a − 4, 4, 2 )
Estudiem els dos casos:

• CAS 1: a=2
d ( P, Q ) = PQ = ( 2, 2, 4 ) = 22 + 22 + 42 = 24 = 2 6

d ( P, R ) = PR = ( 4, −2, 2 ) = 42 + ( −2 ) + 22 = 24 = 2 6
2

d ( R, Q ) = RQ = ( −2, 4, 2 ) = ( −2 ) + 42 + 22 = 24 = 2 6
2

Per tant en aquest cas els tres punts formen un triangle equilàter.

• CAS 2: a = 145
 14  2 246
d ( P, Q ) = PQ =  3  − 4, 2, 4  = ( 225 , 2, 4 ) =
 5  5
 14  2 246
d ( P, R ) = PR =  2  , −2, 2  = ( 285 , −2, 2 ) =
 5  5
2 134
d ( R, Q ) = RQ = ( 145 − 4, 4, 2 ) = ( −56 , 4, 2 ) =
5

Per tant en aquest segon cas els tres punts determinen un triangle isòsceles i la solució
a aquest apartat és: a=2
Tornar a l’enunciat

C/ Pasqual i Batlle, 1-15 08790 Gelida Telèfon: 93 779 04 50 Pàg. 84


e-mail: a8035246@xtec.cat https://agora.xtec.cat/iesgelida
Generalitat de Catalunya
Departament d’Educació
Institut Gelida Departament de Matemàtiques

56) PAU LOGSE 2009 Sèrie 4 Qüestió 1:


x −1 y + 2 z − 5
Donats el punt P = (1, 2,3) i la recta r: = =
2 3 −1
a) Trobeu l’equació cartesiana (és a dir, de la forma Ax + By + Cz + D = 0) del pla 
que passa per P i és perpendicular a la recta r.

Si el pla  és perpendicular a la recta r , aleshores el vector normal al pla n


coincidirà amb el vector director de la recta vr .
Però de l’equació de la recta r tenim que el seu vector
director és: vr = ( 2,3, −1) . Per tant, n = ( 2,3, −1) i
l’equació general del pla  serà de la forma:
 : 2x + 3y − z + D = 0
Falta per determinar el paràmetre D . Aquest el
calcularem a partir de que P  .
P = (1, 2,3)  → 2 1 + 3  2 − 3 + D = 0 → 2 + 6 − 3 + D = 0 → 5 + D = 0 → D = −5 →
→  : 2x + 3y − z − 5 = 0

b) Trobeu el punt de tall entre la recta r i el pla  .

3 ( x − 1) = 2 ( y + 2 )
x −1 y + 2 z − 5  3x − 3 = 2 y + 4 3x − 2 y = 7
r: = = → → →
2 3 −1 −1( x − 1) = 2 ( z − 5 ) − x + 1 = 2 z − 10 − x − 2 z = −11

Per tant, el sistema format per r i  és:


3x − 2 y = 7

− x − 2 z = −11 que matricialment podem escriure com:
2 x + 3 y − z = 5

 3 −2 0 7   −1 0 −2 −11 F → F +3 F  −1 0 −2 −11  F → −1 F
  F1  F2   2 2 1  2 2 2
 −1 0 −2 −11    3 −2 0 7 
 F →F +2 F  0 −2 −6 −26  
 2 3 −1 5   2 3 −1 5  3 3 1  0 3 −5 −17 
     

 −1 0 −2 −11  F →F −3 F  −1 0 −2 −11 
  3 3 2 
  0 1 3 13    0 1 3 13 
 0 3 −5 −17   0 0 −14 −56 
   

−56
−14 z = −56 → z = →z=4
−14
y + 3z = 13 → y + 3  4 = 13 → y + 12 = 13 → y = 1
−x − 2z = −11 → x + 2z = 11 → x + 2  4 = 11 → x + 8 = 11 → x = 3

Així, el punt de tall entre la recta r i el pla  és: Q = (3,1, 4)


Tornar a l’enunciat

C/ Pasqual i Batlle, 1-15 08790 Gelida Telèfon: 93 779 04 50 Pàg. 85


e-mail: a8035246@xtec.cat https://agora.xtec.cat/iesgelida
Generalitat de Catalunya
Departament d’Educació
Institut Gelida Departament de Matemàtiques

57) PAU LOGSE 2009 Sèrie 4 Problema 2:


Siguin r i s dues rectes de l’espai les equacions respectives de les quals, que depenen
bx + y + 3z = 1  x y − b z +1
d’un paràmetre real b, són les següents: r: , s: = =
 x + 2 y + 5z = 1 1 b +1 −1
a) Trobeu el punt de tall de la recta r amb el pla d’equació x = 0 i el punt de tall de la
recta s amb aquest mateix pla.

• Tall de la recta r amb el pla x=0.


El sistema format per la recta r i el pla  : x = 0 és:
bx + y + 3z = 1 b  0 + y + 3z = 1  y + 3z = 1  y + 3z = 1
   F2 → F2 − 2 F1 
 x + 2 y + 5 z = 1 → 0 + 2 y + 5 z = 1 → 2 y + 5 z = 1 ⎯⎯⎯⎯→ − z = −1 →
x = 0 x = 0 x = 0 x = 0
   
→ y + 3 = 1 → y = −2 → ( x, y, z ) = ( 0, −2,1)

• Tall de la recta s amb el pla x=0.


 x y − b z + 1 ( b + 1) x = y − b ( b + 1) x − y = −b
s: = = → →
1 b +1 −1 − x = z + 1  x + z = −1
Per tant, el sistema format per la recta s i el pla  : x = 0 és:
( b + 1) x − y = −b ( b + 1)  0 − y = −b − y = −b
  
 x + z = −1 → 0 + z = −1 →  z = −1 → ( x, y, z ) = ( 0, b, −1)
x = 0 x = 0 x = 0
  

b) Calculeu un vector director per a cada una de les dues rectes.


bx + y + 3z = 1
• Vector director de la recta r: :
 x + 2 y + 5z = 1
Recordem que quan ens donen una recta amb la seva equació general, és a dir, com
a intersecció de dos plans, aleshores el vector director de la recta és el producte
vectorial dels dos vectors normals d’aquestos plans. En el nostre cas:
i j k
vr = ( b,1,3)  (1, 2,5 ) = b 1 3 = 5i + 2bk + 3 j − k − 6i − 5bj =
1 2 5
= −i + ( 3 − 5b ) j + ( 2b − 1) k = ( −1, 3 − 5b, 2b − 1)
NOTA: També podríem calcular per exemple l’equació vectorial de la recta i a partir
d’aquesta trobar el vector director però utilitzant aquesta opció els càlculs resulten
força llargs.

 x y − b z +1
• Vector director de la recta s: = = :
1 b +1 −1
En aquest cas, donat que tenim la recta s expressada mitjançant la seva equació
continua tenim directament que el seu vector director és: (1, b + 1, − 1)

C/ Pasqual i Batlle, 1-15 08790 Gelida Telèfon: 93 779 04 50 Pàg. 86


e-mail: a8035246@xtec.cat https://agora.xtec.cat/iesgelida
Generalitat de Catalunya
Departament d’Educació
Institut Gelida Departament de Matemàtiques

c) Estudieu la posició relativa de les dues rectes en funció del paràmetre b.

Utilitzant els resultats de l’apartat a) tenim que el sistema format per les rectes r i s és:

bx + y + 3 z = 1  b 1 3 1  1 0 1 −1 
 x + 2 y + 5z = 1   F1  F4  
  1 2 5 1  1 2 5 1
 →  
( b + 1) x − y = −b  b + 1 −1 0 −b   b + 1 −1 0 −b 
 x + z = −1    
  1 0 1 −1   b 1 3 1
1 0 1 −1  1 0 1 −1 
F2 → F2 − F1   F2 → 12 F2  
0 2 4 2  0 1 2 1  F3 → F3 + F2
 F3 → F3 − ( b + 1) F1     
 0 −1 −b − 1 1   0 −1 −b − 1 1  F4 → F4 − F2
F4 → F4 − bF1    
 0 1 3 − b 1+ b   0 1 3 − b 1+ b 
1 0 1 −1 1 0 1 −1 

F3 → F3 + F2 0   
 1 2 1  F4 → F4 − F3  0 1 2 1 
 
F4 → F4 − F2  0 0 −b + 1 2   0 0 −b + 1 2 
   
 0 0 −b + 1 b  0 0 0 b − 2

Aleshores:
• Si b = 2 → Rang ( M ) = Rang ( M ') = 3 = nº incògnites → S.C.D. → r i s es tallen en
un punt.
• Si b = 1 aleshores rang ( M ) = 2  3 = Rang ( M ') → S.I . → r i s són paral·leles.

• Si b  1  b  2 → Rang ( M ) = 3  4 = Rang ( M ) → S.I . → r i s es creuen.


NOTA: Si mireu la solució que proposen a l’examen de les PAU, per resoldre aquest
apartat ells utilitzen un altre raonament que nosaltres parcialment també hem fet a
classe.
Tornar a l’enunciat

58) PAU LOGSE 2010 Sèrie 1 Qüestió 1:


Trobeu l’equació general (és a dir, de la forma Ax + By + Cz + D = 0) del pla que conté
x −1 x − y − z = 0
la recta r1 : = y = 2 − z i és paral·lel a la recta r2 :  .
2 x − 2 y + z = 0

x − a1
• En primer lloc escriurem la recta r1 amb tots 3 termes expressats de la forma :
v1
• Anomenem  al pla buscat.
Si  conté la recta r1 , aleshores, el punt P = (1,0,2 ) que pertany a r1 també serà un
punt de  , i el vector director de r1 , v1 = ( 2,1, −1) serà també un vector director de 
.
Com  és paral·lel a la recta r2 , el vector director de r2 serà vector director de  .

C/ Pasqual i Batlle, 1-15 08790 Gelida Telèfon: 93 779 04 50 Pàg. 87


e-mail: a8035246@xtec.cat https://agora.xtec.cat/iesgelida
Generalitat de Catalunya
Departament d’Educació
Institut Gelida Departament de Matemàtiques

x − y − z = 0
Calculem, per tant, el vector de r2 : 
x − 2 y + z = 0
Quan tenim una recta expressada com a intersecció de dos plans podem calcular el
vector director de la recta com el producte vectorial dels vectors normals d’aquests
plans.
i j k
Així: v2 = (1, −1, −1)  (1, −2,1) = 1 −1 −1 = −i − 2k − j + k − 2i − j = −3i − 2 j − k =
1 −2 1
= ( −3, −2, −1) que si el canviem de signe per estalviar signes menys tenim que

v2 = ( 3,2,1) .

• Finalment, construïm el pla  , com el pla que passa pel punt P = (1,0,2 ) i té vectors

directors v1 = ( 2,1, −1) i v2 = ( 3,2,1) .

x −1 2 3
: y 1 2 = 0 →  : x − 1 − 3 y + 4 z − 8 − 3z + 6 + 2 x − 2 − 2 y = 0 →
z − 2 −1 1
→ 3x − 5 y + z − 5 = 0

NOTA: També podríem treure el vector director de la recta r2 calculant la seva


equació vectorial.
x − y − z = 0 1 −1 −1 0  F2 → F2 − F1  1 −1 −1 0 
r2 :  →    
 x − 2 y + z = 0 1 −2 1 0   0 −1 2 0 
− y + 2z = 0 → y = 2z
x − y − z = 0 → x − 2 z − z = 0 → x = 3z
r2 : ( x, y, z ) = (3z, 2 z, z ) → r2 : ( x, y, z ) = ( 0,0,0 ) +  (3, 2,1) → v2 = (3, 2,1)

Tornar a l’enunciat

59) PAU LOGSE 2010 Sèrie 1 Qüestió 4:


x + 5 y −1 z − 2 2 x + y + 2 z + 5 = 0
Donades les rectes r1 : = = i r2 :  :
3 2 −4 2 x − y + z + 11 = 0
a) Comproveu que són paral·leles.

Per comprovar que dues rectes són paral·leles és suficient comprovar que els seus
vectors directors són proporcionals i que si agafem un punt d’una recta, aquest punt
no pertany a l’altra recta.

x + 5 y −1 z − 2
r1 : = = → v1 = ( 3,2, −4 )
3 2 −4
C/ Pasqual i Batlle, 1-15 08790 Gelida Telèfon: 93 779 04 50 Pàg. 88
e-mail: a8035246@xtec.cat https://agora.xtec.cat/iesgelida
Generalitat de Catalunya
Departament d’Educació
Institut Gelida Departament de Matemàtiques

La recta r2 ve definida per la intersecció de dos plans. Per trobar el vector director
d’una recta definida com intersecció de dos plans fem el producte vectorial dels
vectors normals a cadascun dels plans. Així:
i j k
v2 = ( 2,1,2 )  ( 2, −1,1) = 2 1 2 = i − 2k + 4 j − 2k + 2i − 2 j = 3i + 2 j − 4k = ( 3,2, −4 )
2 −1 1
Per tant, els dos vectors directors són proporcionals.
Fins ara sabem que els vectors directors de les dues rectes són proporcionals. Això vol
dir que les rectes o bé són paral·leles o bé són coincidents. Per veure que són
paral·leles agafarem un punt d’una recta i veurem que aquest punt no pertany a
l’altra recta.
Sigui P = ( −5,1,2 )  r1 . Vegem que P  r2 .
Substituint P en les equacions de r2 tenim:

2  ( −5) + 1 + 2  2 + 5 = −10 + 1 + 4 + 5 = −10 + 10 = 0


2 x + y + 2 z + 5 = 0 P =( −5,1,2) 
r2 :  ⎯⎯⎯⎯→ 
2 x − y + z + 11 = 0 2  ( −5) − 1 + 2 + 11 = −10 − 1 + 2 + 11 = −11 + 13 = 2  0

Així P no pertany a la recta r2 perquè no compleix la segona equació i podem
afirmar que les dues rectes són paral·leles.

b) Trobeu l’equació general (és a dir, de la forma Ax + By + Cz + D = 0) del pla que les
conté.

• Podem trobar un únic pla que conté dues rectes si


aquestes es tallen o si són paral·leles no coincidents.
• En la figura adjunta podem observar que si dues
rectes r1 i r2 són paral·leles, per aconseguir el pla que
les conté és suficient aconseguir un punt P i Q de cada
recta i formar un vector PQ .
• Com ja sabem que P = ( −5,1,2 )  r1 solament
necessitarem un punt Q  r2
2 x + y + 2 z + 5 = 0
r2 : 
2 x − y + z + 11 = 0
2 x + y + 5 = 0 2 x + y = −5
Si fem z = 0 tenim  → ⎯⎯⎯ → 4 x = −16 → x = −4
Sumant

 2 x − y + 11 = 0  2 x − y = −11
2 x + y = −5 → 2  ( −4 ) + y = −5 → −8 + y = −5 → y = −5 + 8 → y = 3
Per tant, Q = ( −4,3,0)  r2
• Construïm el vector PQ = Q − P = ( −4,3,0) − ( −5,1,2 ) = (1,2, −2 )
• Finalment podem construir el pla que passa pel punt P = ( −5,1,2 ) i té com a vectors

directors v1 = ( 3,2, −4 ) i PQ = (1,2, −2 ) .

C/ Pasqual i Batlle, 1-15 08790 Gelida Telèfon: 93 779 04 50 Pàg. 89


e-mail: a8035246@xtec.cat https://agora.xtec.cat/iesgelida
Generalitat de Catalunya
Departament d’Educació
Institut Gelida Departament de Matemàtiques

x+5 3 1
 : y −1 2 2 = 0 →
z − 2 −4 −2
→ −4 ( x + 5) − 4 ( y − 1) + 6 ( z − 2 ) − 2 ( z − 2 ) + 8 ( x + 5) + 6 ( y − 1) = 0 →
→ 4 ( x + 5) + 2 ( y − 1) + 4 ( z − 2 ) = 0 → 4 x + 20 + 2 y − 2 + 4 z − 8 = 0 →
2
→ 4 x + 2 y + 4 z + 10 = 0 ⎯⎯ → 2x + y + 2z + 5 = 0
Tornar a l’enunciat

60) PAU LOGSE 2010 Sèrie 2 Qüestió 2:


ax − y = 2
Donats el pla  : 5 x + y + 3z = 4 i la recta r :  , estudieu-ne la posició
2 y + z = −3
relativa en funció del paràmetre a.

• Creem el sistema format pel pla i la recta:


5x + y + 3z = 4 5 1 3 4 

ax − y = 2 → A ' =  a −1 0 2 
2 y + z = −3  0 2 1 −3 
  
Com la matriu de coeficients és quadrada és molt útil utilitzar determinants. Així:
5 1 3
A = a −1 0 = −5 + 6a + 0 − 0 − 0 − a = 5a − 5 = 0 → 5a = 5 → a = 1
0 2 1
• Per tant, hem d’estudiar els casos a = 1 i a  1:

Cas a = 1:
 5 1 3 4  F  F  1 −1 0 2  F → F −5 F  1 −1 0 2  F2 → 1 F2
A ' =  1 −1 0 2    5 1 3 4    0 6 3 −6  
2 1 2 2 1 3

 0 2 1 −3   0 2 1 −3   0 2 1 −3 
     

 1 −1 0 2  F → F − F  1 −1 0 2 
  0 2 1 −2    0 2 1 −2 
3 3 2

 0 2 1 −3   0 0 0 −1 
   

Rang ( A) = 2  3Rang ( A' ) → S.I . → La recta r i el pla  són paral·lels.

Cas a  1:
En aquest cas Rang ( A) = 3 = Rang ( A' ) = nº incògnites → S.C.D. → r i  es tallen en
un punt.

C/ Pasqual i Batlle, 1-15 08790 Gelida Telèfon: 93 779 04 50 Pàg. 90


e-mail: a8035246@xtec.cat https://agora.xtec.cat/iesgelida
Generalitat de Catalunya
Departament d’Educació
Institut Gelida Departament de Matemàtiques

NOTA 1: Si haguéssim volgut raonar per Gauss sense utilitzar el determinant, l’argument
hagués estat el següent:
 5 1 3 4  F2 → F2 − a F1  5 1 3 4 
F2 →5 F2
5 1 3 4 
A ' =  a −1 0 2    0 10 − 4 a 
  0 −5 − a −3a 10 − 4a 
5
−5− a −3a
5 5 5 
 0 2 1 −3  0 −3  0 −3 
   2 1  2 1
Per a que les dues últimes files de la matriu A siguin proporcionals s’ha de complir que
−5 − a −3a
= → −5 − a = −6a → −5 = −5a → a = 1 obtenint evidentment els mateixos
2 1
casos que abans.

NOTA 2: També podríem resoldre el problema per un procediment més geomètric i no


tant algèbric. Seria el següent:

Un pla i una recta a l’espai poden ser paral·lels, tallar-se en un punt o la recta pot estar
continguda en el pla.

v el vector normal al pla  . Evidentment v = ( 5,1,3)


Sigui
Calculem ara el vector director de la recta r . Aquest serà:
i j k
vr = ( a, −1,0 )  ( 0,2,1) = a −1 0 = −i + 2ak + 0 j + 0k − 0i − aj = −i − aj + 2ak =
0 2 1
= ( −1, − a, 2a )

La recta i el pla es tallaran si els vectors n i vr no són


perpendiculars, és a dir, si n  vr  0
n  vr = (5,1,3)  = ( −1, − a, 2a ) = −5 − a + 6a =
= −5 + 5a = 0 → 5a = 5 → a = 1
Per tant, en el cas en que a  1 , la recta i el pla es
tallaran.
En l’altre cas, és a dir, quan a = 1 , tenim dues possibilitats, que la recta i el pla siguin
paral·lels o que la recta estigui continguda en el pla. Per decantar-nos per un cas o
l’altre agafem un punt de la recta i comprovem si aquest punt pertany al pla.
Així:
ax − y = 2 x − y = 2 x − 0 = 2
r: a =1
⎯⎯→ 
y: = 0
⎯⎯⎯ → → ( x, y, z ) = ( 2,0, −3)
2 y + z = −3 2 y + z = −3  z = −3
Tenim que P = ( 2,0, −3)  r . Comprovem si P  

→ 5  2 + 0 + 3  ( −3) = 10 + 0 − 9 = 1  4 → P  r → r i 
P = ( 2,0, −3)
 : 5x + y + 3z = 4 ⎯⎯⎯⎯
paral·lels.
Tornar a l’enunciat

C/ Pasqual i Batlle, 1-15 08790 Gelida Telèfon: 93 779 04 50 Pàg. 91


e-mail: a8035246@xtec.cat https://agora.xtec.cat/iesgelida
Generalitat de Catalunya
Departament d’Educació
Institut Gelida Departament de Matemàtiques

61) PAU LOGSE 2010 Sèrie 2 Qüestió 6:


x + 4 y −1
Considereu la recta r: = = z −1 .
−2 −1
a) Trobeu els dos punts, A i B, de la recta r que estan situats a una distància d = 6 del
punt P = (–1, 1, 2).

• En aquest tipus de problema ens interessa ficar la recta expressada amb la seva
equació vectorial perquè així depèn d’un paràmetre que és el que després forçarem
a complir la propietat que ens demanen. Així:
x + 4 y −1
r: = = z − 1 → ( x, y, z ) = ( −4,1,1) +  ( −2, −1,1) →
−2 −1
→ ( x, y, z ) = ( −4 − 2, 1 − , 1 +  )
Un punt genèric de la recta r és Q = ( −4 − 2 , 1 −  , 1 +  )

• En la imatge adjunta podem apreciar que


en principi, donat el punt P i la recta r, hi
haurà dos punts A i B de r que estaran a una
distància 6 de P.

• Calculem ara, la distància entre aquest


punt Q de la recta i el punt P = ( −1,1,2 ) que
ens dóna l’enunciat.
PQ = Q − P = Q = ( −4 − 2 , 1 −  , 1 +  ) − ( −1,1,2 ) = ( −3 − 2 , −  , − 1 +  )

d ( P, Q ) = PQ = ( −3 − 2 , −  , − 1 +  ) = ( −3 − 2 ) + ( − ) + ( −1 +  ) =
2 2 2

= 9 + 12 + 4 2 +  2 + 1 − 2 +  2 = 6 2 + 10 + 10
d ( P, Q ) = 6 →= 6 2 + 10 + 10 = 6 → 6 2 + 10 + 10 = 6 →
 = −1
→ 6 2 + 10 + 4 = 0 → 
 = 3
−2

 = −1 → A = ( −2,2,0 )

Q = ( −4 − 2 , 1 −  , 1 +  ) → 
 = −32 → B = ( −38 , 53 , 13 )

b) Trobeu l’àrea del triangle de vèrtexs A, B i P.

base  altura
L’àrea d’un triangle és A= . En el nostre cas, mirant la figura anterior,
2
agafarem com a base del triangle la distància entre els punts A i B i com altura del
triangle la distància entre el punt P i la recta r.

C/ Pasqual i Batlle, 1-15 08790 Gelida Telèfon: 93 779 04 50 Pàg. 92


e-mail: a8035246@xtec.cat https://agora.xtec.cat/iesgelida
Generalitat de Catalunya
Departament d’Educació
Institut Gelida Departament de Matemàtiques

AB = B − A = ( −38 , 53 , 1
3 ) − ( −2, 2, 0) = ( −32 , −31 , 13 )
d ( A, B ) = AB = ( −32 , −31 , 13 ) = ( −32 ) + ( −31 ) + ( 13 )
2 2 2
= 4
9
+ 19 + 19 = 6
9
= 2
3
= 3
6

QP  v
d ( P, r ) = on Q és punt qualsevol de la recta r i v és el vector director de r.
v
Agafant Q = A = ( −2,2,0) tenim:

QP = P − Q = ( −1,1,2 ) − ( −2,2,0) = (1, −1,2 )


x + 4 y −1
r: = = z − 1 → v = ( −2, −1,1)
−2 −1
i j k
QP  v = 1 −1 2 = −i − k − 4 j − 2k + 2i − j = i − 5 j − 3k = (1, −5, −3)
−2 −1 1

QP  v (1, −5, −3) 12 + ( −5) + ( −3)


2 2
1 + 25 + 9 35
d ( P, r ) = = = = =
v ( −2, −1,1) ( −2 )
2
+ ( −1) + 12
2
4 +1+1 6

base  altura d ( A, B )  d ( P, r ) 3
6
 35
35
Finalment: A = = = =
6

2 2 2 6
NOTA: Aquest apartat també es pot resoldre utilitzant que el mòdul del producte
vectorial dels vectors AB i AP és l’àrea del paral·lelogram que formen, per tant,
1
l’àrea del nostre triangle serà la meitat, és a dir, AT = AB  AP .
2
Tornar a l’enunciat

62) PAU LOGSE 2010 Sèrie 4 Qüestió 1:


Donats el pla  : x + 2 y + 3 z − 4 = 0 i els punts P = (3, 1, –2) i Q = (0, 1, 2):
a) Calculeu l’equació contínua de la recta perpendicular al pla  que passa pel punt
P.

Sigui r la recta buscada. Si r és perpendicular al pla 


aleshores el vector director de r, vr coincidirà amb el vector
normal a  que és n = (1,2,3) .
Si r també ha de complir que passa pel punt P = ( 3,1, −2 )
tindrem:
r : ( x, y, z ) = ( 3,1, −2 ) +  (1,2,3) d’on podem deduir que
l’equació continua és:
x − 3 y −1 z + 2
r: = =
1 2 3

C/ Pasqual i Batlle, 1-15 08790 Gelida Telèfon: 93 779 04 50 Pàg. 93


e-mail: a8035246@xtec.cat https://agora.xtec.cat/iesgelida
Generalitat de Catalunya
Departament d’Educació
Institut Gelida Departament de Matemàtiques

b) Calculeu l’equació general (és a dir, de la forma Ax + By + Cz + D = 0) del pla


perpendicular a  que passa pels punts P i Q .

Sigui  ' el pla buscat. En la figura adjunta podem


observar que per ser perpendicular a  el vector normal
a  serà un vector director de '.
Per altra banda, si P i Q són punts de ' tindrem que el
vector PQ serà un vector director de  ' .

Per tant, podem calcular ' com el pla que té com a


vectors directors els vector n i PQ i que passa pel punt P .
n = (1,2,3) i PQ = Q − P = ( 0,1,2 ) − ( 3,1, −2 ) = ( −3,0,4 )
Així:
x − 3 1 −3
 ' : y − 1 2 0 = 0 → 8 ( x − 3) − 9 ( y − 1) + 0 ( z + 2 ) + 6 ( z + 2 ) + 0 ( x − 3) − 4 ( y − 1) = 0 →
z+2 3 4

→ 8 ( x − 3) − 13( y − 1) + 6 ( z + 2 ) = 0 → 8x − 24 − 13 y + 13 + 6z + 12 = 0 →
→ 8 x − 13 y + 6 z + 1 = 0
Tornar a l’enunciat

63) PAU LOGSE 2010 Sèrie 5 Qüestió 2:


x−5 y −3 z +3
Donats el punt P = (1, 0, –2) i la recta r: = = :
2 2 −3
a) Trobeu l’equació contínua de la recta que passa pel punt P i talla
perpendicularment la recta r.

• Sigui  el pla perpendicular a r que passa per P. Per


construcció, qualsevol recta continguda en  tallarà
perpendicularment a r.

Sigui Q el punt de tall entre el pla  i la recta r, la recta


que passa per P i Q, estarà continguda en  , per tant,
tallarà perpendicularment a r i evidentment passarà per P
satisfent totes les condicions del problema.

• Càlcul del pla perpendicular a r que passa per P:


vr = ( 2,2 − 3) → n = ( 2,2 − 3) →  : 2 x + 2 y − 3z + k = 0 ⎯⎯⎯⎯⎯
( ) P = 1,0, −2 

→ 2  1 + 2  0 − 3  ( −2 ) + k = 0 → 2 + 0 + 6 + k = 0 → k + 8 = 0 → k = −8 →
 : 2 x + 2 y − 3z − 8 = 0

C/ Pasqual i Batlle, 1-15 08790 Gelida Telèfon: 93 779 04 50 Pàg. 94


e-mail: a8035246@xtec.cat https://agora.xtec.cat/iesgelida
Generalitat de Catalunya
Departament d’Educació
Institut Gelida Departament de Matemàtiques

• Càlcul del punt de tall entre la recta r i el pla  :


x − 5 y − 3 z + 3  x 2−5 =
y −3
→ 2 ( x − 5 ) = 2 ( y − 3) → x − 5 = y − 3 → x − y = 2
r: = = → 2

2 2 −3  2 =
x −5 z +3
−3
→ −3x + 15 = 2 z + 6 → −3x − 2 z = −9 → 3x + 2 z = 9

 : 2 x + 2 y − 3z − 8 = 0  2 2 −3 8  F  F  1 −1 0 2  F → F − 2 F

→ A ' =  1 −1 0 2    2 2 −3 8  
1 2 2 2 1

 x − y = 2
 r : 3 x + 2 z = 9
F → F −3F
 3 0 2 9  3 0 2 9 3 3 1
     

 1 −1 0 2  F →3F  1 −1 0 2  F → F − F  1 −1 0 2 
  0 4 −3 4    0 12 −9 12    0 12 −9 12 
2 2 3 3 2

F →4 F
 0 3 2 3  3 3  0 12 8 12   0 0 17 0 
     

17z = 0 → z = 0
z =0
12 y − 9 z = 12 ⎯⎯→ 12 y = 12 → y = 1
x − y = 2 → x −1 = 2 → x = 3
Q = ( 3,1,0)

• Finalment calculem la recta que passa per P i per Q:


P = (1,0, −2 ) , Q = ( 3,1,0)
PQ = Q − P = ( 3,1,0 ) − (1,0, −2 ) = ( 2,1,2 )
x −1 y z + 2
r ' : ( x, y, z ) = (1,0, −2 ) +  ( 2,1,2 ) → r ' : = =
2 1 2

b) Calculeu la distància del punt P a la recta r.

Si utilitzem la fórmula de la distància entre un punt i una recta tenim:


QP  v
d ( P, r ) = on Q és punt qualsevol de la recta r i v és el vector director de r.
v
Substituint en el nostre cas:
i j k
( 2,1,2 )  ( 2,2, −3) = 2 1 2 = −3i + 4k + 4 j − 2k − 4i + 6 j = −7i + 10 j + 2k = ( −7,10,2 )
2 2 −3

QP  v ( 2,1,2 )  ( 2,2, −3) ( −7,10,2 ) 49 + 100 + 4


d ( P, r ) = = = = =
v ( 2,2, −3) ( 2,2, −3) 4+4+9
153 153
= = = 9= 3
17 17
Tornar a l’enunciat

C/ Pasqual i Batlle, 1-15 08790 Gelida Telèfon: 93 779 04 50 Pàg. 95


e-mail: a8035246@xtec.cat https://agora.xtec.cat/iesgelida
Generalitat de Catalunya
Departament d’Educació
Institut Gelida Departament de Matemàtiques

64) PAU LOGSE 2010 Sèrie 5 Qüestió 5:


Siguin r i s dues rectes d’equacions
y−2 z−a
r : ( x, y, z ) = ( −4,3,4) + t ( 2, −1,1) i s : x + 1 = =
−1 3
a) Trobeu el valor del paràmetre a perquè aquestes rectes es tallin.

Utilitzarem el mètode genèric per comprovar la posició relativa entre dues rectes:
x + 4 y − 3 z − 4  − x − 4 = 2 y − 6 2 = x + 2 y
r : ( x, y, z ) = ( −4,3,4 ) + t ( 2, −1,1) →
= = → →
2 −1 1  y − 3 = −z + 4 y + z = 7
y−2 z−a  − x − 1 = y − 2 1 = x + y
s : x +1 = = → →
−1 3 3 x + 3 = z − a 3 x − z = − a − 3

x + 2 y = 2
y + z = 7


x + y = 1

3 x − z = − a − 3

Restant la 1a i la 3a equació tinc que y = 1 i x =0.


Substituint en la 2a equació tinc que z = 6.
Finalment, substituint en la 4a equació tinc que
3  0 − 6 = −a − 3 → −6 = −a − 3 → a = 3
Per tant, si a = 3 les rectes r i s es tallen i ho fan en el punt ( x, y, z ) = ( 0,1,6)

NOTA: Per demostrar que les rectes r i s es tallen també es pot fer que el sistema de
vectors format pels vectors directors de les dues rectes i un 3r vector que construïm
mitjançant un punt de la recta r i un altre punt de la recta s tingui rang 2.
Seria així:
vr = ( 2, −1,1) i vs = (1, −1,3)
P = ( −4,3,4 )  r i Q = ( −1,2, a )  s
PQ = Q − P = ( −1,2, a ) − ( −4,3,4 ) = ( 3, −1, a − 4 )
v , v , PQ té Rang = 3  u, v , PQ  0 així:
r s

2 1 3
−1 −1 −1 = 0 → −2a + 8 − 9 − 1 + 3 + 6 + a − 4 = 0 → −a + 3 = 0 → a = 3
1 3 a−4
Obtenint, evidentment el mateix resultat que abans.

b) En el cas en què es tallen, trobeu l’equació general (és a dir, de la forma Ax + By +


Cz + D = 0) del pla que les conté.

Per formar un pla necessitem un punt i dos vectors directors.

C/ Pasqual i Batlle, 1-15 08790 Gelida Telèfon: 93 779 04 50 Pàg. 96


e-mail: a8035246@xtec.cat https://agora.xtec.cat/iesgelida
Generalitat de Catalunya
Departament d’Educació
Institut Gelida Departament de Matemàtiques

Agafarem P = ( −4,3,4 )  r , vr = ( 2, −1,1) i vs = (1, −1,3) , aleshores:


x+4 2 1
 : y − 3 −1 −1 = 0 → −3 ( x + 4 ) + ( y − 3) − 2 ( z − 4 ) + ( z − 4 ) + ( x + 4 ) − 6 ( y − 3) = 0 →
z−4 1 3
→ −2 ( x + 4) − 5( y − 3) − ( z − 4 ) = 0 → −2 x − 8 − 5 y + 15 − z + 4 = 0 →
→  : −2 x − 5 y − z + 11 = 0
Tornar a l’enunciat

65) PAU LOGSE 2011 Sèrie 1 Qüestió 1:


2 x − y + 3z = 2
Donada la recta r: :
x + z + 1 = 0
a) Trobeu-ne un vector director.

Hi ha diferents formes de buscar aquest vector director quan la recta ve donada per la
intersecció de dos plans. Potser la forma més comuna de trobar aquest vector és fer el
producte vectorial dels vectors normals de cada pla. Així:
i j k
vr = ( 2, −1,3)  (1,0,1) = 2 −1 3 = −i + 0k + 3 j + k − 0i − 2 j = −i + j + k = ( −1,1,1)
1 0 1
NOTA: Un mètode alternatiu seria resoldre el sistema compatible indeterminat que
formen les dues equacions de la recta per trobar l’equació vectorial de la recta d’on
podríem obtenir el vector director. Seria així:
 2 x − y + 3z = 2  2 −1 3 2   1 0 1 −1  1 0 1 −1
r: → A' =    
x + z +1 = 0  1 0 1 −1  2 −1 3 2   0 −1 1 4 
z =
− y + z = 4 ⎯⎯ ⎯→− y +  = 4 → y =  − 4
2 x − y + 3z = 2 → 2 x −  + 4 + 3 = 2 → 2 x + 2 + 4 = 2 → 2 x = −2 − 2 →
→ x = −2−22 → x = −1 − 

Així: r : ( x, y, z ) = ( −1 −  ,  − 4,  ) = ( −1, −4,0 ) +  ( −1,1,1) → vr = ( −1,1,1)

b) Calculeu l’equació contínua de la recta paral·lela a r que passa pel punt P=(1, 0,
−1).

Sigui s la recta buscada. Per ser paral·lela a r tindrà el mateix vector director que r, per
tant:
vs = vr = ( −1,1,1)
Aleshores, s passa per P = (1,0, −1) amb vector director vs = ( −1,1,1) , aleshores:
x −1 y − 0 z +1
s: = =
−1 1 1
Tornar a l’enunciat

C/ Pasqual i Batlle, 1-15 08790 Gelida Telèfon: 93 779 04 50 Pàg. 97


e-mail: a8035246@xtec.cat https://agora.xtec.cat/iesgelida
Generalitat de Catalunya
Departament d’Educació
Institut Gelida Departament de Matemàtiques

66) PAU LOGSE 2011 Sèrie 1 Qüestió 5:


y − 3 1− z x+3 z +1
Siguin i r1 : x − 2 =
= i r2 : = y +1 = .
2 2 2 2
a) Comproveu que r1 i r2 són perpendiculars.

Dues rectes són perpendiculars sii ho són els seus vectors directors. Dos vectors són
perpendiculars sii el seu producte escalar dóna zero.

y − 3 1− z y − 3 z −1
r1 : x − 2 = = →r:x−2= = → v1 = (1,2, −2 )
2 2 2 −2

x+3 z +1
r2 : = y +1 = → v2 = ( 2,1,2 )
2 2

v1  v2 = (1,2, −2 )  ( 2,1,2 ) = 2 + 2 − 4 = 0 → v1 ⊥ v2 → r1 ⊥ r2

b) Comproveu que es tallen mitjançant la determinació del punt de tall.

y − 3 1 − z  x − 2 = 2 → 2 x − 4 = y − 3 → 2 x − y = 1
y −3
2 x − y = 1
r1 : x − 2 = = →  y −3 → r1 : 
2 2  2 = 1−2 z → y − 3 = 1 − z → y + z = 4 y + z = 4

z + 1  2 = y + 1 → x + 3 = 2 y + 2 → x − 2 y = −1  x − 2 y = −1
x +3
x+3
r2 : = y +1 = → → r : 
 y + 1 = 2 → 2 y + 2 = z + 1 → 2 y − z = −1 2 y − z = −1
z +1 2
2 2

2 x − y = 1 2 x − y = 1
 y + z = 4 F2  F3  x − 2 y = −1
 
  
 x − 2 y = −1 y + z = 4
2 y − z = −1 2 y − z = −1

Sumant les files 3 i 4 tenim 3 y = 3 → y = 1 i z =3


Substituint y en la 2a equació tenim x − 2 1 = −1 → x − 2 = −1 → x = 1

Finalment comprovem si també es compleix la 1a equació:


2 x − y = 2 1 − 1 = 2 − 1 = 1

Així les rectes es tallen en el punt P = (1,1,3)


Tornar a l’enunciat

C/ Pasqual i Batlle, 1-15 08790 Gelida Telèfon: 93 779 04 50 Pàg. 98


e-mail: a8035246@xtec.cat https://agora.xtec.cat/iesgelida
Generalitat de Catalunya
Departament d’Educació
Institut Gelida Departament de Matemàtiques

67) PAU LOGSE 2011 Sèrie 2 Qüestió 2:


2 x − y + 3z = 2 
Donada la recta  , calculeu l’equació general (és a dir, de la forma
x + z +1 = 0 
Ax + By + Cz +D= 0) del pla perpendicular a la recta que passa pel punt P = (1, 0, –1).

Primer trobarem el vector director de la recta.

Quan una recta ve donada per la intersecció de dos plans,


podem trobar el vector director de la recta mitjançant el
producte vectorial dels vectors d’aquests plans. En el nostre
cas:

2 x − y + 3z = 2  n = ( 2, −1,3)
→ → vr = ( 2, −1,3)  (1,0,1) =
x + z + 1 = 0  n ' = (1,0,1)

i j k
= 2 −1 3 = −i + 0k + 3 j + k − 0i − 2 j = −i + j + k = ( −1,1,1) → vr = ( −1,1,1)
1 0 1 Estem

buscant un pla perpendicular a la recta r, aleshores, el vector normal al pla serà el


vector director de la recta r, és a dir, el pla serà de la forma:
( ) P = 1,0, −1
− x + y + z + k = 0 ⎯⎯⎯⎯ → −1 + 0 − 1 + k = 0 → −2 + k = 0 → k = 2

Aleshores el pla buscat és −x + y + z + 2 = 0

NOTA: Una altra manera de trobar el vector director de la recta era passar de la seva
equació general a per exemple l’equació vectorial. Això es fa resolvent el sistema
compatible indeterminat que suposen les dues equacions de la recta. En el nostre cas:

2 x − y + 3z = 2 2 x − y + 3z = 2 z = 2 x − y + 3 = 2
  ⎯⎯⎯ →
x + z + 1 = 0  x + z = −1  x +  = −1 → x = −1 − 

x =−1−
2 x − y + 3 = 2 ⎯⎯⎯ ⎯→ 2  ( −1 −  ) − y + 3 = 2 → 2  ( −1 −  ) + 3 − 2 = y →
→ y = −2 − 2 + 3 − 2 → y = −4 + 

Així: ( x, y, z ) = ( −1 − , −4 + ,  ) → ( x, y, z ) = ( −1, −4,0) +  ( −1,1,1) → vr = ( −1,1,1)


Tornar a l’enunciat

C/ Pasqual i Batlle, 1-15 08790 Gelida Telèfon: 93 779 04 50 Pàg. 99


e-mail: a8035246@xtec.cat https://agora.xtec.cat/iesgelida
Generalitat de Catalunya
Departament d’Educació
Institut Gelida Departament de Matemàtiques

68) PAU LOGSE 2011 Sèrie 2 Qüestió 5:


x −1 y + 2
Considereu la recta r: = = z − a i el pla  : 2 x + y − 5 z = 5 .
3 −1
a) Estudieu la posició relativa de la recta r i el pla  en funció del paràmetre a.

Una recta i un pla poden tenir 3 posicions relatives. Que


la recta talli al pla en un punt, que sigui paral·lela al pla
o que estigui continguda en el pla.
En els dos últims casos (paral·lela i continguda) podem
veure que aleshores el vector director de la recta i el
vector normal al pla han de ser ortogonals, és a dir el seu
producte escalar ha de ser zero.
vr  n = ( 3, −1,1)  ( 2,1, −5) = 6 − 1 − 5 = 0
Per tant, ja sabem que o bé, r és paral·lela a  o bé r està continguda en  .
Per decidir en quin dels dos casos ens trobem, agafem un punt de r i comprovem si
pertany a  .
Sigui P = (1, −2, a )  r .
P = (1, −2, a )
 : 2 x + y − 5z = 5 ⎯⎯⎯⎯ → 2 − 2 − 5a = 5 → −5a = 5 → a = −1
Aleshores:
Si a = −1 la recta r està continguda en el pla  i si a  −1 aleshores la recta r és
paral·lela a  .

NOTA: Evidentment també es pot discutir la posició relativa de la recta i el pla resolvent
el sistema d’equacions lineals que formen i utilitzar el teorema de Rouché Frobenius.
Seria així:

x −1 y + 2  x3−1 = y−+12 → − x + 1 = 3 y + 6 → − x − 3 y = 5 − x − 3 y = 5
r: = = z − a →  y +2 →r:
3 −1  −1 = z − a → y + 2 = − z + a → y + z = a − 2 y + z = a − 2

Aleshores el sistema serà:


− x − 3 y = 5  −1 −3 0 5   −1 −3 0 5  F →1 F
  
F3 → F3 + 2 F1
  3 53
 y + z = a − 2 → A' =  0 1 1 a − 2    0 1 1 a − 2  
2 x + y − 5z = 5  2 1 −5 5   0 −5 −5 15 
   

 −1 −3 0 5 
F3 → F3 + F2
 −1 −3 0 5 
  0 1 1 a − 2    0 1 1 a − 2 
  
 0 −1 −1 3   0 0 0 a +1
  

Aleshores:
Si a = −1 , Rang ( A) = Rang ( A' ) = 2  3 = nº incògnites → S.C.I → r   .
Si a  −1 , Rang ( A) = 2  3 = Rang ( A' ) → S.I → r i  paral·lels.

C/ Pasqual i Batlle, 1-15 08790 Gelida Telèfon: 93 779 04 50 Pàg. 100


e-mail: a8035246@xtec.cat https://agora.xtec.cat/iesgelida
Generalitat de Catalunya
Departament d’Educació
Institut Gelida Departament de Matemàtiques

b) Quan a = 3 , calculeu la distància de la recta r al pla  .

En el cas en que a = 3 sabem que r i  són paral·lels. Aleshores podrem calcular la


distància entre la recta r i el pla  com la distància de qualsevol punt P  r al pla 
.

Quan a = 3 tenim que el punt P = (1, −2,3)  r


Donat un punt P = ( p1 , p2 , p3 ) i un pla  : Ax + By + Cz + D = 0 tenim que:
Ap1 + Bp2 + Cp3 + D
d ( P,  ) =
A2 + B 2 + C 2
2 1 − 2 − 5  3 − 5
Aleshores d ( r, ) = d ( P, ) = d ( (1, −2,3) , 2 x + y − 5z − 5 = 0 ) = =
2 + 1 + ( −5)
2 2 2

2 − 2 − 15 − 5 −20 20 20 30 2 30
= = = = =
4 + 1 + 25 30 30 30 3
Tornar a l’enunciat

69) PAU LOGSE 2011 Sèrie 4 Qüestió 2:


Donat el pla  : 2 x + y − z = 5 :
a) Calculeu l’equació del pla paral·lel al pla  que passa pel punt P=(1, 0, −1).

Sigui ' el pla buscat:


Per ser paral·lel a  l’equació de ' és de la forma  ' : 2x + y − z + k = 0
Com P = (1,0, −1)  ' aleshores:

 ' : 2  1 + 0 − ( −1) + k = 0 → 2 + 0 + 1 + k = 0 → 3 + k = 0 → k = −3 →
→  ' : 2x + y − z − 3 = 0

b) Determineu també la distància entre el punt P i el pla  .

Donat un punt P = ( p1 , p2 , p3 ) i un pla  : Ax + By + Cz + D = 0 tenim que:


Ap1 + Bp2 + Cp3 + D
d ( P,  ) =
A2 + B 2 + C 2

En el nostre cas P = (1,0, −1) i  : 2 x + y − z − 5 = 0


2  1 + 0 − ( −1) − 5 2 + 0 +1− 5 −2 2 2 6 6
d ( P,  ) = = = = = =
22 + 12 + ( −1)
2
4 +1+1 6 6 6 3

C/ Pasqual i Batlle, 1-15 08790 Gelida Telèfon: 93 779 04 50 Pàg. 101


e-mail: a8035246@xtec.cat https://agora.xtec.cat/iesgelida
Generalitat de Catalunya
Departament d’Educació
Institut Gelida Departament de Matemàtiques

NOTA: Donat que el punt P pertany al pla  ' també


podem calcular la distància entre el punt P i el pla  com
la distància entre els plans paral·lels  i ' . És a dir:
d ( P, ' ) = d ( , ' ) =
= d ( : 2 x + y − z − 5 = 0,  ' :2 x + y − z − 3 = 0) =
−5 − ( −3) −5 + 3 −2 2 6
= = = = =
22 + 12 + ( −1)
2
4 +1+1 6 6 3
Tornar a l’enunciat

70) PAU LOGSE 2011 Sèrie 4 Qüestió 5:


Calculeu l’equació general (és a dir, de la forma Ax+By+Cz+D=0) dels plans que
y = 2
contenen la recta r: i que formen un angle de 45° amb el pla z = 0 .
z = 1

MÈTODE 1: Imaginant els plans solució:

y = 2
En el gràfic podem observar amb color blau la recta r: . Per dibuixar-la
z = 1
solament cal representar dos punts que compleixin les dues equacions, per exemple,
els punts A = ( 0,2,1) i B = (1,2,1) i després unir-los.
També he intentat representar, amb poc èxit, els dos plans  i  ' que contenen a la
recta r i que formen un angle de 45º amb el pla horitzontal z = 0 representat en color
groc.

• Podem observar que el pla  , a banda de contenir a la recta r, passa per exemple
pel punt C = ( 0,3,0) mentre que el pla ' conté a la recta r i passa pel punt

D = ( 0,1,0) , per tant, ja tenim suficient informació per trobar les seves equacions.

Restant els dos punts A i B de r obtindrem el seu vector director:


AB = B − A = (1,2,1) − ( 0,2,1) = (1,0,0 ) → vr = (1,0,0 )
C  i A , aleshores CA = A − C = ( 0,2,1) − ( 0,3,0 ) = ( 0, −1,1)
• Aleshores podem construir el pla  com el pla que passa pel punt A = ( 0,2,1) i té

com a vectors directors vr = (1,0,0) i CA = ( 0, −1,1) , així:


C/ Pasqual i Batlle, 1-15 08790 Gelida Telèfon: 93 779 04 50 Pàg. 102
e-mail: a8035246@xtec.cat https://agora.xtec.cat/iesgelida
Generalitat de Catalunya
Departament d’Educació
Institut Gelida Departament de Matemàtiques

x 1 0
 : y − 2 0 −1 = 0 → −1( z − 1) − 1( y − 2 ) = 0 → − z + 1 − y + 2 = 0 →  : y + z − 3 = 0
z −1 0 1

• Anàlogament D = ( 0,1,0)  ' i A ' , restant-los obtindrem un vector director de

 ' : DA = A − D = ( 0,2,1) − ( 0,1,0) = ( 0,1,1)


Finalment podem construir el pla ' com el pla que passa pel punt A = ( 0,2,1) i té

com a vectors directors vr = (1,0,0) i DA = ( 0,1,1) , així:


x 1 0
 ' : y − 2 0 1 = 0 → 1( z − 1) − 1( y − 2 ) = 0 → z − 1 − y + 2 = 0 →  : − y + z + 1 = 0
z −1 0 1

MÈTODE 2:
y = 2
La recta r: conté tots els punts en que la coordenada y val 2 i la
z = 1
coordenada z val 1. Per tant, passa pels punts A = ( 0,2,1) i A' = (1,2,1) .
Restant-los obtenim que el vector director de r és:
vr = AA ' = A '− A = (1,2,1) − ( 0,2,1) = (1,0,0 ) .
Si el pla  que busquem conté la recta r , aleshores el vector normal a  i el vector
director de r seran ortogonals. És a dir, si  : Ax + By + Cz + D = 0 tindrem que
vr ⊥ n → (1,0,0)  ( A, B, C ) = 0 → A = 0 .
Així, donat que A = 0, el pla  serà de la forma  : By + Cz + D = 0 i per tant el seu
vector normal serà de la forma n = ( 0, B, C ) .
Finalment utilitzarem el fet de que el pla  que estem buscant forma un angle de 45º
amb el pla  z : z = 0 que té com a vector normal nz = ( 0,0,1)

Per definició de producte escalar:


n  nz ( 0, B, C )  ( 0,0,1)
( )
cos n , nz =
n  nz
→ cos 45º =
( 0, B, C )  ( 0,0,1)

2
2
=
C
B2 + C 2  1

2 2
2 C  2  C  2 C2 1 C2
→ = ⎯⎯→ 
x2
 =  → = 2 → = 2 →
2 B2 + C 2  2   B +C 
2 2 4 B + C2 2 B + C2

→ B 2 + C 2 = 2C 2 → B 2 = C 2 → C =  B 2 → C =  B

Per tant, tenim dues solucions C = B i C = −B .


Així si, per exemple fem B = 1 tenim:
( 0,2,1)
 : By + Cz + D = 0 ⎯⎯
B =1
⎯ C=B
→ y + Cz + D = 0 ⎯⎯⎯ → y + z + D = 0 ⎯⎯⎯⎯ →

C/ Pasqual i Batlle, 1-15 08790 Gelida Telèfon: 93 779 04 50 Pàg. 103


e-mail: a8035246@xtec.cat https://agora.xtec.cat/iesgelida
Generalitat de Catalunya
Departament d’Educació
Institut Gelida Departament de Matemàtiques

→ 2 + 1 + D = 0 → D = −3 →  : y + z − 3 = 0
I en el cas en que C = −B tenim:
( 0,2,1) '
 ' : By + Cz + D = 0 ⎯⎯⎯ B =1 C =− B
→ y + Cz + D = 0 ⎯⎯⎯ → y − z + D = 0 ⎯⎯⎯⎯ →

→ 2 − 1 + D = 0 → D = −1 →  ' : y − z − 1 = 0

MÈTODE 3: Per feixos de plans:


y = 2 y − 2 = 0
Donada la recta r:  sabem que el feix de plans que la conté és
z = 1 z − 1 = 0
  : ( y − 2) +  ( z − 1) = 0 →   : y − 2 +  z −  = 0 →   : y +  z − 2 −  = 0 i per tant,

el seu vector normal serà n = ( 0,1,  ) .

Si el pla  i el pla z : z = 0 han de formar un angle de 45º, aleshores els seus vectors

normals n = ( 0,1,  ) i nz = ( 0,0,1) també formaran el mateix angle.

Per definició de producte escalar:


2 2
n  nz = n nz cos 45º → ( 0,1,  )  ( 0,0,1) = 12 +  2  12  →  = 1+ 2  →
2 2
→ 2  = 1 +  2  2 ⎯⎯ → 4 2 = (1 +  2 )  2 ⎯⎯
2
→ 2 2 = 1 +  2 →  2 = 1 →
2
x

→  = 1 →  = 1
 Si  = 1 → y + z − 3 = 0

 : y + z − 2 −  = 0 → 
 Si  = −1 → y − z − 1 = 0

Tornar a l’enunciat

71) PAU LOGSE 2012 Sèrie 1 Qüestió 1:


Donats els plans  1 : 3x + y − 2 z + 15 = 0 i  2 : x + y + 2 z − 103 = 0 ,
a) Comproveu que són perpendiculars.

Dos plans seran perpendiculars sii ho són els seus vectors normals. Així
 1 ⊥  2  n ⊥ n
1 2
.

Dos vectors són perpendiculars sii el seu producte escalar és zero. Així:
1 : 3x + y − 2z + 15 = 0 → n = (3,1, −2) 1

 2 : x + y + 2 z − 103 = 0 → n = (1,1,2 ) 2

n1  n 2 = ( 3,1, −2)  (1,1,2 ) = 3 + 1 − 4 = 0 → 1 ⊥  2

C/ Pasqual i Batlle, 1-15 08790 Gelida Telèfon: 93 779 04 50 Pàg. 104


e-mail: a8035246@xtec.cat https://agora.xtec.cat/iesgelida
Generalitat de Catalunya
Departament d’Educació
Institut Gelida Departament de Matemàtiques

b) Calculeu l’equació cartesiana (és a dir, de la forma Ax+By+Cz+D=0) del pla


perpendicular a  1 i  2 , que passa pel punt P=(1, 3, 2).

Sigui 3 el pla buscat. Per ser perpendicular als plans 1 i  2 tenim que el vector
normal al nostre pla 3 també serà perpendicular als vectors normals a 1 i  2 . És a
dir, per buscar el vector normal a 3 solament necessitem buscar un vector ortogonal
a n 1 i a n 2 .
Això ho podem fer mitjançant el producte vectorial d’aquests dos vectors. És a dir:
i j k
n 3 = n1  n 2 = 3 1 −2 = 2i + 3k − 2 j − k + 2i − 6 j = 4i − 8 j + 2k = ( 4, −8,2 )
1 1 2
Dividint les components del vector entre dos podem agafar el vector n 3 = ( 2, − 4, 1)
Per tant, l’equació del pla 3 és de la forma  3 : 2x − 4 y + z + k = 0

Imposant que passi pel punt P=(1, 3, 2) tenim:


P =(1,3,2) 3
 3 : 2 x − 4 y + z + k = 0 ⎯⎯⎯⎯⎯
→ 2  1 − 4  3 + 2 + k = 0 → 2 − 12 + 2 + k = 0 →
→ −8 + k = 0 → k = 8 →  3 : 2 x − 4 y + z + 8 = 0
Tornar a l’enunciat

72) PAU LOGSE 2012 Sèrie 1 Qüestió 6:

3 x + y = 1
Siguin  : x − 3y + 2z = 1 i r :  . Estudieu-ne la posició relativa segons
2 x − y + mz = 1
el valor del paràmetre m.

3
Una recta i un pla de admeten 3 posicions relatives diferents. La recta talla al pla
en un punt, la recta és paral·lela al pla o la recta està continguda en el pla. D’entrada
podem resoldre l’exercici de tres formes diferents.

• MÈTODE 1: Amb el teorema de Rouché-Frobenius actuant per determinants:


Donat que la matriu de coeficients A és una matriu quadrada podem aprofitar per
calcular el seu determinant i estudiar els valors que l’anul·lin. És a dir:
 1 −3 2 
A =  3 1 0  = m − 6 + 0 − 4 + 0 + 9m = 10m − 10
 2 −1 m 
 
A = 0 → 10m − 10 = 0 → 10m = 10 → m = 1 . Per tant, hem d’estudiar els casos m = 1 i
m 1.

Cas m = 1 , en aquest cas:

C/ Pasqual i Batlle, 1-15 08790 Gelida Telèfon: 93 779 04 50 Pàg. 105


e-mail: a8035246@xtec.cat https://agora.xtec.cat/iesgelida
Generalitat de Catalunya
Departament d’Educació
Institut Gelida Departament de Matemàtiques

 1 −3 2 1 F → F −3F  1 −3 2 1  F2 → 1 F2  1 −3 2 1  F → F − F
A ' =  3 1 0 1   0 10 −6 −2    0 5 −3 −1 
2 2 1 2 3 3 2

F →F −2 F
 2 −1 1 1 3 3 1  0 5 −3 −1   0 5 −3 −1
     

 1 −3 2 1 
  0 5 −3 −1
0 0 0 0 
 

Aleshores:
• Si m = 1 , Rang ( A) = Rang ( A' ) = 2  3 = nº incògnites → SC.I . →  solucions → La
recta està continguda en el pla.
• Si m  1 A  0 → Rang ( A) = Rang ( A' ) = 3 = nº incògnites → SC.D. →
solució única → La recta i el pla es tallen en un punt.

• MÈTODE 2: Amb el teorema de Rouché-Frobenius esgraonant la matriu pel mètode


de Gauss:

 : x − 3 y + 2 z = 1  1 −3 2 1 F → F −3F  1 −3 2 1  F →1 F
    −6 −2  
2 2 1 2 2 2

 3 x + y = 1 → A ' =  3 1 0 1   0 10
 r : 2 x − y + mz = 1
F →F −2 F
 2 −1 m 1 3 3 1  0 5 m − 4 −1 
     

 1 −3 2 1
F3 → F3 − F2
 1 −3 2 1

 0 5 
−3 −1   0 5  −3 −1 → m − 1 = 0 → m = 1
 0 5 m − 4 −1 0 0 m −1 0 
   

• Si m = 1 Rang ( A) = Rang ( A' ) = 2  3 = nº incògnites → SC.I . →  solucions → La


recta està continguda en el pla.

• Si m  1 Rang ( A) = Rang ( A' ) = 3 = nº incògnites → SC.D. → solució única → La


recta i el pla es tallen en un punt.

• MÈTODE 3: Podríem anomenar-lo mètode geomètric.


En la figura adjunta podem observar que si la recta r és
paral·lela o està continguda en el pla  aleshores el
vector normal al pla  , n i el vector director de la recta
r, vr seran perpendiculars.

Càlcul del vector normal a  :


 : x − 3 y + 2 z = 1 → n = (1, −3,2 )
Càlcul del vector director de la recta r:

C/ Pasqual i Batlle, 1-15 08790 Gelida Telèfon: 93 779 04 50 Pàg. 106


e-mail: a8035246@xtec.cat https://agora.xtec.cat/iesgelida
Generalitat de Catalunya
Departament d’Educació
Institut Gelida Departament de Matemàtiques

i j k
3 x + y = 1
r: → vr = ( 3,1,0 )  ( 2, −1, m ) = 3 1 0 =
2 x − y + mz = 1
2 −1 m
= mi − 3k + 0 j − 2k + 0i − 3mj = mi − 3mj − 5k = ( m, −3m, −5) → vr = ( m, −3m, −5)

Estudiem ara, quan els vectors n i vr són perpendiculars:


n  vr = (1, −3,2)  ( m, −3m, −5) = m + 9m − 10 = 10m − 10 = 0 → m = 1 .

Per tant, en el cas en que m 1 tindrem que els vectors n i vr no seran


perpendiculars i per tant la recta i el pla es tallaran en un punt.

Queda per estudiar el cas m = 1 . En aquest cas tenim dues possibilitats, que la recta r
sigui paral·lela al pla o que la recta estigui continguda en el pla. Per decantar-nos per
una o l’altra possibilitat agafarem un punt de la recta i comprovarem si pertany o no al
pla.
3 x + y = 1 m =1 3 x + y = 1
r: ⎯⎯⎯ →r : 
2 x − y + mz = 1 2 x − y + z = 1

Per trobar un punt fem per exemple x = 0 , aleshores, de la 1a equació tenim que
y =1 que substituint en la 2a equació ens dóna que
2  0 −1 + z = 1 → 0 −1 + z = 1 → z = 2 .

Per tant, el punt P = ( 0,1,2 ) és un punt de la recta r. Comprovem ara si P  


substituint en l’equació de  .
P = ( 0,1,2 )
 : x − 3 y + 2 z = 1 ⎯⎯⎯⎯ → 0 − 3 1 + 2  2 = 0 − 3 + 4 = 1 → P  → r  

Per tant, amb aquest tercer mètode, com era evident, hem arribat a la mateixa
conclusió que amb els altres dos, que és que:
Si m  1 la recta i el pla es tallen en un punt.
Si m = 1 la recta està continguda en el pla.
Tornar a l’enunciat

73) PAU LOGSE 2012 Sèrie 3 Qüestió 1:


Digueu per a quin valor del paràmetre m els plans 1 : x − y + mz = 1 ,  2 : x − y + z = m
i  3 : my + 2 z = 3 tenen com a intersecció una recta.

Estudiem el sistema que formen els 3 plans mitjançant el teorema de Rouché-Frobenius:


 x − y + mz = 1  1 −1 m 1 
  
 x − y + z = m → A ' =  1 −1 1 m 
my + 2 z = 3 0 m 2 3 
  

C/ Pasqual i Batlle, 1-15 08790 Gelida Telèfon: 93 779 04 50 Pàg. 107


e-mail: a8035246@xtec.cat https://agora.xtec.cat/iesgelida
Generalitat de Catalunya
Departament d’Educació
Institut Gelida Departament de Matemàtiques

MÈTODE 1: Donat que la matriu de coeficients A és quadrada el mètode més segur és


fer el seu determinant. Així:

1 −1 1
A = 1 −1 m = −3 + m + 0 − 0 − m2 + 3 = −m 2 + m = 0 → m  ( −m + 1) = 0 →
0 m 3
m = 0
→
 −m + 1 = 0 → m = 1

• Cas m = 0:
 1 −1 0 1   1 −1 0 1   1 −1 0 1 
A ' =  1 −1 1 0    0 0 1 −1   0 0 1 −1 →
 0 0 2 3  0 0 2 3   0 0 0 5 
     
→ Rang ( A) = 2  3 = Rang ( A' ) → S.I .

En aquest cas els tres plans no tenen cap punt en comú. Si ens ho demanessin caldria
estudiar els plans dos a dos per acabar de decidir la seva posició relativa.

• Cas m = 1:
 1 −1 1 1   1 −1 1 1 
 1 −1 1 1 
A ' =  1 −1 1 1    0 0 0 0    →
 0 1 2 3  0 1 2 3   0 1 2 3
   

→ Rang ( A) = 2 = Rang ( A' )  3 = nº incògnites → S.C.I .

En aquest cas els tres plans es tallen en una recta, per tant la resposta al problema és
m =1

• En el cas en quem  1 i m  0 tindríem:


A  0 → Rang ( A) = Rang ( A' ) = 3 = nº incògnites → S.C.D. → Els tres plans es tallen
en un punt.

MÈTODE 2: Esgraonant la matriu mitjançant el mètode de Gauss.

 1 −1 m 1  F → F − F  1 −1 m 1 
F2  F1
 1 −1 m 1 
  2 2 1
 
A ' =  1 −1 1 m    0 0 1 − m m − 1   0 m  2 3 
0 m 2 3  0 m 3   0 0 1 − m m − 1
   2  

Una vegada esgraonada la matriu, els valors que anul·larien el determinant de A són
els que estan en la diagonal de A, per tant, cal estudiar els valors que fan que
1  m  (1 − m ) = 0 i evidentment obtenim els mateixos valors que abans.
Tornar a l’enunciat

C/ Pasqual i Batlle, 1-15 08790 Gelida Telèfon: 93 779 04 50 Pàg. 108


e-mail: a8035246@xtec.cat https://agora.xtec.cat/iesgelida
Generalitat de Catalunya
Departament d’Educació
Institut Gelida Departament de Matemàtiques

74) PAU LOGSE 2012 Sèrie 3 Qüestió 3:


x + y + z = 0
Donats el pla  : x − y + 2 z − 5 = 0 i la recta r : 
2 x − y + z = 10
a) Calculeu el punt d’intersecció entre el pla i la recta.

 x − y + 2z − 5 = 0  x − y + 2z = 5  1 −1 2 5  F → F − F  1 −1 2 5 
 
  x + y + z = 0 → A ' =  1 1 1 0    0 2 −1 −5  
2 2 1

x + y + z = 0 F →F −2 F
2 x − y + z = 10 2 x − y + z = 10  2 −1 1 10  3 3 1  0 1 −3 0 
     

 1 −1 2 5  F → F − 2 F  1 −1 2 5 
F2  F3
  0 1 −3 0    0 1 −3 0 
3 3 2

 0 2 −1 −5   0 0 5 −5 
   

5z = −5 → z = −1
y − 3z = 0 → y − 3( −1) = 0 → y + 3 = 0 → y = −3
x − y + 2z = 5 → x + 3 − 2 = 5 → x + 1 = 5 → x = 4

Així el punt de tall entre el pla i la recta és P = ( 4, −3, −1) .

b) Trobeu l’equació contínua de la recta s continguda en el pla  , que és


perpendicular a la recta r i talla la recta r.

MÈTODE 1:
Quan donem una recta
mitjançant la seva
equació general en
realitat l’estem donant
com a intersecció de dos
plans.
Intentarem donar la recta
s que ens demanen com a
intersecció de dos plans.
Com la recta s està continguda en el pla  , un dels dos plans que la determinarà serà
 : x − y + 2 z − 5 = 0
precisament el pla  , per tant, s serà de la forma: s: .
 ' :
Per tant, el que ens queda es calcular el pla  ' .
Sabem que s ha de ser perpendicular a la recta r, aleshores si fem el pla perpendicular
a r que passa pel punt d’intersecció P tindrem precisament el pla  ' que necessitem.
Així:  ' ⊥ r → n ' = vr
i j k
x + y + z = 0
r: → vr = 1 1 1 = i − k + 2 j − 2k + i − j = 2i + j − 3k → vr = ( 2,1, −3)
2 x − y + z = 10
2 −1 1

C/ Pasqual i Batlle, 1-15 08790 Gelida Telèfon: 93 779 04 50 Pàg. 109


e-mail: a8035246@xtec.cat https://agora.xtec.cat/iesgelida
Generalitat de Catalunya
Departament d’Educació
Institut Gelida Departament de Matemàtiques

Com n ' = vr = ( 2,1, −3) aleshores:

→ 2  4 − 3 − 3  ( −1) + k = 0 → 8 −3 + 3 + k = 0 →
P = ( 4, −3, −1) '
 ' : 2 x + y − 3z + k = 0 ⎯⎯⎯⎯⎯
→ k = −8 →  ' : 2 x + y − 3z − 8 = 0 i per tant la recta buscada és:

 : x − y + 2 z − 5 = 0
s:
 ' : 2 x + y − 3z − 8 = 0

MÈTODE 2: La recta s està continguda en el pla  , per tant, el vector director de s serà
perpendicular al vector normal al pla  . És a dir, vs ⊥ n = (1, −1,2 ) .
Però també ens diuen que s ha de ser perpendicular a la recta r, per tant el vector
director de s també ha de ser perpendicular al vector director de la recta r que ja hem
calculat abans que és
vr = ( 2,1, −3) .
Donat que el vector director vs ha de ser perpendicular als vectors n = (1, −1,2 ) i
vr = ( 2,1, −3) podem calcular el vector vs com el producte vectorial d’aquests dos, és
a dir:
i j k
vs = 1 −1 2 = 3i + k + 4 j + 2k − 2i + 3 j = i + 7 j + 3k → vs = (1,7,3)
2 1 −3
Donat que la recta s passa pel punt P = ( 4, −3, −1) tenim que:

x − 4 y + 3 z +1
s: = =
1 7 3
Tornar a l’enunciat

75) PAU LOGSE 2012 Sèrie 3 Qüestió 6:


Donats els punts P = (1, 0, 0), Q= (0, 2, 0), R = (0, 0, 3) i S = (1, 2, 3),
a) Calculeu l’equació cartesiana (és a dir, de la forma Ax+By+Cz+D=0) del pla que
conté els punts P, Q i R.

Per formar el pla necessitem un punt i dos vectors directors.


PQ = Q − P = ( 0,2,0 ) − (1,0,0 ) = ( −1,2,0 )
PR = R − P = ( 0,0,3) − (1,0,0 ) = ( −1,0,3)
El pla que passa pel punt P = (1,0,0) i té com a vectors directors PQ = ( −1,2,0 ) i
PR = ( −1,0,3) és:
x − 1 −1 −1
: y 2 0 = 0 → 6 ( x − 1) + 2 z + 3 y = 0 → 6 x − 6 + 2 z + 3 y = 0 →
z 0 3
→  : 6x + 3y + 2z − 6 = 0

C/ Pasqual i Batlle, 1-15 08790 Gelida Telèfon: 93 779 04 50 Pàg. 110


e-mail: a8035246@xtec.cat https://agora.xtec.cat/iesgelida
Generalitat de Catalunya
Departament d’Educació
Institut Gelida Departament de Matemàtiques

b) Comproveu si els quatre punts són coplanaris (és a dir, si els quatre estan continguts
en un mateix pla).

Per comprovar-ho, solament caldrà comprovar si el punt D satisfà l’equació del pla
que passa pels altres tres punts:
S = (1,2,3)
 : 6 x + 3 y + 2 z − 6 = 0 ⎯⎯⎯⎯ → 6  1 + 3  2 + 2  3 − 6 = 6 + 6 + 6 − 6 = 12  0 → S   →
Els quatre punts no són coplanaris.
Tornar a l’enunciat

76) PAU 2012 Sèrie 3 Qüestió 6:


Considereu les rectes de l’espai següents:
x +1 z −1 x − 4 y −1 z − 2
r: = y −1 = , s: = =
2 −1 3 −1 2
a) Comproveu que són secants.

Per comprovar que efectivament les dues rectes es tallen en un punt hi ha dos
mètodes, resoldre el sistema que formen o bé demostrar que si agafem un punt P de r,
un punt Q de s i construïm el vector PQ , aleshores aquest vector, el vector director de
r i el vector director de s són coplanaris.

MÈTODE 1: Resolució del sistema que formen les dues rectes.


z − 1  2 = y − 1 → x + 1 = 2 y − 2 → x − 2 y = −3
x +1
x +1
r: = y −1 = →
2 −1  y − 1 = −1 → − y + 1 = z − 1 → 2 = z + y → y + z = 2
z −1

x − 4 y −1 z − 2 
y −1
 3 = −1 → − x + 4 = 3 y − 3 → 7 = 3 y + x → x + 3 y = 7
x −4
s: = = →  y −1
3 −1 2  −1 = 2 → 2 y − 2 = − z + 2 → 2 y + z = 4
 z −2

 x − 2 y = −3   x − 2 y = −3 Restant
y + z = 2  ⎯⎯⎯→ 5 y = 10 → y = 2 x = 1
  x + 3 y = 7
 →
x + 3y = 7   y + z = 2 ⎯⎯⎯
Restant
→y=2 z=0
 2 y + z = 4  2 y + z = 4

Així, les dues rectes es tallen en el punt (1,2,0 )

MÈTODE 2: Agafant un punt P de r, un punt Q de s, construint el vector PQ i


demostrant que PQ , el vector director de r i el vector director de s són coplanaris.
x +1  P = ( −1,1,1)  r
z −1 
r: = y −1 = →
2 −1 vr = ( 2,1, −1)

Q = ( 4,1,2 )  s
x − 4 y −1 z − 2 
s: = = →
3 −1 2 vs = ( 3, −1,2 )

PQ = Q − P = ( 4,1,2 ) − ( −1,1,1) = (5,0,1)
C/ Pasqual i Batlle, 1-15 08790 Gelida Telèfon: 93 779 04 50 Pàg. 111
e-mail: a8035246@xtec.cat https://agora.xtec.cat/iesgelida
Generalitat de Catalunya
Departament d’Educació
Institut Gelida Departament de Matemàtiques

5 2 3
PQ, vr , vs = 0 1 −1 = 10 + 0 − 2 − 3 − 5 − 0 = 0 → r i s es tallen en un punt
1 −1 2

b) Calculeu l’equació contínua de la recta que les talla i que és perpendicular a totes
dues.

Sigui t la recta que busquem. Com t talla a les dues rectes r i s , aleshores t passa
pel punt de tall de r i s , és a dir, A = (1,2,0)  t .
Com t és perpendicular a r i a s , aleshores el vector director de t , vt serà
perpendicular a vr i vs .
i j k
vt = vr  vs = 2 1 −1 = 2i − 2k − 3 j − 3k − i − 4 j = i − 7 j − 5k = (1, −7, −5 )
3 −1 2
t : ( x, y, z ) = (1,2,0) +  (1, −7, −5) que si passem a la seva equació continua tenim:

x −1 y − 2 z
t: = =
1 −7 −5
Tornar a l’enunciat

77) PAU 2013 Sèrie 3 Qüestió 1:


Sigui  : 3 x − 2 y + z = 10 .
a) Trobeu l’equació continua de la recta r perpendicular a  que passa pel punt
P = ( −1,3,2) .

x − ( −1) y −3 z −2
r ⊥  → vr = n → vr = ( 3, −2,1) → r : = = →
3 −2 1
x +1 y − 3 z − 2
→ r: = =
3 −2 1

b) Trobeu també l’equació cartesiana (és a dir, de la forma Ax + By + Cz + D = 0 del


pla 1 paral·lel a  que passa pel mateix punt P.

 tenen la forma 1 : 3x − 2 y + z + D = 0
Els plans paral·lels a

P = ( −1,3,2)  1 → 1 : 3  ( −1) − 2  3 + 2 + D = 0 → −3 − 6 + 2 + D = 0 → −7 + D = 0 →
→ D = 7 → 1 : 3x − 2 y + z + 7 = 0
Tornar a l’enunciat

C/ Pasqual i Batlle, 1-15 08790 Gelida Telèfon: 93 779 04 50 Pàg. 112


e-mail: a8035246@xtec.cat https://agora.xtec.cat/iesgelida
Generalitat de Catalunya
Departament d’Educació
Institut Gelida Departament de Matemàtiques

78) PAU 2013 Sèrie 3 Qüestió 4:


Un triangle d’àrea 3/ 2 té dos dels vèrtexs als punts P = ( 0,0,0) i Q = ( 2,0,1) . El tercer
x + y + z = 0
vèrtex, R , és un punt de la recta r :  i té la primera coordenada no
y =1
nul·la. Calculeu les coordenades del vèrtex R .

Primer calcularem un punt genèric de la recta


r . Per a aconseguir-ho hem d’expressar la
recta com a suma d’un punt més el múltiple
d’un vector.

 x + y + z = 0  x + y = − z z := 
r: → ⎯⎯⎯ →
y =1 y =1
 x + 1 = −  x = −1 − 
→ → →
y =1 y =1

→ ( x, y, z ) = ( −1 − , 1,  )
Així R = ( −1 − , 1,  )
Recordant que el mòdul del producte vectorial de dos vectors és l’àrea del
paral·lelogram que formen i que l’àrea del triangle PQR serà la meitat de l’àrea del
paral·lelogram de costats PQ i PR tenim:

PQ = Q − P = ( 2,0,1) − ( 0,0,0 ) = ( 2,0,1)


PR = R − P = ( −1 −  , 1,  ) − ( 0,0,0 ) = ( −1 −  , 1,  )
i j k
PQ  PR = ( 2,0,1)  ( −1 −  , 1,  ) = 2 0 1 = 2k + ( −1 −  ) j − i − 2 j =
−1 −  1 

= −i + ( −1 − 3 ) j + 2k = ( −1, − 1 − 3 , 2 ) →

→ PQ  PR = ( −1, − 1 − 3 , 2 ) = ( −1) + ( −1 − 3 ) + 22 = 1 + 1 + 6 + 9 2 + 4 =
2 2

= 9 2 + 6 + 6

3
Àrea triangle PQR = → Àrea paral·lelogram de costats PQ i PR = 3 →
2
→ PQ  PR = 3 → 9 2 + 6 + 6 = 3 → 9 2 + 6 + 6 = 9 → 9 2 + 6 − 3 = 0 ⎯⎯
3

C/ Pasqual i Batlle, 1-15 08790 Gelida Telèfon: 93 779 04 50 Pàg. 113


e-mail: a8035246@xtec.cat https://agora.xtec.cat/iesgelida
Generalitat de Catalunya
Departament d’Educació
Institut Gelida Departament de Matemàtiques

 = 13
→ 3 + 2 − 1 = 0 → 
2

 = −1
 ⎯⎯⎯ → ( −34 ,1, 13 )
 = 13

R = ( −1 −  , 1,  ) → 
 =−1
 ⎯⎯⎯ → ( 0,1, −1)
Com l’enunciat diu que la primera coordenada del punt R és no nul·la, aleshores hem
de descartar la segona solució i quedar-nos amb R = ( −34 ,1, 13 )
Tornar a l’enunciat

79) PAU 2013 Sèrie 4 Qüestió 5:


Donats els punts P = (1,0, −1) i Q = ( −1,2,3) , trobeu un punt R de la recta
x+3 y +4 z −3
r: = = que compleixi que el triangle de vèrtexs P , Q i R és isòsceles,
2 3 −1
en què PR i QR són els costats iguals del triangle.

x+3 y +4 z −3
r: = = →
2 3 −1
→ ( x, y, z ) = ( −3, −4,3) +  ( 2,3, −1)

Un punt genèric de la recta seria:


R = ( −3 + 2, −4 + 3,3 −  )

PR = R − P = ( −3 + 2 , −4 + 3 ,3 −  ) − (1,0, −1) = ( −4 + 2 , −4 + 3,4 −  )


QR = R − Q = ( −3 + 2 , −4 + 3 ,3 −  ) − ( −1,2,3) = ( −2 + 2 , −6 + 3 , − )

PR = QR → ( −4 + 2 , −4 + 3 ,4 −  ) = ( −2 + 2 , −6 + 3 , − ) →

→ ( −4 + 2 ) + ( −4 + 3 ) + (4 −  ) = ( −2 + 2 ) + ( −6 + 3 ) + ( − ) ⎯⎯ →
2 2 2 2 2 2 2
x

→ ( −4 + 2 ) + ( −4 + 3 ) + ( 4 −  ) = ( −2 + 2 ) + ( −6 + 3 ) + ( − ) →
2 2 2 2 2 2

→ 16 − 16 + 4 2 + 16 − 24 + 9 2 + 16 − 8 +  2 = 4 − 8 + 4 2 + 36 − 36 + 9 2 +  2 →

→ −48 + 48 = −44 + 40 → 48 − 40 = 48 − 44 → 8 = 4 →  = 2

Per tant: R = ( −3 + 2 , −4 + 3 ,3 −  ) ⎯⎯⎯


 =2
→ R = (1,2,1)
Tornar a l’enunciat
C/ Pasqual i Batlle, 1-15 08790 Gelida Telèfon: 93 779 04 50 Pàg. 114
e-mail: a8035246@xtec.cat https://agora.xtec.cat/iesgelida
Generalitat de Catalunya
Departament d’Educació
Institut Gelida Departament de Matemàtiques

80) PAU 2013 Sèrie 5 Qüestió 1:


Siguin  1 el pla 2 x + 3 y − z = 4 i  2 el pla x − 2 y − 4 z = 10 .
a) Comproveu que els plans 1 i  2 són perpendiculars.
b) Trobeu l’equació continua de la recta paral·lela als plans 1 i  2 i que passa pel

que P = ( −1,3,2) .
Els plans 1 i  2 seran perpendiculars si ho són els seus vectors normals, és a dir, si el
producte escalar dels seus vectors normals és zero.
1 : 2 x + 3 y − z = 4 → n1 = ( 2,3, −1)
 2 : x − 2 y − 4 z = 10 → n2 = (1, −2, −4)
n1  n2 = ( 2,3, −1)  (1, −2, −4) = 2 − 6 + 4 = 0 → 1 ⊥  2

Sigui r la recta buscada. Si r és paral·lela als plans 1 i  2 , aleshores el vector


director de r serà perpendicular als vectors normals dels dos plans, és a dir:
i j k
vr = n1  n2 = ( 2,3, −1)  (1, −2, −4 ) = 2 3 −1 = −12i − 4k − j − 3k − 2i + 8 j =
1 −2 −4
= −14i + 7 j − 7k = ( −14,7, −7 )
Per tant, vr té la mateixa direcció que el vector ( −14,7, −7 ) que dividint-lo entre −7
tenim:
vr = ( 2, −1,1)
Aleshores r és la recta que passa pel punt P = ( −1,3,2) i té com a vector director
vr = ( 2, −1,1) i per tant la seva equació continua serà:

x +1 y − 3 z − 2
r: = =
2 −1 1
Tornar a l’enunciat

81) PAU 2013 Sèrie 5 Qüestió 3:


Donats els punts P = (1, −1,2) , Q = ( 2,0,1) i R = ( 3,2, −1) ,
a) Trobeu l’equació cartesiana (és a dir, de la forma Ax + By + Cz + D = 0 ) del pla
que determinen.
x − 5 y −1 z − 5
b) Trobeu un punt S pertanyent a la recta r: = = , de manera que el
2 −1 −3
tetraedre de vèrtexs P , Q , R i S tinguin un volum igual a 1/ 2 .

A partir d’aquests tres punts trobarem un punt i dos vectors.


P = (1, −1,2) , Q = ( 2,0,1) i R = ( 3,2, −1)

C/ Pasqual i Batlle, 1-15 08790 Gelida Telèfon: 93 779 04 50 Pàg. 115


e-mail: a8035246@xtec.cat https://agora.xtec.cat/iesgelida
Generalitat de Catalunya
Departament d’Educació
Institut Gelida Departament de Matemàtiques

PQ = Q − P = ( 2,0,1) − (1, −1,2 ) = (1,1, −1)


PR = R − P = ( 3,2, −1) − (1, −1,2 ) = ( 2,3, −3)

x −1 1 2
 : y + 1 1 3 = 0 → −3 ( x − 1) − 2 ( y + 1) + 3 ( z − 2 ) − 2 ( z − 2 ) + 3 ( x − 1) + 3 ( y + 1) = 0 →
z − 2 −1 −3

→ ( y + 1) + ( z − 2 ) = 0 → y + z − 1 = 0

El volum del tetraedre format per 4 punts és la sisena part del volum del
paral·lelepípede que determinen. El volum d’aquest paral·lelepípede es pot calcular
com el valor absolut del producte mixt dels 3 vectors amb origen comú que formen els
4 punts.
x − 5 y −1 z − 5
Un punt genèric de la recta r: = = és:
2 −1 −3
S : ( x, y, z ) = ( 5 + 2,1 − ,5 − 3 )
Per tant, tenim els punts:
P = (1, −1,2) , Q = ( 2,0,1) , R = ( 3,2, −1) i S = ( 5 + 2,1 − ,5 − 3 )
Que formen els vectors amb origen comú en P :
PQ = Q − P = ( 2,0,1) − (1, −1,2 ) = (1,1, −1)
PR = R − P = ( 3,2, −1) − (1, −1,2 ) = ( 2,3, −3)
PS = S − P = ( 5 + 2 ,1 −  ,5 − 3 ) − (1, −1,2 ) = ( 4 + 2 ,2 −  ,3 − 3 )
1 2 4 + 2
 PQ, PR, PS  = 1 3 2 −  =
 
−1 −3 3 − 3
= 3( 3 − 3 ) − 3( 4 + 2 ) − 2 ( 2 −  ) + 3( 4 + 2 ) + 3( 2 −  ) − 2 (3 − 3 ) =
= ( 3 − 3 ) + ( 2 −  ) = 5 − 4
1 x6
Volum tetaedre = ⎯⎯→Volum paral·lelepípede = 3 →  PQ, PR, PS  = 3 →
2
5 − 4 = 3 → 2 = 4 →  = 12
→ 5 − 4 = 3 → 5 − 4 = 3 → 
5 − 4 = −3 → 8 = 4 →  = 2

Per tant, hi ha dos punts S amb les condicions del problema:


 = 12
S1 = ( 5 + 2  12 ,1 − 12 ,5 − 3  12 ) → S1 = ( 6, 12 , 72 )
 =2
S2 = ( 5 + 2  2,1 − 2,5 − 3  2 ) → S2 = ( 9, −1, −1)
Tornar a l’enunciat

C/ Pasqual i Batlle, 1-15 08790 Gelida Telèfon: 93 779 04 50 Pàg. 116


e-mail: a8035246@xtec.cat https://agora.xtec.cat/iesgelida
Generalitat de Catalunya
Departament d’Educació
Institut Gelida Departament de Matemàtiques

82) PAU 2013 Sèrie 5 Qüestió 5:


Considereu els punts A = ( −1,2,4 ) i B = ( 3,0, −2 ) .
a) Trobeu l’equació del pla format per tots els punts que equidisten de A i B .
b) Donat un punt C = ( x, y, z ) , dividim el segment AC en tres parts iguals i obtenim
els punts A, A1 , B i C . Trobeu el punt C .

AB = B − A = ( 3,0, −2 ) − ( −1,2,4 ) = ( 4, −2, −6 )


El punt mitjà del segment AB serà:
1 1
M AB = A + AB = ( −1, 2, 4 ) + ( 4, −2, −6 ) =
2 2
= ( −1, 2, 4 ) + ( 2, −1, −3) = (1,1,1)

El pla buscat serà el pla que passa per M AB = (1,1,1) i

té com a vector normal el vector AB = ( 4,0, −6 ) , per


tant:
(1,1,1)
 : 4 x + 0 y − 6 z + D = 0 → 4 x − 6 z + D = 0 ⎯⎯⎯⎯ → 4 1 − 6 1 + D = 0 →
→ 4 − 6 + D = 0 → −2 + D = 0 → D = 2 →  : 4 x − 6 z + 2 = 0 ⎯⎯ 2

→  : 2 x − 3z + 1 = 0

D’acord amb el dibuix tenim:


3 3
C = A+ AB = ( −1,2,4 ) + ( 4, −2, −6 ) =
2 2
= ( −1,2,4 ) + ( 6, −3, −9 ) = ( 5, −1, −5 )

Tornar a l’enunciat

83) PAU 2013 Sèrie 1 Qüestió 3:


x −1 z+m
Donats el pla  : x + 2y − z = 3 i la recta r : =y= ,
2 4
a) Comproveu que el vector característic (o normal) de  i el vector director de r són
perpendiculars.
b) Estudieu la posició relativa de  i r en funció del paràmetre m.

El vector normal al pla  : x + 2y − z = 3 és n := (1,2, −1) mentre que el vector


x −1 z+m
director de la recta r : =y= és vr = ( 2,1,4) .
2 4
Aquests vectors seran perpendiculars sii el seu producte escalar és zero.

C/ Pasqual i Batlle, 1-15 08790 Gelida Telèfon: 93 779 04 50 Pàg. 117


e-mail: a8035246@xtec.cat https://agora.xtec.cat/iesgelida
Generalitat de Catalunya
Departament d’Educació
Institut Gelida Departament de Matemàtiques

n  vr = (1,2, −1)  ( 2,1,4) = 2 + 2 − 4 = 0 → n ⊥ v

El fet de que n i vr siguin perpendiculars fa que el pla


 i la recta r solament puguin ser o bé paral·lels o que
la recta estigui continguda en el pla tal i com es pot
apreciar en el dibuix adjunt.

Donat que el paràmetre m no influeix en el vector de la


recta per a qualsevol valor de m solament hi haurà les dos possibilitats anteriors, és a
dir, que la recta sigui paral·lela al pla o que hi estigui continguda.

Sigui A = (1,0, −m) un punt de la recta r . Aleshores si A  la recta estarà


continguda en el pla i si A  aleshores serà paral·lela.

Substituint el punt A en l’equació del pla  tenim que


1 + 2  0 − ( −m ) = 1 + 0 + m = 1 + m = 3 → m = 2

Per tant,
Si m = 2 aleshores la recta r està continguda en el pla  mentre que
Si m  2 aleshores la recta r és paral·lela a  .

NOTA: També podíem treballar per rangs arribant com és evident al mateix resultat.
 x −1
 = y → x −1 = 2y → x − 2y = 1
x −1 z+m  2 x − 2 y = 1
r: =y= → →
2 4  y = z + m → 4 y = z + m → 4 y − z = m 4 y − z = m

 4

 : x + 2 y − z = 3  x + 2 y − z = 3  1 2 −1 3  F → F − F  1 2 −1 3 
    2 2 1 
 x − 2 y = 1  x − 2 y = 1   1 −2 0 1    0 −4 1 −2  
r : 4 y − z = m  4 y − z = m  0 4 −1 m   
    0 4 −1 m 

 1 2 −1 3  F → F + F  1 2 −1
F2  F3
3 
  3 3 2 
  0 4 −1 m    0 4 −1 m 
 0 −4 1 −2  0 0 0 m − 2
   

m−2 =0→m= 2

• m = 2 → Rang ( A) = Rang ( A ') = 2  3 = nº incògnites → S.C.I. amb un grau de


llibertat → la recta r i el pla  tenen una recta en comú →r  .

• m  2 → Rang ( A) = 2  3 = Rang ( A ') → S.I . → r i  no tenen cap punt en comú


→r .
Tornar a l’enunciat

C/ Pasqual i Batlle, 1-15 08790 Gelida Telèfon: 93 779 04 50 Pàg. 118


e-mail: a8035246@xtec.cat https://agora.xtec.cat/iesgelida
Generalitat de Catalunya
Departament d’Educació
Institut Gelida Departament de Matemàtiques

84) PAU 2013 Sèrie 1 Qüestió 5:


Donats el pla  : 2 x − y + 3 z − 8 = 0 i el punt P = ( 6, −3,7 ) ,
a) Trobeu l’equació contínua de la recta que passa per P i és perpendicular a .
b) Trobeu el punt del pla  que està més proper al punt P .

Sigui r la recta buscada. Per ser perpendicular a  tindrem que el vector director de
r coincidirà amb el vector normal al pla  , és a dir: vr = n = ( 2, −1,3) .

Per tant la recta r serà la recta que passa pel punt P = ( 6, −3,7 ) i té com a vector

x−6 y+3 z−7


director vr = ( 2, −1,3) és a dir: r: = =
2 −1 3

Òbviament el punt del pla  que està més proper a P serà


la intersecció entre el pla  i la recta r . Aquest punt
l’anomenarem Q .

Així, per trobar el nostre punt Q solament hem de resoldre el


sistema format per la recta i el pla.
x −6 y +3
x−6 y+3 z−7  2 = −1 → − x + 6 = 2 y + 6 − x = 2 y
r: = = → → r:
2 −1 3  y + 3 = z − 7 → 3y + 9 = −z + 7 3 y + z = −2
 −1 3
− x = 2 y 1 2 0 0  1 2 0 0 
    
r   : 3 y + z = −2 →  0 3 1 −2    0 3 1 −2  
2 x − y + 3z − 8 = 0  2 −1 3 8   0 −5 3 8 
    

1 2 0 0  1 2 0 0 
   
  0 15 5 −10    0 15 5 −10 
 0 −15 9 24   0 0 14 14 
   

14z = 14 → z = 1

3 y + z = −2 → 3 y + 1 = −2 → 3 y = −3 → y = −1

x + 2 y = 0 → x + 2  ( −1) = 0 → x − 2 = 0 → x = 2

Per tant, Q = ( 2, −1,1) .


Tornar a l’enunciat

C/ Pasqual i Batlle, 1-15 08790 Gelida Telèfon: 93 779 04 50 Pàg. 119


e-mail: a8035246@xtec.cat https://agora.xtec.cat/iesgelida
Generalitat de Catalunya
Departament d’Educació
Institut Gelida Departament de Matemàtiques

85) PAU 2014 Sèrie 3 Qüestió 2:


Considereu el punt A = (1,2,3) .
a) Calculeu el punt simètric del punt A respecte de la recta d’equació
r : ( x, y, z ) = ( 3 + , 1, 3 −  ) .

• Calculem el pla  perpendicular a la recta r i que passa pel punt A.


r : ( x, y, z ) = ( 3 + , 1, 3 −  ) → vr = (1,0, −1)

Per ser perpendicular a r , aquest pla  tindrà com a vector normal el vector director
de r . És a dir, n = vr = (1,0, −1) →  :1x + 0 y − 1z + D = 0 →  : x − z + D = 0 →
( )
A = 1,2,3 
⎯⎯⎯⎯ → 1− 3 + D = 0 → D = 2 → : x − z + 2 = 0

• Calculem el punt de tall entre r i :


x − 3 y −1 z − 3 0 = y − 1 → y = 1
r : ( x, y, z ) = ( 3 +  , 1, 3 −  ) → r : = = →r:
1 0 −1 − x + 3 = z − 3 → x + z − 6 = 0

Així el sistema format per r i  és:

y =1 y =1
 
x + z − 6 = 0  x + z = 6 Sumant les dues últimes equacions tenim
 x − z + 2 = 0  x − z = −2
 
2x = 4 → x = 2 → z = 4

Per tant el pla  i la recta r es tallen en el punt P = ( 2,1,4 ) .

• Finalment el simètric de A respecte de r serà A ' = P + AP .


AP = P − A = ( 2,1,4 ) − (1,2,3) = (1, −1,1)

A ' = P + AP = ( 2,1,4 ) + (1, −1,1) → A ' = ( 3,0,5 )

b) Calculeu el punt simètric del punt A respecte del pla que té per equació
 : x + y + z = 3.

• Calculem l’equació de la recta r perpendicular


al pla  i que passa pel punt A .
r ⊥  → vr = n → vr = (1,1,1) ⎯⎯⎯
Ar

r : ( x, y, z ) = (1, 2,3) +  (1,1,1)

• Calculem el punt de tall entre r i .


Un punt genèric de r és P = (  + 1,  + 2,  + 3) . Si
obliguem a que aquest punt pertany també a  tenim:

C/ Pasqual i Batlle, 1-15 08790 Gelida Telèfon: 93 779 04 50 Pàg. 120


e-mail: a8035246@xtec.cat https://agora.xtec.cat/iesgelida
Generalitat de Catalunya
Departament d’Educació
Institut Gelida Departament de Matemàtiques

(  + 1) + (  + 2) + (  + 3) = 3 → 3 + 6 = 3 → 3 = −3 →  = −1 →
→ P = ( −1 + 1, − 1 + 2, − 1 + 3) → P = ( 0, 1, 2)

• Finalment el simètric de A serà A ' = P + AP .


AP = P − A = ( 0, 1, 2 ) − (1, 2, 3) = ( −1, −1, −1)

A ' = P + AP = ( 0,1,2 ) + ( −1, −1, −1) → A ' = ( −1,0,1)


Tornar a l’enunciat

86) PAU 2014 Sèrie 3 Qüestió 5:


x−2 z +1
Siguin r i s les rectes de
3
d’equacions r: =y= i
3 4
s : ( x, y, z ) = (1 + 2 , 3 −  , 4 + 3 ) , amb   .
a) Comproveu que els punts mitjans dels segments que tenen un extrem situat sobre la
recta r i l’altre extrem situat sobre la recta s formen un pla.

x−2 z +1
r: =y= → r : ( x, y, z ) = ( 2,0, −1) +  ( 3,1, 4 ) Per tant un punt genèric de r
3 4
serà P = ( 2 + 3 ,  , − 1 + 4 )

s : ( x, y, z ) = (1 + 2 , 3 −  , 4 + 3 ) → Un punt genèric de s serà


Q = (1 + 2 , 3 −  , 4 + 3 ) .

Per calcular el punt mitjà d’aquest segment PQ sumem els punts i dividim el resultat
P + Q ( 2 + 3 ,  , − 1 + 4 ) + (1 + 2 , 3 −  , 4 + 3 )
entre 2, és a dir: M PQ = = =
2 2
=
( 3 + 3 + 2 , 3 +  −  , 3 + 4 + 3 ) =  3 , 3 , 3  +   3 , 1 , 2  +  1, − 1 , 3 
     
2 2 2 2 2 2   2 2

Multiplicant els vectors per 2 per tal d’evitar denominadors, tenim que els punts mitjans
dels segments formats amb un extrem en la recta r i l’altre extrem en la recta s són:

3 3 3
M PQ =  , ,  +  ( 3, 1, 4 ) +  ( 2, − 1, 3) , per tant aquests punts formen el pla
2 2 2
3 3 3
que passa pel punt  , ,  i té com a vectors directors u = ( 3,1,4) i v = ( 2, −1,3) .
2 2 2

Evidentment és un pla perquè els vectors u iv són linealment independents perquè


3 1
 .
2 −1

C/ Pasqual i Batlle, 1-15 08790 Gelida Telèfon: 93 779 04 50 Pàg. 121


e-mail: a8035246@xtec.cat https://agora.xtec.cat/iesgelida
Generalitat de Catalunya
Departament d’Educació
Institut Gelida Departament de Matemàtiques

b) Trobeu l’equació general (és a dir, que té la forma Ax + By + Cz = D ) del pla de


l’apartat anterior.

3
x−
3 2
2
3 3 3 3
 :  , ,  +  ( 3, 1, 4 ) +  ( 2, − 1, 3 ) →  : y − 1 −1 = 0 →
2 2 2 2
3
z− 4 3
2

 3  3  3  3  3  3
→ 3 x −  + 8 y −  − 3 z −  − 2  z −  + 4  x −  − 9  y −  = 0 →
 2  2  2  2  2  2

 3  3  3 21 3 15
→ 7   x −  −  y −  − 5   z −  = 0 → 7 x − − y + − 5z + =0→
 2  2  2 2 2 2

3
→ 7 x − y − 5z − = 0 ⎯⎯
x2
→  : 14 x − 2 y − 10 z − 3 = 0
2
Tornar a l’enunciat

87) PAU 2014 Sèrie 4 Qüestió 3:


Siguin els punts P = (1,1,0) , Q = (1,0,1) i R = ( 0,1,1) i el pla  :x+ y+z =4.
a) Trobeu l’equació general (es a dir, que té la forma Ax + By + Cz = D ) del pla que
passa pels punts P, Q i R.

Per determinar un pla  ' necessitem un punt i dos vectors directors. Podem calcular el
pla  ' que passa pels punts P, Q i R com el pla que passa per P i té com a vectors
directors PQ i PR .
PQ = Q − P = (1,0,1) − (1,1,0 ) = ( 0, −1,1)
PR = R − P = ( 0,1,1) − (1,1,0 ) = ( −1,0,1)
x − 1 0 −1
 ' : y − 1 −1 0 = 0 →
z −0 1 1

→  ' : −1( x − 1) − 1( y − 1) + 0 ( z − 0) − 1( z − 0 ) − 0 ( x − 1) − 0 ( y − 1) = 0 →

→  ' : −1( x − 1) − 1( y − 1) − 1( z − 0) = 0 →  ' : − x + 1 − y + 1 − z = 0 →

( )x −1
→  ' : − x − y − z = −2 ⎯⎯⎯ →  ': x + y + z = 2

C/ Pasqual i Batlle, 1-15 08790 Gelida Telèfon: 93 779 04 50 Pàg. 122


e-mail: a8035246@xtec.cat https://agora.xtec.cat/iesgelida
Generalitat de Catalunya
Departament d’Educació
Institut Gelida Departament de Matemàtiques

b) Si S és un punt de  , comproveu que el volum del tetraedre de vèrtexs P, Q, R i S no


depèn del punt S.

El volum del tetraedre de vèrtexs P, Q, R i S és la sisena part del volum del


paral·lelepípede de costats PQ , PR i PS .

Per altra banda sabem que el volum d’aquest paral·lelepípede coincideix amb el
valor absolut del producte mixt dels vectors PQ , PR i PS .

Sigui S un punt genèric del pla  :x+ y+z =4 , aleshores S serà de la forma
S = ( x, y,4 − x − y ) i per tant:

PQ = Q − P = (1,0,1) − (1,1,0 ) = ( 0, −1,1)


PR = R − P = ( 0,1,1) − (1,1,0 ) = ( −1,0,1)
PS = S − P = ( x, y,4 − x − y ) − (1,1,0 ) = ( x − 1, y − 1, 4 − x − y )

Finalment:
0 −1 x −1
1 1 1
V =   PQ, PR, PS  =  −1 0 y − 1 =  −1( x − 1) − 1( y − 1) − 1( 4 − x − y ) =
6 6 6
1 1 4− x− y

1 1 1 2 1
=  − x + 1 − y + 1 − 4 + x + y =  −2 =  2 = =
6 6 6 6 3
1
Com podem observar el volum del tetraedre sempre és i no depèn del punt S triat.
3
NOTA: També podem calcular el volum del
tetraedre de vèrtexs P, Q, R i S mitjançant la
fórmula del volum d’una piràmide. El volum d’una
piràmide és un terç de l’àrea de la base per
l’altura. Si considerem com a base de la piràmide
el triangle de vèrtexs P, Q i R que pertanyen al pla
 ' , el volum del tetraedre seria l’àrea del triangle
PQR per l’altura del tetraedre. Una manera de
veure que el volum de la piràmide és fix i no
depèn de l’elecció del punt S de  seria
demostrar que els plans  ' i  són paral·lels, així
l’altura del tetraedre no dependria de l’elecció
de S.

Però el paral·lelisme entre els dos plans és evident perquè  ': x + y + z = 2 i


 :x+ y+z =4 , per tant, n ' = (1,1,1) i n = (1,1,1) . Així els plans  ' i  tenen el
mateix vector normal i per tant són paral·lels.
Tornar a l’enunciat

C/ Pasqual i Batlle, 1-15 08790 Gelida Telèfon: 93 779 04 50 Pàg. 123


e-mail: a8035246@xtec.cat https://agora.xtec.cat/iesgelida
Generalitat de Catalunya
Departament d’Educació
Institut Gelida Departament de Matemàtiques

88) PAU 2014 Sèrie 4 Qüestió 4:


Donats els plans 1 : x − 4 y + z = 2m − 1 i  2 : 2 x − ( 2m + 2) y + 2 z = 3m + 1,
a) Determineu els valors de m perquè els plans  1 i  2 s’intersequin en una recta i
calculeu un vector director de la recta resultant que no depengui de m.

Dos plans s’intersequen en una recta sii el sistema que formen les seves equacions és
compatible determinat amb un grau de llibertat. Considerem la matriu ampliada del
sistema:
1 −4 1 2m − 1 F2 → F2 − 2 F1  1 −4 1 2m − 1 
A' =     
 2 −2m − 2 2 3m + 1   0 −2m + 6 0 −m + 3 

−2m + 6 = 0 → 2m = 6 → m = 3
Noteu que:
• Si m = 3 tota la segona fila es converteix en zeros i aleshores
Rang ( A) = Rang ( A ') = 1 i el sistema és compatible indeterminat amb dos graus de
llibertat, és a dir, el sistema format pels dos plans té tot un pla de solucions i per tant el
que està passant és que en aquest cas els dos plans són el mateix.

• Si m3 aleshores Rang ( A) = Rang ( A ') = 2  3 = nº incògnites → Sistema


Compatible Indeterminat amb un grau de llibertat, per tant, en aquest cas els dos
plans tenen una recta en comú, és a dir s’intersequen en una recta. Així la resposta a
aquest apartat és m3

El vector director de la recta que defineixen dos plans és perpendicular a cadascun


dels vectors normals d’aquests plans, per tant, es pot calcular com el producte
vectorial dels vectors normals a 1 i  2 .
1 : x − 4 y + z = 2m − 1 → n1 = (1, −4,1)
 2 : 2 x − ( 2m + 2) y + 2 z = 3m + 1 → n2 = ( 2, − 2m − 2, 2 )
i j k
vr = n1  n2 = (1, −4,1)  ( 2, − 2m − 2, 2 ) = 1 −4 1 =
2 −2m − 2 2
= −8i + ( −2m − 2 ) k + 2 j + 8k + ( 2m + 2 ) i − 2 j = ( 2m − 6 ) i + 0 j + ( −2m + 6 ) k =
(*)
= ( 2m − 6, 0, − 2m + 6 )  (1, 0, − 1)

(*) Com estem en el cas en que m3 aleshores 2m − 6  0 i podem dividir tot el
vector per 2m − 6 .

Per tant la solució al problema és vr = (1,0, −1)

C/ Pasqual i Batlle, 1-15 08790 Gelida Telèfon: 93 779 04 50 Pàg. 124


e-mail: a8035246@xtec.cat https://agora.xtec.cat/iesgelida
Generalitat de Catalunya
Departament d’Educació
Institut Gelida Departament de Matemàtiques

b) Sigui el pla : 3x − 2 y + 3z = 8 . Estudieu la posició relativa del pla  amb la recta r


definida per la intersecció dels plans  1 i  2 quan m = 1 .

 : 3x − 2 y + 3z = 8 → n = (3, −2,3)
vr  n = (1,0, −1)  (3, −2,3) = 3 − 0 − 3 = 0 → vr ⊥ n → r és paral·lela a  o r està
continguda en  .
Per distingir en quin dels dos casos ens trobem agafem un punt de r i comprovem si
aquest punt pertany a  .
Si en les equacions de r , fem per exemple x = 0 :
1 −4 1 2m − 1 F2 → F2 − 2 F1  1 −4 1 2m − 1  m =1
A' =     
 2 −2m − 2 2 3m + 1   0 −2m + 6 0 −m + 3 
 1
 0 − 4  + z = 1 z = 3
 1 −4 1 1   x − 4 y + z = 1 x =0  2 
 → r: ⎯⎯→  → 1
0 4 0 2 4 y = 2 y = 1  y=
  2
 2
 1 
Per tant un punt de la recta r és P =  0, , 3  . Substituint en l’equació de
 2 
1
 : 3x − 2 y + 3z = 8 tenim 3 0 − 2  + 3 3 = 0 −1+ 9 = 8 → P  → r està
2
continguda en .
NOTA: Una altra manera de resoldre el problema és la següent:
r és la recta intersecció dels plans 1 : x − 4 y + z = 2m − 1 i
 2 : 2 x − ( 2m + 2) y + 2 z = 3m + 1 quan m = 1 .

La matriu ampliada del sistema format per 1 i  2 és:


1 −4 1 2m − 1 F2 → F2 − 2 F1  1 −4 1 2m − 1  m =1  1 −4 1 1 
A' =      
 2 −2m − 2 2 3m + 1   0 −2m + 6 0 −m + 3   0 4 0 2 

Si ara volem intersecar la recta anterior amb el pla  : 3x − 2 y + 3z = 8 l’únic que hem
de fer és afegir al sistema l’equació del pla, és a dir, obtenim el sistema:

 1 −4 1 1  F → F − 3 F  1 −4 1 1  F2 → 1 F2  1 −4 1 1 F = F
  3 3 1  2   2 3  1 −4 1 1
A ' =  0 4 0 2    0 4 0 2  1  0 2 0 1   →
 3 −2 3 8 
F → F
 0 10 0 5  3 5 3  0 2 0 1  0 2 0 1
     

→ Rang ( A) = Rang ( A ') = 2 → La recta r i el pla  formen un sistema compatible


indeterminat amb un grau de llibertat, és a dir, r i  tenen una recta en comú → r
està continguda en .
Tornar a l’enunciat

C/ Pasqual i Batlle, 1-15 08790 Gelida Telèfon: 93 779 04 50 Pàg. 125


e-mail: a8035246@xtec.cat https://agora.xtec.cat/iesgelida
Generalitat de Catalunya
Departament d’Educació
Institut Gelida Departament de Matemàtiques

89) PAU 2014 Sèrie 5 Qüestió 1:


3
Siguin r i s les rectes de que tenen les equacions següents:
z −3 x − 3 y − 2 z +1
r:x+5= y −5= i s: = =
2 2 3 −1
a) Estudieu el paral·lelisme i la perpendicularitat entre les rectes r i s .

z −3 x+5 y −5 z −3
r:x+5= y −5= →r: = =
2 1 1 2
Els vectors directors de les rectes són vr = (1,1,2 ) i vs = ( 2,3, −1)
• Paral·lelisme:
1 1
 → vr i vs no proporcionals → r i s no són paral·leles
2 3
• Perpendicularitat:
vr  vs = (1,1,2)  ( 2,3, −1) = 2 + 3 − 2 = 3  0 → r i s no són perpendiculars

b) Trobeu l’equació general (és a dir, que té la forma Ax + By + Cz + D = 0 ) del pla 


que conté la recta r i és paral·lel a la recta s . Calculeu la distància entre la recta s i
el pla  obtingut.

Si el pla  conté a la recta r aleshores el punt A = ( −5,5,3) de r també serà un punt


de  i el vector director de r , vr = (1,1,2 ) serà un vector director de .
Si el pla  és paral·lel a la recta s, aleshores el vector director de s , vs = ( 2,3, −1)
també serà vector director de .

Per tant,  és el pla que passa pel punt A = ( −5,5,3) i té com a vectors directors
vr = (1,1,2 ) i vs = ( 2,3, −1) .
x+5 1 2
 : y − 5 1 3 = 0 → − ( x + 5) + 4 ( y − 5 ) + 3 ( z − 3) − 2 ( z − 3) − 6 ( x + 5 ) + 1( y − 5 ) = 0 →
z − 3 2 −1

→ −7 ( x + 5) + 5( y − 5) + 1( z − 3) = 0 → −7 x − 35 + 5 y − 25 + z − 3 = 0 →

→ −7 x + 5 y + z − 63 = 0 →  : −7 x + 5 y + z = 63

Com que la recta s és paral·lela al pla  la distància entre s i serà igual a la


distància entre qualsevol punt Ps de s i.

x − 3 y − 2 z +1
s: = = → Ps = ( 3, 2, −1)  s
2 3 −1
−7  3 + 5  2 − 1 − 63
d ( s,  ) = d ( Ps ,  ) = d ( ( 3,2, −1) , − 7 x + 5 y + z − 63 = 0 ) = =
( −7 ) + 52 + 12
2

C/ Pasqual i Batlle, 1-15 08790 Gelida Telèfon: 93 779 04 50 Pàg. 126


e-mail: a8035246@xtec.cat https://agora.xtec.cat/iesgelida
Generalitat de Catalunya
Departament d’Educació
Institut Gelida Departament de Matemàtiques

−21 + 10 − 1 − 63 −75 75 75 75
= = = = = 75 = 5 3
49 + 25 + 1 75 75 75
Tornar a l’enunciat

90) PAU 2015 Sèrie 2 Qüestió 2:


x −1 y + 3
Sigui r la recta de l’espai que té per equació r : = = z i sigui P el punt de
2 −1
coordenades ( 6,0, −1) .
a) Trobeu l’equació cartesiana (és a dir, que té la forma Ax + By + Cz = D del pla que
passa pel punt P i talla perpendicularment la recta r .

Anomenem  al pla buscat:


P = ( 6,0, −1)
 ⊥ r → n = vr → n = ( 2, −1,1) →  : 2 x − y + z + D = 0 ⎯⎯⎯⎯⎯ →

→ 12 − 0 − 1 + D = 0 → 11 + D = 0 → D = −11 →  : 2 x − y + z − 11 = 0

b) Trobeu l’equació paramètrica del pla que passa pel punt P i conté la recta r .

La recta r passa pel punt Q = (1, −3,0) i té com a vector director vr = ( 2, −1,1)
Sigui ' el pla buscat. Com ' conté la recta r aleshores el vector vr serà vector
director de  ' i el punt Q també serà un punt de  '.
Però si els punts P iQ són de  ' aleshores el vector PQ serà vector director de  '.
PQ = Q − P = (1, −3,0 ) − ( 6,0, −1) = ( −5, −3,1) .
Finalment calculem l’equació del pla  ' com el pla que passa pel punt P = ( 6,0, −1) i
té com a vectors directors vr = ( 2, −1,1) i PQ = ( −5, −3,1) , per tant:
 ' : ( x, y, z ) = ( 6,0, −1) +  ( 2, −1,1) +  ( −5, −3,1)
Tornar a l’enunciat

91) PAU 2015 Sèrie 2 Qüestió 4:


Considereu a
3
la recta que té per equació r : ( x, y, z ) = ( −4 + 2, −2,1 −  ) i els
plans 1 i  2 1 : x + 2 y + 2 z = −1 i  2 : x − 2 y + 2 z = −3 , respectivament.
d’equacions
a) Determineu la posició relativa de  1 i  2 .

1 : x + 2 y + 2 z = −1 → 1 : x + 2 y + 2 z + 1 = 0
 2 : x − 2 y + 2 z = −3 →  2 : x − 2 y + 2 z + 3 = 0

C/ Pasqual i Batlle, 1-15 08790 Gelida Telèfon: 93 779 04 50 Pàg. 127


e-mail: a8035246@xtec.cat https://agora.xtec.cat/iesgelida
Generalitat de Catalunya
Departament d’Educació
Institut Gelida Departament de Matemàtiques

1 2
 → n1 i n2 no proporcionals → n1 i n2 no paral·lels → 1 i  2 es tallen en una
1 −2
recta.

b) Comproveu que tots els punts de la recta r estan situats a la mateixa distància dels
plans 1 i  2 .

NOTA: Podeu calcular la distància d’un pont de coordenades ( x0 , y0 , z0 ) al pla


Ax0 + By0 + Cz0 + D
d’equació Ax + By + Cz + D = 0 amb l’expressió
A2 + B 2 + C 2

Hem d’agafar un punt genèric de la recta r i comprovar que la distància d’aquest


punt a cadascun dels plans és la mateixa.

r : ( x, y, z ) = ( −4 + 2, −2,1 −  ) → Un punt genèric P de la recta r és

P = ( −4 + 2, −2,1 −  ) .

−4 + 2 + 2 ( −2 ) + 2 (1 −  ) + 1 −4 + 2 − 4 + 2 − 2 + 1 −5 5
d ( P,  1 ) = = = =
1 +2 +2
2 2 2
1+ 4 + 4 9 3

−4 + 2 − 2 ( −2 ) + 2 (1 −  ) + 3 −4 + 2 + 4 + 2 − 2 + 3 5 5
d ( P,  2 ) = = = =
12 + ( −2 ) + 22
2
1+ 4 + 4 9 3
Tornar a l’enunciat

92) PAU 2015 Sèrie 4 Qüestió 3:


Siguin el punt P = ( 2,0,2) i el pla  d’equació x − y + z = 1 .
a) Calculeu l’equació paramètrica de la recta que passa pel punt P i és
perpendicular al pla  .

Sigui r la recta buscada. r ⊥  → vr = n → vr = (1, −1,1)


Per tant, la recta buscada és:

 x = 2 + 1 x = 2 + 
 
r : ( x, y, z ) = ( 2,0, 2 ) +  (1, −1,1) → r :  y = 0 − 1 → r :  y = −
 z = 2 + 1 z = 2 + 
 

C/ Pasqual i Batlle, 1-15 08790 Gelida Telèfon: 93 779 04 50 Pàg. 128


e-mail: a8035246@xtec.cat https://agora.xtec.cat/iesgelida
Generalitat de Catalunya
Departament d’Educació
Institut Gelida Departament de Matemàtiques

b) Calculeu la distància del punt P al pla  .


NOTA: Podeu calcular la distància d’un punt de coordenades ( x0 , y0 , z0 ) al pla
Ax0 + By0 + Cz0 + D
d’equació Ax + By + Cz + D = 0 amb l’expressió .
A2 + B 2 + C 2

 : x − y + z = 1 →  : x − y + z −1 = 0

Ax0 + By0 + Cz0 + D 1 2 − 1 0 + 1 2 − 1 2 − 0 + 2 −1 3 3


d ( P,  ) = = = = = = 3
A + B +C
2 2 2
1 + ( −1) + 1
2 2 2 1+1+1 3 3
Tornar a l’enunciat

93) PAU 2015 Sèrie 4 Qüestió 6:


Siguin a
3
P = ( 2,3,3) i la recta r : ( x, y, z ) = (1,2,3) + t (1,1,1) .
el punt
a) Calculeu l’equació paramètrica del pla que passa pel punt P i conté la recta r .

Sigui A = (1,2,3)  r
Sigui  el pla que ens demanen. Com  conté la recta r aleshores el punt A  i
per tant com que P també pertany a 
podem assegurar que el vector AP és un
vector director de . Així podem calcular el pla  com el pla que passa pel punt
P = ( 2,3,3) i té com a vectors directors vr = (1,1,1) i

AP = P − A = ( 2,3,3) − (1,2,3) = (1,1,0 ) . Així:

 : ( x, y, z ) = ( 2,3,3) +  (1,1,1) +  (1,1,0 ) →  : ( 2 +  +  , 3 +  +  , 3 +  )

b) Calculeu l’equació cartesiana (és a dir, que té la forma Ax + By + Cz = D del pla


que passa pel punt P i és perpendicular a la recta r .

Sigui  el pla demanat. Com que  ⊥r aleshores n = vr = (1,1,1)


Per tant  és de la forma 1x + 1y + 1z + D = 0

Finalment com que  P tenim que P  i així:


passa pel punt
P = ( 2,3,3)   : x + y + z + D = 0 → 2 + 3 + 3 + D = 0 → D = −8 →
→ : x + y + z −8 = 0 →  : x + y + z = 8
Tornar a l’enunciat

C/ Pasqual i Batlle, 1-15 08790 Gelida Telèfon: 93 779 04 50 Pàg. 129


e-mail: a8035246@xtec.cat https://agora.xtec.cat/iesgelida
Generalitat de Catalunya
Departament d’Educació
Institut Gelida Departament de Matemàtiques

94) PAU 2015 Sèrie 5 Qüestió 2:


A l’espai tridimensional considereu la recta r : ( x, y, z ) = ( 3 + 2 , −  , 3 −  ) i els plans
1 : x + y + z = −1 i  2 : ( x, y, z ) = ( 2 + ,1 −  + ,  ) .
a) Calculeu l’equació cartesiana (és a dir, que té la forma Ax + By + Cz = D del pla
2

 2 : ( x, y, z ) = ( 2 + ,1 −  + ,  ) → ( x, y, z ) = ( 2,1,0 ) +  (1, −1,0 ) +  ( 0,1,1) →


x−2 1 0
→  : y − 1 −1 1 = 0 → −1( x − 2 ) + z − 1( y − 1) = 0 → − x + 2 + z − y + 1 = 0 →
z 0 1
( ) x −1
→ − x − y + z + 3 = 0 ⎯⎯⎯ → 2 : x + y − z − 3 = 0

b) Trobeu els dos punts de la recta r que equidisten dels plans 1 i  2 .


NOTA: Podeu calcular la distància d’un punt de coordenades ( x0 , y0 , z0 ) al pla
Ax0 + By0 + Cz0 + D
d’equació Ax + By + Cz + D = 0 amb l’expressió .
A2 + B 2 + C 2
Un punt genèric de la recta r : ( x, y, z ) = ( 3 + 2 , −  , 3 −  ) serà

P = ( 3 + 2 , −  , 3 −  ) .

Calculem la distància d’aquest punt als dos plans 1 : x + y + z + 1 = 0 i


2 : x + y − z − 3 = 0:
1  ( 3 + 2 ) + 1  ( − ) + 1  ( 3 −  ) + 1 3 + 2 −  + 3 −  + 1 7 7
d ( P, 1 ) = = = =
12 + 12 + 12 1+1+1 3 3
1  ( 3 + 2 ) + 1  ( − ) − 1  ( 3 −  ) − 3 3 + 2 −  − 3 +  − 3 2 − 3
d ( P,  2 ) = = =
12 + 12 + ( −1)
2
1+1+1 3

7 2 − 3 x 3
d ( P, 1 ) = d ( P,  2 ) → = ⎯⎯⎯ → 7 = 2 − 3 → 2 − 3 = 7 →
3 3

2 − 3 = 7 2 = 7 + 3 2 = 10  = 5 P = ( 3 + 2 , − , 3 − )


→ → → → ⎯⎯⎯⎯⎯⎯⎯ →
2 − 3 = −7 2 = −7 + 3 2 = −4  = −2

 P = 13, −5, −2
 ( )
→
 P = ( −1,2,5 )

Tornar a l’enunciat

C/ Pasqual i Batlle, 1-15 08790 Gelida Telèfon: 93 779 04 50 Pàg. 130


e-mail: a8035246@xtec.cat https://agora.xtec.cat/iesgelida
Generalitat de Catalunya
Departament d’Educació
Institut Gelida Departament de Matemàtiques

95) PAU 2015 Sèrie 5 Qüestió 4:


Siguin els plans de
3
1 : − y + z = 2 ,  2 : −2 x + y + z = 1 i  3 : 2 x − 2 z = −1 .
a) Calculeu la posició relativa dels tres plans.

• Primer estudiarem les solucions del sistema format pels tres plans:
− y + z = 2  0 −1 1 2  F  F  2 0 −2 −1 F → F + F
   1 3  2 2 1
−2 x + y + z = 1   −2 1 1 1    −2 1 1 1  
2 x − 2 z = −1    
  2 0 −2 −1  0 −1 1 2 

 2 0 −2 −1 F → F + F  2 0 −2 −1
  3 3 2 
  0 1 −1 0    0 1 −1 0  →
 0 −1 1 2  0 0 0 2 
   

→ Rang ( A) = 2  3 = Rang ( A ') → S.I . → Els tres plans no tenen cap punt en comú.

• Ara estudiem la existència de plans paral·lels:


0 −1
 →  1 i  2 no són paral·lels
−2 1

0 −1
 →  1 i  3 no són paral·lels
2 0

−2 1
 →  2 i  3 no són paral·lels
2 0

Per tant no hi ha cap parell de plans


paral·lels i estem en el cas 7, els plans
es tallen dos a dos. En la figura de la
dreta hem representat els plans.

b) Comproveu que el pla 3 és paral·lel a la recta definida per la intersecció dels


plans 1 i  2 .

Calculem primer l’equació vectorial de la recta intersecció de 1 i  2 .


1 : − y + z = 2 − y = 2 − z y = z − 2  := z y =  − 2
   ⎯⎯⎯ →
 2 : −2 x + y + z = 1 −2 x + y = 1 − z −2 x + y = 1 − z −2 x + y = 1 − 

y = −2 3 − 2 2 − 3
−2 x + y = 1 −  ⎯⎯⎯ →−2 x +  − 2 = 1 −  → −2 x = 3 − 2 → x = →x=
−2 2

 2 − 3   −3 
Per tant: ( x, y, z ) =  ,  − 2,   → ( x, y, z ) =  , −2,0  +  (1,1,1)
 2   2 

C/ Pasqual i Batlle, 1-15 08790 Gelida Telèfon: 93 779 04 50 Pàg. 131


e-mail: a8035246@xtec.cat https://agora.xtec.cat/iesgelida
Generalitat de Catalunya
Departament d’Educació
Institut Gelida Departament de Matemàtiques

 −3 
Així la intersecció dels plans 1 i  2 és la recta que passa pel punt A =  , −2,0  i té
 2 
com a vector director v = (1,1,1) .

Finalment per comprovar que el pla  3 és paral·lel a aquesta recta demostrarem que
v és ortogonal al vector normal de  3 i que el punt A no pertany a aquest pla.
 3 : 2 x − 2 z = −1 → n3 = ( 2,0, −2) → n3 (1,0, −1)
v  n3 = (1,1,1)  (1,0, −1) = 1 + 0 − 1 = 0 → v ⊥ n3
 −3 ?
A =  , −2,0  3
 2 
−3
2 − 2  0 = −3 − 0 = −3  −1 → A   3 i per tant la recta és paral·lela a aquest pla.
2
Tornar a l’enunciat

96) PAU 2016 Sèrie 1 Qüestió 1:


Siguin la recta r: (x, y, z) = (5 + k, k, –2 – 2k) i els punts P = (1, 0, –1) i Q = (2, 1, 1).
a) Calculeu l’equació paramètrica de la recta que passa pel punt Q i és perpendicular
al pla determinat per la recta r i el punt P.

• En primer lloc calcularem el pla determinat per la recta r i el punt P.


Per determinar aquest pla que anomenarem  necessitem un punt i dos vectors
directors.
Com que el vector director de la recta r és vr = (1,1, −2) aleshores aquest vector
també serà vector director del pla.
Com que el punt R = ( 5,0, −2 ) és un punt de la recta r també serà un punt del pla 
i per tant el vector RP = P − R = (1,0, −1) − ( 5,0, −2 ) = ( −4,0,1) serà un vector del pla
 donat que tant el punt P com el punt R són punts de .
Finalment calculem el pla  com el pla que passa pel punt P = (1,0, −1) i té com a
vectors directors vr = (1,1, −2) i RP = ( −4,0,1) . Aquest pla és el següent:
x − 1 1 −4
: y 1 0 = 0 → ( x − 1) + 8 y + 4 ( z + 1) − y = 0 → x − 1 + 7 y + 4 z + 4 = 0 →
z + 1 −2 1

→  : x + 7 y + 4z + 3 = 0

Finalment calculem la recta que ens demana l’enunciat, és a dir, la recta


perpendicular al pla  que passa pel punt Q = ( 2,1,1) .
Anomenem s a la recta buscada, aleshores tenim: s ⊥  → vs = n = (1,7,4) i
finalment les equacions paramètriques de la recta s seran:

C/ Pasqual i Batlle, 1-15 08790 Gelida Telèfon: 93 779 04 50 Pàg. 132


e-mail: a8035246@xtec.cat https://agora.xtec.cat/iesgelida
Generalitat de Catalunya
Departament d’Educació
Institut Gelida Departament de Matemàtiques

x = 2 + k

s :  y = 1 + 7k k
 z = 1 + 4k

b) Calculeu el punt de la recta r que equidista dels punts P i Q.

Un punt genèric de la r té la forma A = ( 5 + k , k , −2 − 2k ) .


La distància d’aquest punt A als punts P iQ és el mòdul del vector que determinen.
Així:
AP = P − A = (1,0, −1) − ( 5 + k , k , −2 − 2k ) = ( −4 − k , −k ,1 + 2k )
AQ = Q − A = ( 2,1,1) − ( 5 + k , k , −2 − 2k ) = ( −3 − k ,1 − k ,3 + 2k )
d ( A, P ) = d ( A, Q ) → AP = AQ → ( −4 − k , −k ,1 + 2k ) = ( −3 − k ,1 − k ,3 + 2k ) →

→ ( −4 − k ) + ( −k ) + (1 + 2k ) = ( −3 − k ) + (1 − k ) + (3 + 2k ) ⎯⎯ →
2 2 2 2 2 2 2
x

→ ( −4 − k ) + ( −k ) + (1 + 2k ) = ( −3 − k ) + (1 − k ) + ( 3 + 2k ) →
2 2 2 2 2 2

→ 16 + 8k + k 2 + k 2 + 1 + 4k + 4k 2 = 9 + 6k + k 2 + 1 − 2k + k 2 + 9 + 12k + 4k 2 →
1
 12k + 17 = 16k + 19 → 17 − 19 = 16k − 12k → −2 = 4k → k = −
2

Per tant, el punt buscat serà:

 9 −1 
A = ( 5 + k , k , −2 − 2k ) ⎯⎯⎯
k = −21
→ A =  , , −1
2 2 
Tornar a l’enunciat

97) PAU 2016 Sèrie 1 Qüestió 6:


Responeu a les qüestions següents:
a) Calculeu l’equació cartesiana (és a dir, que té la forma Ax + By + Cz = D) del pla
que passa pel punt de coordenades (0, 0, 1) i és perpendicular als plans 3x + y – z = 1 i x
+ y + 2z = 5.

Sigui el pla que busquem.


 ⊥ 1 → El vector normal a 1 serà vector director de  .
 ⊥  2 → El vector normal a  2 serà vector director de  .
Per tant, en el nostre cas, n1 = ( 3,1, −1) i n2 = (1,1,2 ) seran vectors directors de  .
Així podem calcular el pla  com el pla que passa pel punt ( 0,0,1) i té com a vectors
directors
n1 = ( 3,1, −1) i n2 = (1,1,2 ) .

C/ Pasqual i Batlle, 1-15 08790 Gelida Telèfon: 93 779 04 50 Pàg. 133


e-mail: a8035246@xtec.cat https://agora.xtec.cat/iesgelida
Generalitat de Catalunya
Departament d’Educació
Institut Gelida Departament de Matemàtiques

x−0 3 1 x 3 1
 : y−0 1 1 =0→ y 1 1 = 0 → 2 x − y + 3 ( z − 1) − ( z − 1) + x − 6 y = 0 →
z − 1 −1 2 z − 1 −1 2

→ 3x − 7 y + 2 ( z − 1) = 0 →  : 3x − 7 y + 2 z − 2 = 0

b) Suposeu que un pla 1 és perpendicular a un segon pla 2 i que el pla 2 és a la


vegada perpendicular a un tercer pla  3 . Expliqueu raonadament si necessàriament
els plans 1 i  3 han de ser perpendiculars entre ells.

Agafem com a pla 1 el pla que


conté els eixos X i Y, és a dir, el pla
z =0.

Agafem com a pla 2 el pla que


conté els eixos Y i Z, és a dir, el pla
x = 0.

Agafem com a pla 3 el pla


y + z = 3.

Podem observar que 1 ⊥  2


perquè el vector normal de  1 està
contingut en el pla  2 .
També podem observar que  2 ⊥  3 perquè el vector normal de  2 està contingut en
el pla  3 .
Però els plans  1 i  3 no són perpendiculars perquè el vector normal de  1 que és el
vector ( 0,0,1) , és a dir, l’eix Z, no està contingut en el pla  3 .
NOTA 1: També es pot raonar analíticament.
Dos plans són perpendiculars sii ho son els seus vectors normals. Dos vectors són
perpendiculars sii el seu producte escalar dóna zero.
1 : z = 0 → n1 = ( 0,0,1)
 2 : x = 0 → n2 = (1,0,0)
 3 : y + z = 3 → n3 = ( 0,1,1)
n1  n2 = ( 0,0,1)  (1,0,0) = 0 + 0 + 0 = 0 → 1 ⊥  2
n2  n3 = (1,0,0)  ( 0,1,1) = 0 + 0 + 0 = 0 →  2 ⊥  3

Però
n1  n3 = ( 0,0,1)  ( 0,1,1) = 0 + 0 + 1 = 1 → 0 → 1 ⊥  3

C/ Pasqual i Batlle, 1-15 08790 Gelida Telèfon: 93 779 04 50 Pàg. 134


e-mail: a8035246@xtec.cat https://agora.xtec.cat/iesgelida
Generalitat de Catalunya
Departament d’Educació
Institut Gelida Departament de Matemàtiques

NOTA 2: Una altra il·lustració gràfica encara més fàcil serien dos plans paral·lels tallats
per un perpendicular comú. Per exemple agafant 1 : z = 0  2 : x + y = 3 i  3 : z = 6
podem observar que 1 ⊥  2 , que 2 ⊥ 3 però 1 ⊥  3 perquè de fet 1 i  3 són
paral·lels.
Tornar a l’enunciat

98) PAU 2016 Sèrie 3 Qüestió 2:

A
3
, siguin la recta r que té per equació ( x, y, z ) = (1 + , , 1 −  ) i el pla 
d’equació 2 x − y + z = −2 .
a) Determineu la posició relativa de la recta r i el pla .

r : ( x, y, z ) = (1 +  ,  , 1 −  ) → r : ( x, y, z ) = (1,0,1) +  (1,1, −1) → vr = (1,1, −1)


 : 2 x − y + z = −2 →  : 2 x − y + z + 2 = 0 → n = ( 2, −1,1)
vr  n = (1,1, −1)  ( 2, −1,1) = 2 − 1 − 1 = 0 → vr ⊥ n → o bé r és paral·lela a  o bé r
està continguda en el pla .
Per saber en quin dels dos casos ens trobem agafem un punt de r i comprovem si
pertany a  :

Sigui P = (1,0,1)  r aleshores 2  1 − 0 + 1 + 2 = 2 − 0 + 1 + 2 = 5  0 → P   → r 

NOTA: També podríem calcular la posició relativa entre la recta i el pla mitjançant la
resolució del sistema format per les seves equacions generals. En aquest cas tindríem:
x −1 y z −1 x −1 = y x − y = 1
r : ( x, y, z ) = (1 +  ,  , 1 −  ) → = = → 
1 1 −1 − y = z − 1  y + z = 1

Per tant el sistema format per la recta i el pla és:

2 x − y + z = −2  2 −1 1 −2  E E  1 −1 0 1  E →E −2 E

→ A ' =  1 −1 0 1    2 −1 1 −2  
1 2 2 2 1

x − y = 1
y + z =1 0 1 1 1  0 1 1 1 
    

 1 −1 0 1  E →E − E  1 −1 0 1 
  0 1 1 −4    0 1 1 −4  → Rang ( A) = 2  3 = Rang ( A ' ) → r i 
3 3 2

0 1 1 1  0 0 0 5 
   

no tenen cap punt en comú →r 

C/ Pasqual i Batlle, 1-15 08790 Gelida Telèfon: 93 779 04 50 Pàg. 135


e-mail: a8035246@xtec.cat https://agora.xtec.cat/iesgelida
Generalitat de Catalunya
Departament d’Educació
Institut Gelida Departament de Matemàtiques

b) Calculeu la distància entre la recta r i el pla .


Nota: Podeu calcular la distància d’un punt de coordenades ( x0 , y0 , z0 ) al pla
Ax0 + By0 + Cz0 + D
d’equació Ax + By + Cz + D = 0 amb l’expressió .
A2 + B 2 + C 2

Com la recta r és paral·lela al pla  , la distància entre qualsevol punt de r i el pla 


serà constant i coincidirà amb la distància entre r i  . És a dir:

2 1 − 0 + 1 + 2
P = (1,0,1)  r ⎯⎯→
r
d ( r ,  ) = d ( P,  ) = =
2 + ( −1) + 1
2 2 2

2 − 0 +1+ 2 5 5 5 6
= = = =
4 +1+1 6 6 6

NOTA: En lloc d’aplicar una fórmula també es pot calcular el punt Q projecció
perpendicular de P sobre  i calcular d ( P, Q ) .
Tornar a l’enunciat

99) PAU 2016 Sèrie 5 Qüestió 2:


Siguin a
3
el pla  d’equació x − y + 2 z = 2 i els punts A = ( 3, −1,2 ) i B = (1,1, −2 ) .
a) Comproveu que els punts AiB són simètrics respecte del pla .

Per veure que els dos punts AiB són simètrics respecte del pla  demostrarem que
el vector que formen AB és perpendicular al pla  i que el punt mitjà del segment
AB és un punt de  .
Noteu que NO seria suficient demostrar que estan a
la mateixa distància del pla tal i com mostra la figura.

AB = B − A = (1,1, −2 ) − ( 3, −1,2 ) = ( −2,2, −4 )


Aquest segment serà perpendicular al pla  si té la mateixa direcció que el vector
normal al pla n , és a dir, si els dos vectors són proporcionals. És evident que

AB = ( −2,2, −4 )
i n = (1, −1,2 ) són proporcionals perquè AB = −2n .
Finalment demostrem que el punt mitjà del segment AB és un punt del pla.
Calculem el punt mitjà mitjançant la fórmula:
1 1
M AB = A + AB = ( 3, −1,2 ) + ( −2,2, −4 ) = ( 3, −1,2 ) + ( −1,1, −2 ) = ( 2,0,0 )
2 2
Aquest punt pertany al pla perquè compleix la seva equació, és a dir: 2 − 0 + 20 = 2 .
Aleshores podem assegurar que AiB són simètrics respecte del pla .

NOTA: També haguéssim pogut demostrar que el vector AB és perpendicular al pla i


desprès comprovar que la distància entre els dos punts i el pla és la mateixa.
C/ Pasqual i Batlle, 1-15 08790 Gelida Telèfon: 93 779 04 50 Pàg. 136
e-mail: a8035246@xtec.cat https://agora.xtec.cat/iesgelida
Generalitat de Catalunya
Departament d’Educació
Institut Gelida Departament de Matemàtiques

b) Si r és la recta dels punts P que té per equació P = B + v , en què  és un


paràmetre real i v = (1,1,0 ) , verifiqueu que els punts mitjans dels segments AP
pertanyen al pla .

Si ens imaginem la situació, donat que els punts A i B


són simètrics respecte del pla  , per a que es
compleixi la condició solament seria necessari que el
vector director de la recta r fos paral·lel al pla, és a
dir, és suficient demostrar que v = (1,1,0 ) és
perpendicular a n = (1, −1,2) . Però n  v = (1, −1,2)  (1,1,0 ) = 1 − 1 + 0 = 0 → n ⊥ v .

● També podem resoldre el problema fent exactament el que ens marca l’enunciat tot
i que evidentment és més llarg.

Agafem un punt genèric de la recta r:


r : P = B + v = (1,1, −2) +  (1,1,0) → P = (1 + ,1 + , −2 )

Calculem el punt mitjà del segment AP com la semisuma dels seus extrems:
A + P ( 3, −1,2 ) + (1 +  ,1 +  , −2 ) ( 4 +  ,  , 0 )
M= = = = ( 2 + 2 , 2 , 0 )
2 2 2

Finalment comprovem que aquest punt mitjà M = ( 2 + 2 , 2 , 0 ) pertany al pla  :


 : x − y + 2 z − 2 = 0 ⎯⎯
M
→ 2 + 2 − 2 + 2  0 − 2 = 0 → M  
Tornar a l’enunciat

100) PAU 2016 Sèrie 5 Qüestió 5:


x−3
Siguin les rectes r : ( x, y, z ) = ( 2,3, −3) +  (1, −1,0 ) i s : = y −5= z +2.
2
a) Estudieu si les rectes r i s són paral·leles o perpendiculars.

vr = (1, −1,0) i vs = ( 2,1,1)

1 −1
 → vr i vs no proporcionals → r i s no són paral·leles.
2 1

vr  vs = (1, −1,0)  ( 2,1,1) = 2 − 1 + 0 = 1  0 → vr i vs no són perpendiculars → r i s no


són perpendiculars

Per tant, les rectes r i s no són ni paral·leles ni perpendiculars.

C/ Pasqual i Batlle, 1-15 08790 Gelida Telèfon: 93 779 04 50 Pàg. 137


e-mail: a8035246@xtec.cat https://agora.xtec.cat/iesgelida
Generalitat de Catalunya
Departament d’Educació
Institut Gelida Departament de Matemàtiques

b) Determineu la posició relativa de les rectes r i s i calculeu l’equació paramètrica


de la recta t que talla perpendicularment la recta r i la recta s .

• Posició relativa de les rectes r i s:


2 +  = 3 + 2  2 +  = 1  = −1
r : ( x, y, z ) = ( 2,3, −3) +  (1, −1,0 ) 
  
 → 3 −  = 5 +  → 3 −  = 4 →  = −1 →
 s : ( x, y, z ) = ( 3,5, −2 ) +  ( 2,1,1)
 −3 = −2 +  →  = −1   = −1   = −1
  
Per tant, el sistema format per les dues rectes és compatible determinat i r i s es tallen
en un punt que és:
r : ( x, y, z ) = ( 2,3, −3) +  (1, −1,0 )  =−1 
 ( x, y, z ) = (1,4, −3)
 ⎯⎯⎯ → → P = (1,4, −3)

 s : ( x , y , z ) = ( 3,5, −2 ) +  ( 2,1,1)  =−1

( x , y , z ) = (1,4, −3 )
NOTA: Una altra manera de saber la posició relativa entre les rectes r i s és fer el
següent:
Agafem un punt de r , en aquest cas el més fàcil és el punt A = ( 2,3, −3) i un punt de
s , en aquest cas el més fàcil seria B = ( 3,5, −2 ) .
Construïm el vector AB = B − A = ( 3,5, −2 ) − ( 2,3, −3) = (1,2,1) .
Si  AB, vr , vs  = 0 aleshores les rectes es tallen en un punt mentre que si
 
 AB, vr , vs   0 aleshores les rectes es creuen. En aquest cas:
 
1 1 2
 AB, vr , vs  = 2 −1 1 = −1 + 0 + 1 + 2 − 0 − 2 = 0 → r i s es tallen.
 
1 0 1
Aquest mètode, en aquest cas, té l’inconvenient de que no ens dóna el punt de tall
necessari per acabar el problema.

• Equació paramètrica de la recta t que talla perpendicularment la recta r i la recta


s.
Observant l’esquema de la dreta podem veure que la recta t
buscada passarà pel punt P i tindrà com a vector director vt
un vector que ha de ser perpendicular a vr ia vs , per tant:
i j k
vt = vr  vs = 1 −1 0 = −i + k + 2k − j = −i − j + 3k = ( −1, −1,3)
2 1 1

x = 1 − k

Finalment la recta t serà: t :  y = 4 − k
 z = −3 + 3k

Tornar a l’enunciat

C/ Pasqual i Batlle, 1-15 08790 Gelida Telèfon: 93 779 04 50 Pàg. 138


e-mail: a8035246@xtec.cat https://agora.xtec.cat/iesgelida
Generalitat de Catalunya
Departament d’Educació
Institut Gelida Departament de Matemàtiques

101) PAU 2017 Sèrie 1 Qüestió 2:


a) Considereu els plans 1 : 5 x − y − 7 z = 1 i  2 : 2 x + 3 y + z = 5 .
Determineu l’equació general (és a dir, la que té la forma Ax + By + Cz = D ) del pla
que passa per l’origen de coordenades i és perpendicular als plans 1 i  2 .

Sigui el pla buscat.


 ⊥ 1 i  ⊥  2 aleshores n ⊥ n 1
i n ⊥ n 2 . Per tant, podem calcular n com el
producte vectorial dels vectors n1 i n 2 .

i j k
Aleshores tenim que n = n1  n 2 = ( 5, −1, −7 )  ( 2,3,1) = 5 −1 −7 =
2 3 1
= −i + 15k − 14 j + 2k + 21i − 5 j = 20i − 19 j + 17k = ( 20, − 19, 17 )
I si ha de passar per l’origen de coordenades aleshores D=0 obtenint que l’equació
del pla buscat és:
 : 20 x − 19 y + 17 z = 0

b) Calculeu l’angle que formen els plans 1 i  2 .

L’angle entre els plans 1 i  2 serà el mateix que l’angle entre els seus vectors normals.
Calculem aquest últim angle mitjançant la definició de producte escalar.
u v
u  v = u v cos  → cos  =
u v
n1  n 2 ( 5, −1, −7 )  ( 2, 3, 1)
En el nostre cas cos  = = =
n1 n 2 ( 5, −1, −7 ) ( 2, 3, 1)
10 − 3 − 7 0
= = = 0 → cos  = 0 →  = arcos ( 0 ) →
52 + ( −1) + ( −7 )  22 + 32 + 12
2 2
75  14

→  = 90º
Tornar a l’enunciat

102) PAU 2017 Sèrie 2 Qüestió 1:


Considereu el pla  : x + y + z =1 i la recta r que passa pels punts P = ( 0,0,6 ) i
Q = (1, 2,3)
a) Estudieu la posició relativa de la recta r i el pla .

El vector director de la recta r és vr = PQ = Q − P = (1, 2,3) − ( 0,0,6 ) = (1, 2, −3)


vr  n = (1, 2, −3)  (1,1,1) = 1 + 2 − 3 = 0 → vr ⊥ n → O bé la recta i el pla són paral·lels
o bé la recta està inclosa en el pla.

C/ Pasqual i Batlle, 1-15 08790 Gelida Telèfon: 93 779 04 50 Pàg. 139


e-mail: a8035246@xtec.cat https://agora.xtec.cat/iesgelida
Generalitat de Catalunya
Departament d’Educació
Institut Gelida Departament de Matemàtiques

Per saber en quin dels dos casos ens trobem, agafem P  r i comprovem si aquest
punt pertany a  .
P = ( 0,0,6)  r i 0 + 0 + 6 = 6  1 → P  → r paral·lela a  .

b) Calculeu la distància entre la recta r i el pla .


NOTA: Podeu calcular la distància d’un punt de coordenades ( x0 , y0 , z0 ) al pla
Ax0 + By0 + Cz0 + D
d’equació Ax + By + Cz + D = 0 amb l’expressió .
A2 + B 2 + C 2

 : x + y + z = 1 →  : x + y + z −1 = 0
0 + 0 + 6 −1 5
d ( r ,  ) = d ( P,  ) = d ( ( 0, 0, 6 ) , x + y + z − 1 = 0 ) =
5 5 3
= = =
12 + 12 + 12 3 3 3
Tornar a l’enunciat

103) PAU 2017 Sèrie 2 Qüestió 5:


x − z = 2
i el punt P = ( 0,1, −1) .
3
A , siguin la recta r:
2 y + z = 4
a) Calculeu l’equació general (és a dir, la que té la forma Ax + By + Cz = D ) del pla 
perpendicular a la recta r i que passa pel punt P.

 ⊥ r → n = vr
i j k
vr = (1, 0, −1)  ( 0, 2,1) = 1 0 −1 = 0i + 2k − 0 j − 0k + 2i − j = 2i − j + 2k = ( 2, −1, 2 )
0 2 1
n = ( 2, −1, 2 ) →  : 2 x − y + 2 z + D = 0 ⎯⎯⎯⎯⎯
( )
→ 2  0 − 1 + 2  ( −1) + D = 0 →
P = 0,1, −1 

→ 0 − 1 − 2 + D = 0 → −3 + D = 0 → D = 3 →  : 2 x − y + 2 z + 3 = 0

b) Calculeu el punt simètric del punt P respecte del pla x + y + z = −3 .

1) Calculem la recta r perpendicular al pla  : x + y + z = −3 i que passa pel punt


P = ( 0,1, −1) :
r ⊥  → vr = n = (1,1,1) 
 x − 0 y −1 z + 1
 → r : ( x, y, z ) = ( 0,1, −1) +  (1,1,1) → r : = = →
P = ( 0,1, −1)  r 
 1 1 1

x = y −1  x − y = −1
→r:  r:
 y −1 = z + 1 y − z = 2

C/ Pasqual i Batlle, 1-15 08790 Gelida Telèfon: 93 779 04 50 Pàg. 140


e-mail: a8035246@xtec.cat https://agora.xtec.cat/iesgelida
Generalitat de Catalunya
Departament d’Educació
Institut Gelida Departament de Matemàtiques

2) Calculem el punt Q intersecció del pla  amb la recta r :


Per fer-ho resolem el sistema d’equacions format per  i per r :

 x + y + z = −3  1 1 1 −3  F →F − F  1 1 1 −3  F  F
   2 2 1  2 3
 x − y = −1 → A ' =  1 −1 0 −1    0 −2 −1 2  
y − z = 2    
  0 1 −1 2   0 1 −1 2 

 1 1 1 −3  F →F + 2 F  1 1 1 −3 
  3 3 2 
  0 1 −1 2    0 1 −1 2 
 0 −2 −1 2   0 0 −3 6 
   

−3z = 6 → z = −2
y − z = 2 → y − ( −2) = 2 → y + 2 = 2 → y = 0
x + y + z = −3 → x + 0 − 2 = −3 → x = −3 + 2 → x = −1

Per tant, el punt Q intersecció entre la recta i el pla és ( x, y, z ) = ( −1,0, −2)

3) Finalment calculem el simètric de P que anomenarem P ' amb la fórmula


P ' = Q + PQ :
PQ = Q − P = ( −1,0, −2 ) − ( 0,1, −1) = ( −1, −1, −1)

P ' = Q + PQ = ( −1, 0, −2 ) + ( −1, −1, −1) = ( −2, −1, −3)


Tornar a l’enunciat

104) PAU 2018 Sèrie 1 Qüestió 2:


A = ( 0,1,1) i B = (1,1, −1) .
Sigui r la recta que passa pels punts
a) Trobeu l’equació paramètrica de la recta r .

El vector de la recta serà AB = B − A = (1,1, −1) − ( 0,1,1) = (1,0, −2 ) → vr = (1,0, −2 )

x = 0 + 

Per tant, l’equació paramètrica de r serà: r :  y = 1
 z = 1 − 2

b) Calculeu tots els punts de la recta r que estan a la mateixa distància dels plans
1 : x + y = −2 i  2 : x − z = 1 .
NOTA: Podeu calcular la distància d’un punt de coordenades ( x0 , y0 , z0 ) al pla
Ax0 + By0 + Cz0 + D
d’equació Ax + By + Cz + D = 0 amb l’expressió .
A2 + B 2 + C 2

C/ Pasqual i Batlle, 1-15 08790 Gelida Telèfon: 93 779 04 50 Pàg. 141


e-mail: a8035246@xtec.cat https://agora.xtec.cat/iesgelida
Generalitat de Catalunya
Departament d’Educació
Institut Gelida Departament de Matemàtiques

Per calcular els punts de r que equidisten dels plans 1 i  2 hem d’agafar un punt
genèric de r i notar que tal i com tenim expressats els plans no podem emprar la
fórmula de la distància entre un punt i un pla perquè per emplear-la el terme
independent ha d’estar a l’esquerra. Així:
1 : x + y = −2 → 1 : x + y + 2 = 0 i  2 : x − z = 1 →  2 : x − z − 1 = 0
Un punt genèric de r serà: P = (  , 1, 1 − 2 ) i aleshores:
 +1+ 2  +3
d ( P,  1 ) = =
2
1 +1 + 0
2 2
2
 − (1 − 2 ) − 1  − 1 + 2 − 1 3 − 2
d ( P,  2 ) = = =
12 + 02 + ( −1)
2
2 2
 +3 3 − 2
d ( P, 1 ) = d ( P,  2 ) → = →  + 3 = 3 − 2 →  + 3 =  ( 3 − 2 ) →
2 2

 + 3 = 3 − 2 5 = 2  = 5

→ → → 2

 + 3 = −3 + 2 4 = −1  =


−1
4

Amb = 5
2
obtenim P = (  , 1, 1 − 2 ) = ( 52 , 1, − 4)
Amb  = −41 obtenim P = (  , 1, 1 − 2 ) = ( −41 , 1, 23 )
Tornar a l’enunciat

105) PAU 2018 Sèrie 1 Qüestió 4:


Considereu els punts P = ( 3, −2,1) , Q = ( 5,0,3) , R = (1,2,3) i la recta

x + y + 1 = 0
r:
2 y + 3 z − 5 = 0
a) Determineu l’equació general (és a dir, que té la forma Ax + By + Cz = D del pla
que passa per P i Q i és paral·lel a la recta r .

Sigui  el pla buscat. Com que P i Q són punts de  el vector PQ serà vector
director de  . Com que  és paral·lel a la recta r el vector director de r també serà
vector director de  .

PQ = Q − P = ( 5,0,3) − ( 3, −2,1) = ( 2,2,2 ) ⎯⎯


2
→ Podem agafar com a vector

PQ = (1,1,1)
Càlcul del vector director de la recta r :
i j k
vr = (1,1,0 )  ( 0, 2,3) = 1 1 0 = 3i − 3 j + 2k = ( 3, −3, 2 )
0 2 3

C/ Pasqual i Batlle, 1-15 08790 Gelida Telèfon: 93 779 04 50 Pàg. 142


e-mail: a8035246@xtec.cat https://agora.xtec.cat/iesgelida
Generalitat de Catalunya
Departament d’Educació
Institut Gelida Departament de Matemàtiques

Finalment calculem el pla  com el pla que passa pel punt P = ( 3, −2,1) i té com a
vectors directors PQ = (1,1,1) i vr = ( 3, −3,2) .
x−3 1 3
 : y + 2 1 −3 = 0 → 5  ( x − 3) + ( y + 2 ) − 6  ( z − 1) = 0 →
z −1 1 2

→ 5 x − 15 + y + 2 − 6 z + 6 = 0 →  : 5 x + y − 6 z − 7 = 0

b) Donats el pla x + 2 y + mz = 7 i el pla que passa per P , Q i R , trobeu m perquè


siguin paral·lels i no coincidents.

Primer calculem el pla  que passa pels punts P , Q i R com el pla que passa pel
punt P i té com a vectors directors PQ i PR .
PQ = Q − P = ( 5,0,3) − ( 3, −2,1) = ( 2,2,2 ) ⎯⎯
2
→ Podem agafar PQ = (1,1,1)
PR = R − P = (1,2,3) − ( 3, −2,1) = ( −2,4,2 ) ⎯⎯
2
→ Podem agafar PR = ( −1,2,1)
x − 3 1 −1
Per tant:  : y + 2 1 2 = 0 → − ( x − 3) − 2  ( y + 2 ) + 3  ( z − 1) = 0 →
z −1 1 1

→ − x + 3 − 2 y − 4 + 3z − 3 = 0 →  : − x − 2 y + 3z − 4 = 0

NOTA: Dos plans  : Ax + By + Cz + D = 0 i  ' : A ' x + B ' y + C ' z + D ' = 0 són paral·lels
A B C D
si = = 
A' B ' C ' D '

En el nostre cas  : − x − 2 y + 3z − 4 = 0 i  ' : x + 2 y + mz − 7 = 0 seran paral·lels sii


−1 −2 3 −4 −1 3
= =  → = → − m = 3 → m = −3
1 2 m −7 1 m
Tornar a l’enunciat

106) PAU 2018 Sèrie 3 Qüestió 3:


Considereu el pla que té com a vectors directors u = ( −1,3,2 ) i v = ( 2,1,0) i que passa
pel punt A = (1,0,3) .
a) Calculeu l’equació de la recta que és perpendicular al pla i passa pel punt A.

L’equació del pla serà:

C/ Pasqual i Batlle, 1-15 08790 Gelida Telèfon: 93 779 04 50 Pàg. 143


e-mail: a8035246@xtec.cat https://agora.xtec.cat/iesgelida
Generalitat de Catalunya
Departament d’Educació
Institut Gelida Departament de Matemàtiques

x − 1 −1 2
: y 3 1 = 0 → ( x − 1)  ( −2 ) − y  ( −4 ) + ( z − 3)  ( −7 ) = 0 →
z −3 2 0
( ) x −1
→ −2 x + 2 + 4 y − 7 z + 21 = 0 → −2 x + 4 y − 7 z + 23 = 0 ⎯⎯⎯ →  : 2 x − 4 y + 7 z − 23 = 0

Sigui r la recta que hem de calcular.


Com que r ⊥ aleshores vr = n = ( 2, −4,7 )
Com r passa per A = (1,0,3) tenim que l’equació vectorial de r serà:

r : ( x, y, z ) = (1,0,3) +  ( 2, −4,7 )

b) Calculeu la distància del punt P = (1,5,0 ) al pla.


NOTA: Podeu calcular la distància d’un punt de coordenades ( x0 , y0 , z0 ) al pla
Ax0 + By0 + Cz0 + D
d’equació Ax + By + Cz + D = 0 amb l’expressió .
A2 + B 2 + C 2

Substituint el punt P = (1,5,0 ) i el pla  : 2 x − 4 y + 7 z − 23 = 0 en la fórmula


Ax0 + By0 + Cz0 + D
d ( P,  ) = obtenim que:
A2 + B 2 + C 2
2  1 − 4  5 + 7  0 − 23 2 − 20 + 0 − 23 −41 41 41 69
d ( P,  ) = = = = = 4,94u 2
22 + ( −4 ) + 7 2
2
4 + 16 + 49 69 69 69
Tornar a l’enunciat

107) PAU 2018 Sèrie 3 Qüestió 5:


Considereu els punts de l’espai tridimensional A = (1,1,0) , B = ( 3,5,0) i C = (1,0,0) i la
z
recta r : x = y − 1 = .
2
a) Trobeu el punt d’intersecció de la recta r amb el pla que passa pels punts A, B i
C.

Per poder calcular la intersecció del pla i la recta prèviament hem de calcular
l’equació del pla.

● Càlcul del pla  que passa pels punts A, B i C.


 serà el pla que passa pel punt A i té vectors directors AB i AC .
AB = B − A = ( 3,5,0 ) − (1,1,0 ) = ( 2,4,0 )
AC = C − A = (1,0,0 ) − (1,1,0 ) = ( 0, −1,0 )

C/ Pasqual i Batlle, 1-15 08790 Gelida Telèfon: 93 779 04 50 Pàg. 144


e-mail: a8035246@xtec.cat https://agora.xtec.cat/iesgelida
Generalitat de Catalunya
Departament d’Educació
Institut Gelida Departament de Matemàtiques

Noteu que per calcular l’equació de  podríem agafar vectors proporcionals a AB i


a AC donat que el primer es pot dividir entre 2 i el segon entre −1 .

Per tant:
x −1 2 0
 ( −2 )
 : y − 1 4 −1 = 0 → z  ( −2 ) = 0 → −2 z = 0 ⎯⎯⎯ → :z=0
z 0 0

Per tant el pla  és el pla horitzontal que conté els eixos OX i OY, z =0.

z
● Càlcul del punt d’intersecció entre la recta r : x = y − 1 = i el pla  : z = 0
2
x = y −1
z   x = y − 1
r : x = y −1 = →  z  r:
2  y − 1 = 2 2  ( y − 1) = z
Que intersectada amb el pla z = 0 tindrem:
z = 0 → 2  ( y − 1) = 0 → y − 1 = 0 → y = 1
x = y −1 = 1−1 = 0 → x = 0
Per tant el punt d’intersecció és ( x, y, z ) = ( 0,1,0 )

b) Trobeu els punts P de la recta r per als quals el tetraedre de vèrtexs P, A, B i C


3
té un volum de 2u .
NOTA: El volum d’un tetraedre de vèrtexs P, Q, R i S es pot calcular amb l’expressió
1
6
(
det PQ, PR, PS . )
Per poder trobar el punt P de la recta r que compleixi la condició que ens demanen
agafarem un punt genèric de la recta.
z  A = ( 0,1,0 )  r

r : x = y −1 = → → r : ( x, y, z ) = ( 0,1,0 ) +  (1,1,2 )
2  v
 r = ( 1,1,2 )
Així un punt genèric de r serà: P = (  ,  + 1, 2 )

Finalment calcularem el volum del tetraedre de vèrtexs P, A, B i C mitjançant la


fórmula que ens proporcionen en l’enunciat del problema.

Com que el punt P té l’expressió més complicada de tots, enlloc d’agafar-lo com a
referència i calcular els vectorsPA , PB i PC , agafarem com a origen el punt C i
calcularem els vectors CA , CB i CP .

Aleshores obtindrem:
CA = A − C = (1,1,0 ) − (1,0,0 ) = ( 0,1,0 )
C/ Pasqual i Batlle, 1-15 08790 Gelida Telèfon: 93 779 04 50 Pàg. 145
e-mail: a8035246@xtec.cat https://agora.xtec.cat/iesgelida
Generalitat de Catalunya
Departament d’Educació
Institut Gelida Departament de Matemàtiques

CB = B − C = ( 3,5,0 ) − (1,0,0 ) = ( 2,5,0 )


CP = P − C = (  ,  + 1, 2 ) − (1,0,0 ) = (  − 1,  + 1, 2 )

Calculem el volum del tetraedre:

0 2  −1
2  −1
( )
det CA, CB, CP = 1 5  + 1 = −1 
0 2
= −1  4 = −4
0 0 2

V=
1
6
( 1
det CA, CB, CP = −4
6
)
1 −4 = 12  = −3
V = 2 → −4 = 2 → −4 = 12 →  → →  = 3
6 −4 = −12  = 3
Substituint els valors de  en el punt genèric P = (  ,  + 1, 2 ) obtenim:
 = 3 → P = ( 3, 3 + 1, 2  3) = (3,4,6 )
 = −3 → P ' = ( −3, − 3 + 1, 2  ( −3) ) = ( −3, −2, −6 )
Per tant hi ha dos punts de r que compleixen la condició del volum.

Són P = ( 3,4,6 ) i P ' = ( −3, −2, −6 )


Tornar a l’enunciat

108) PAU 2018 Sèrie 5 Qüestió 2:


Siguin el pla d’equació  : x + y − z = 0 i el punt P = ( 2,3,2) .
a) Calculeu el punt simètric del punt P respecte del pla  .

Tal i com mostra la figura per calcular el punt P '


simètric del punt P respecte del pla 
calcularem:
1) La recta r perpendicular al pla  que passa
pel punt P .
2) El punt de tall A entre la recta r i el pla .
3) El punt P' com P ' = A + PA .

1) Comencem aleshores amb la recta r perpendicular al pla  que passa pel punt P
.
r ⊥  → vr = n = (1,1, −1)
P = ( 2,3,2)  r → r : ( x, y, z ) = ( 2,3,2) +  (1,1, −1)

2) Calculem el punt de tall A entre la recta r i el pla  .


Per poder calcular el punt de tall necessitem expressar la recta r amb la seva equació
general per plantejar un sistema.

C/ Pasqual i Batlle, 1-15 08790 Gelida Telèfon: 93 779 04 50 Pàg. 146


e-mail: a8035246@xtec.cat https://agora.xtec.cat/iesgelida
Generalitat de Catalunya
Departament d’Educació
Institut Gelida Departament de Matemàtiques

− ( x − 2 ) = z − 2
x−2 y −3 z −2 
r : ( x, y, z ) = ( 2,3,2 ) +  (1,1, −1) → = = → →
1 1 −1 
 − ( y − 3 ) = z − 2
− x + 2 = z − 2 4 = x + z
→ →r:
− y + 3 = z − 2 5 = y + z
x + y − z = 0

Per tant el sistema format per  i r serà:  x + z = 4
y + z = 5

 1 1 −1 0  F → F − F  1 1 −1 0  F → F + F  1 1 −1 0 
  2 2 1  3 3 2 
A ' =  1 0 1 4    0 −1 2 4    0 −1 2 4 
0 1 1 5 0 1 1 5 0 0 3 9
     

3z = 9 → z = 3
− y + 2 z = 4 → − y + 2  3 = 4 → − y + 6 = 4 → − y = −2 → y = 2
x + y − z = 0 → x + 2 − 3 = 0 → x −1 = 0 → x = 1
Per tant el punt de tall és A = (1,2,3)

3) Finalment calculem el punt P' com P ' = A + PA :

PA = A − P = (1,2,3) − ( 2,3,2 ) = ( −1, −1,1)

P ' = A + PA = (1,2,3) + ( −1, −1,1) = ( 0,1,4 ) → P ' = ( 0,1,4 )

b) Calculeu l’equació cartesiana (és a dir, que té la forma Ax + By + Cz = D ) dels dos


plans paral·lels a  que estan a una distància 3 del punt P.
NOTA: Podeu calcular la distància d’un punt de coordenades ( x0 , y0 , z0 ) al pla
Ax0 + By0 + Cz0 + D
d’equació Ax + By + Cz + D = 0 amb l’expressió .
A2 + B 2 + C 2

Els plans paral·lels al pla  :x+ y−z =0 seran de la forma  ': x + y − z + D' = 0
Donat que volem que la distància d’aquests plans al punt P sigui
3 tindrem:
2 + 3 − 2 + D'
d ( ', P ) = 3 → d ( ' : x + y − z + D ' = 0, P ( 2,3,2 ) ) = 3 → = 3→
1 + 1 + ( −1)
2 2 2

3 + D' 3 + D ' = 3 D ' = 0


→ = 3 → 3 + D ' = 3 → 3 + D ' = 3 →  →
3 3 + D ' = −3  D ' = −6

C/ Pasqual i Batlle, 1-15 08790 Gelida Telèfon: 93 779 04 50 Pàg. 147


e-mail: a8035246@xtec.cat https://agora.xtec.cat/iesgelida
Generalitat de Catalunya
Departament d’Educació
Institut Gelida Departament de Matemàtiques

Tal i com mostra la figura adjunta, donats el pla  i el


punt P hi haurà dos plans paral·lels a  que estiguin a
una distància determinada del punt P (un per sota i
l’altre per sobre de P ). En aquest cas són:
 ': x + y − z = 0 que casualment coincideix amb el

pla  del primer apartat i el pla  '' : x + y − z − 6 = 0 .

Tornar a l’enunciat

109) PAU 2018 Sèrie 5 Qüestió 5:


y−2
Siguin les rectes r1 : x − 1 == z − 5 i r2 : ( x, y, z ) = ( 2 − 3, −1 + ,2 ) .
−1
a) Trobeu l’equació cartesiana (és a dir, que té la forma Ax + By + Cz = D ) del pla
que conté la recta r1 i és paral·lel a la recta r2 .

Per definir un pla necessitem un punt i dos vectors directors.


Sigui  el pla buscat.

Com que la recta r1 està continguda en el pla  aleshores el punt A = (1,2,5) de r


també serà un punt de  i el vector director de r1 , v1 = (1, −1,1) serà vector director
de .

Com que  és paral·lel a la recta r2 aleshores el vector director de r2 , v2 = ( −3,1,0 )


serà vector director de .

Per tant ja podem calcular  com el pla que passa per A = (1,2,5) i té vectors
directors v1 = (1, −1,1) i v2 = ( −3,1,0 ) .
x − 1 1 −3
 : y − 2 −1 1 = 0 → ( x − 1)  ( −1) − ( y − 2 )  3 + ( z − 5 )  ( −2 ) = 0 →
z −5 1 0

( ) x −1
→ − x + 1 − 3 y + 6 − 2 z + 10 = 0 → − x − 3 y − 2 z + 17 = 0 ⎯⎯⎯ →  : x + 3 y + 2 z − 17 = 0

C/ Pasqual i Batlle, 1-15 08790 Gelida Telèfon: 93 779 04 50 Pàg. 148


e-mail: a8035246@xtec.cat https://agora.xtec.cat/iesgelida
Generalitat de Catalunya
Departament d’Educació
Institut Gelida Departament de Matemàtiques

b) Digueu quina condició s’ha de complir perquè existeixi un pla que contingui la recta
r1 i sigui perpendicular a la recta r2 . Amb les rectes r1 i r2 de l’enunciat, comproveu si
existeix un pla que contingui la recta r1 i sigui perpendicular a la recta r2 .

Anomenem  al pla buscat.


És més fàcil raonar en ordre invers. Si  és perpendicular a la recta r2 està clar que el
vector director de r2 és el vector normal de . Això ens
donaria la següent construcció:

Si ara afegim que el pla  contingui la recta r1 hem de poder “acoplar” r1 dintre del
pla, i això solament ho podrem fer si r1 és perpendicular a r2
. Per tant, les dues rectes han de ser perpendiculars.

Noteu que la recta r1 la podem girar per dintre del pla però
no aixecar-la de manera que deixi d’estar continguda en el
pla. Per tant, totes les rectes r1 vàl·lides són perpendiculars a
r2 tal i com mostra el dibuix de la dreta.

Finalment, per comprovar si amb les rectes de l’enunciat existeix aquest tipus de pla
hem de comprovar si les rectes són perpendiculars, és a dir, si el producte escalar dels
seus vectors directors dóna zero.

y−2
r1 : x − 1 = = z − 5 → v1 = (1, −1,1)
−1

r2 : ( x, y, z ) = ( 2 − 3, −1 + ,2) → v2 = ( −3,1,0 )

v1  v2 = (1, −1,1)  ( −3,1,0) = −3 − 1 + 0 = −4  0 → r1 i r2 no són perpendiculars i per tant


en aquest cas no existeix aquest tipus de pla.
Tornar a l’enunciat

C/ Pasqual i Batlle, 1-15 08790 Gelida Telèfon: 93 779 04 50 Pàg. 149


e-mail: a8035246@xtec.cat https://agora.xtec.cat/iesgelida
Generalitat de Catalunya
Departament d’Educació
Institut Gelida Departament de Matemàtiques

110) PAU 2019 Sèrie 1 Qüestió 3:


Un dron es troba en el puntP = ( 2, −3,1) i volem dirigir-lo en línia recta fins al punt més
proper del pla d’equació  : 3x + 4z + 15 = 0 .
a) Calculeu l’equació de la recta, en forma paramètrica, que ha de seguir el dron.
Quina distància ha de recórrer fins a arribar al pla?

La recta que seguirà el dron és la perpendicular al pla que passa per P.


Per tant, hem de calcular la recta perpendicular al pla
 : 3x + 4z + 15 = 0 que passa pel punt P = ( 2, −3,1) .

Sigui r la recta buscada. Tenim que el vector director


de r serà el vector normal de  . Per tant,
vr = n = ( 3,0,4 ) .

Com que sabem que el punt P és un punt de r


aleshores l’equació vectorial de r serà:
r : ( x, y, z ) = ( 2, −3,1) +  ( 3,0,4 ) i per tant la paramètrica:
 x = 2 + 3

r :  y = −3
 z = 1 + 4

La distància que ha de recórrer el dron és la distància que el separa del pla, és a dir:
3  2 + 4  1 + 15 6 + 4 + 15
d ( P,  ) = d ( ( 2, −3,1) ,  : 3x + 4 z + 15 = 0 ) = = =
32 + 02 + 42 9 + 0 + 16
25 25
= = = 5
25 5

b) Trobeu les coordenades del punt del pla on arribarà el dron.


NOTA: Podeu calcular la distància que hi ha d’un punt de coordenades ( x0 , y0 , z0 ) al
Ax0 + By0 + Cz0 + D
pla d’equació Ax + By + Cz + D = 0 amb l’expressió .
A2 + B 2 + C 2

El punt del pla on arribarà el dron és la intersecció Q entre


la recta r i el pla  .
Calculem per tant la intersecció entre la recta
 x = 2 + 3

r :  y = −3 i el pla  : 3x + 4z + 15 = 0 .
 z = 1 + 4

Per crear el sistema d’equacions, en primer lloc hem de passar la recta r a la seva
equació general:

C/ Pasqual i Batlle, 1-15 08790 Gelida Telèfon: 93 779 04 50 Pàg. 150


e-mail: a8035246@xtec.cat https://agora.xtec.cat/iesgelida
Generalitat de Catalunya
Departament d’Educació
Institut Gelida Departament de Matemàtiques

 x = 2 + 3  x − 2 = 3  x −3 2 = 
    x − 2 = z 4−1 4  ( x − 2 ) = 3  ( z − 1)
r :  y = −3 →  y = −3 →  y = −3 →  3 → →
 z = 1 + 4  z − 1 = 4  z −1 =   y = −3  y = −3
   4

4 x − 8 = 3 z − 3 4 x − 3 z = 5
→ →
 y = −3  y = −3

Per tant, el sistema format per la recta i el pla és:

 4 x − 3 z = 5 4 x − 3 z = 5 4 x − 3z = 5 F → 4 F 16 x − 12 z = 20
r :    1 1

  y = −3   y = −3  3x + 4 z = −15  9 x + 12 z = −45
F2 → 3 F2
 : 3x + 4 z + 15 = 0 3x + 4 z = −15  y = −3  y = −3
   

Sumant les dues primeres equacions obtenim que 25x = −25 i per tant x = −1
x =−1
4 x − 3z = 5 ⎯⎯⎯
Substituint en l’equació → 4  ( −1) − 3z = 5 → −4 − 3z = 5 →
→ −3z = 5 + 4 → −3z = 9 → z = −3

I per tant el punt de tall entre la recta i el pla serà: Q = ( −1, −3, −3)

NOTA: Una manera molt original de trobar el punt Q és aprofitar que en un moment de
l’exercici hem expressat la recta r mitjançant les seves equacions paramètriques.
 x = 2 + 3

Recuperant aquestes equacions tenim: r :  y = −3
 z = 1 + 4

Això vol dir que qualsevol punt de la recta r és de la forma ( 2 + 3, −3,1 + 4 ) .
Si volem intersecar la recta amb el pla  : 3x + 4z + 15 = 0 només necessitem substituir:
3  ( 2 + 3 ) + 4  (1 + 4 ) + 15 = 0 → 6 + 9 + 4 + 16 + 15 = 0 → 25 + 25 = 0 →  = −1
I per tant:
( 2 + 3 , − 3, 1 + 4 ) ⎯⎯⎯
 =−1
→ Q = ( −1, −3, −3)
Tornar a l’enunciat

C/ Pasqual i Batlle, 1-15 08790 Gelida Telèfon: 93 779 04 50 Pàg. 151


e-mail: a8035246@xtec.cat https://agora.xtec.cat/iesgelida
Generalitat de Catalunya
Departament d’Educació
Institut Gelida Departament de Matemàtiques

111) PAU 2019 Sèrie 4 Qüestió 2:


x = 2
Siguin la recta r: i el pla  : x − z = 3 .
y − z =1
a) Calculeu l’equació paramètrica de la recta que és perpendicular al pla  i que el
talla en el mateix punt en què el talla la recta r.

Primer haurem de calcular el punt de tall entre la recta r i el pla :


 x = 2 x = 2
r :  
 y − z =1 y − z =1
 : x − z = 3  x − z = 3
 
x=2
x − z = 3 ⎯⎯→ 2 − z = 3 → 2 − 3 = z → z = −1
z =−1
y − z = 1 ⎯⎯⎯ → y − ( −1) = 1 → y + 1 = 1 → y = 0

Per tant, el punt de tall entre r i  és P = ( 2,0, −1)

Ara hem de calcular l’equació paramètrica de la recta s perpendicular a  :x−z =3 i


que passa pel punt P = ( 2,0, −1) .
x = 2 + 

s ⊥  → vs = n = (1,0, −1) → s : ( x, y, z ) = ( 2,0, −1) +  (1,0, −1) → s :  y = 0
 z = −1 − 

b) Trobeu els punts de r que estan a una distància de8 unitats del pla  .
NOTA: Podeu calcular la distància que hi ha d’un punt de coordenades ( x0 , y0 , z0 ) al
Ax0 + By0 + Cz0 + D
pla d’equació Ax + By + Cz + D = 0 amb l’expressió .
A2 + B 2 + C 2

Buscarem un punt genèric de la recta r i obligarem a que la distància d’aquest punt al


pla  sigui 8.

x = 2 x = 2 x = 2
r: ⎯⎯ z =
⎯→ → → ( x, y, z ) = ( 2, 1 +  ,  )
y − z =1 y −  =1 y =1+ 
Sigui P = ( 2, 1 +  ,  ) un punt genèric de r, obliguem ara a que la distància entre P i

el pla  sigui 8 .
d ( P,  ) = 8 → d ( ( 2, 1 +  ,  ) ,  : x − z = 3) = 8 →
2− −3 − − 1
→ d ( ( 2, 1 +  ,  ) ,  : x − z − 3 = 0 ) = 8 → = 8→ = 8→
12 + 02 + ( −1)
2
1+ 0 +1
− − 1  − − 1 = 4
→ = 8 → − − 1 = 8  2 → − − 1 = 4 → − − 1 = 4 →  →
2 − − 1 = −4

C/ Pasqual i Batlle, 1-15 08790 Gelida Telèfon: 93 779 04 50 Pàg. 152


e-mail: a8035246@xtec.cat https://agora.xtec.cat/iesgelida
Generalitat de Catalunya
Departament d’Educació
Institut Gelida Departament de Matemàtiques

−1 − 4 =   = −5 P = ( 2, 1+  ,  )  P = ( 2, −4, −5 )
→ → ⎯⎯⎯⎯⎯ → 
−1 + 4 =   = 3 Q = ( 2, 4, 3)

NOTA: Cal vigilar amb la forma en que ens donin el pla. Per poder utilitzar la fórmula de
la distància entre un punt P = ( x0 , y0 , z0 ) i un pla  : Ax + By + Cz + D = 0 necessitem
que tots els termes de  estiguin a l’esquerra de l’igual, és a dir, si utilitzéssim 
expressat com  : Ax + By + Cz = − D el raonament seria incorrecte.
Tornar a l’enunciat

112) PAU 2019 Sèrie 4 Qüestió 5:


Considereu els plans 1 : 2 x + ay + z = 5 ,  2 : x + ay + z = 1 i
 3 : 2 x + ( a + 1) y + ( a + 1) z = 0 , en què a és un paràmetre real.
a) Estudieu per a quins valors del paràmetre a els tres plans es tallen en un punt.

Els plans es tallaran en un únic punt sii el sistema que formen és compatible determinat.
Estudiem la matriu ampliada del sistema.

2 a 1 5
 
A' =  1 a 1 1
 2 a +1 a +1 0
 

Aprofitant que la matriu de coeficients és quadrada treballarem per determinants.

2 a 1 
  Desenvolupant a 1 2 1 2 a
A = 1 a 1  ⎯⎯⎯⎯⎯
3a fila
→ A = 2  − ( a + 1) + ( a + 1) =
 2 a + 1 a + 1 a 1 1 1 1 a
 

= 2  0 − ( a + 1)  1 + ( a + 1)  a = 0 − a − 1 + a 2 + a = a 2 − 1

A = 0  a 2 − 1 = 0  a 2 = 1  a = 1  a = 1

Per tant els 3 plans es tallaran en un únic punt sempre que a sigui diferent de 1 i de -1.

b) Comproveu que per al cas a = 1 la interpretació


geomètrica del sistema format per les equacions dels
tres plans és la que es mostra en la imatge següent:

2 a 1 5  2 1 1 5
  a =1  
A' =  1 a 1 1  ⎯⎯→ A ' =  1 1 1 1  
 2 a +1 a +1 0  2 2 2 0
   

C/ Pasqual i Batlle, 1-15 08790 Gelida Telèfon: 93 779 04 50 Pàg. 153


e-mail: a8035246@xtec.cat https://agora.xtec.cat/iesgelida
Generalitat de Catalunya
Departament d’Educació
Institut Gelida Departament de Matemàtiques

 1 1 1 1  F →F −2F  1 1 1 1 
F1  F2
  2 2 1 
  2 1 1 5    0 −1 −1 3  → Rang ( A) = 2  3 = Rang ( A ' ) → S.I . →
F →F −2F
 2 2 2 0  3 3 1  0 0 0 −2 
   

→ Els tres plans no tenen cap punt en comú.


Hi ha 4 posicions relatives entre 3 plans de manera que no tinguin cap punt en comú,
són les següents:

Cas 1 Cas 2 Cas 3 Cas 4


Dos plans
Tres plans que es
coincidents Dos plans paral·lels
Tres plans paral·lels tallen 2 a 2 formant
paral·lels a un i l’altre oblicu
rectes paral·leles
tercer

En els dos primers casos el paral·lelisme entre tots tres plans fa que Rang ( A) = 1 . Com
que en el nostre cas Rang ( A) = 2 podem descartar aquests dos casos.

Per tant ens queda saber si ens trobem en el cas 3 o en el cas 4. Ens hem de plantejar...
¿què diferencia aquests dos casos? La resposta és òbvia, en el cas 3 existeixen plans
paral·lels i en el cas 4 no. Per tant ens preguntem: Existeixen plans paral·lels?

Noteu que l’existència de plans paral·lels l’hem de mirar a la matriu A ' original perquè
les transformacions Gaussianes roten els plans i per tant trenquen el paral·lelisme.

 2 1 1 5
 
A ' =  1 1 1 1  on podem observar que les files 2 i 3 corresponen a plans paral·lels
 2 2 2 0
 
1 1 1 1
perquè = =  . Per tant, estem en el cas 3 que és el que correspon amb la
2 2 2 0
figura proposada.
Tornar a l’enunciat

C/ Pasqual i Batlle, 1-15 08790 Gelida Telèfon: 93 779 04 50 Pàg. 154


e-mail: a8035246@xtec.cat https://agora.xtec.cat/iesgelida

You might also like